patho evolve-cardiac/GI/muscular/renal

Ace your homework & exams now with Quizwiz!

Upon assessment a nurse found tophi. What did the nurse observe? 1 Kyphosis 2 Yellowish sclera 3 Spots that coalesce in a malar rash 4 Small, white nodules visible through the skin

4

Which information from the client indicates teaching by the nurse was successful for identifying one major modifiable risk factor for bladder cancer? 1 Age 2 Race 3 Gender 4 Cigarette smoking

4

Which assessment finding should the nurse expect to find in a client with cirrhosis of the liver? 1 Ascites 2 Hyperglycemia 3 Pain radiating to the back 4 Difficulty digesting starch and fat

1 Cirrhosis is the most common cause of ascites. Hypoglycemia occurs in cirrhosis. Hyperglycemia, pain radiating to the back, and difficulty digesting starch and fats indicate pancreatitis, not cirrhosis of the liver.

When assessing intentional tremors, what will the nurse observe when a client has myoclonus? 1 Spasm of a muscle 2 Loss of facial expressions 3 Irregular flapping movement 4 Uncoordinated muscle movemen

1 Myoclonus is defined as a muscle spasm in which there is throwing movements of a limb. Hypomimesis occurs when there is an inability to express emotions in facial expressions. Asterixis is an irregular flapping movement of the hands. Dyspraxia is uncoordinated motor movements.

A client has renal insufficiency. How should the nurse interpret this finding? 1 A decline in renal function, to about 25% of normal, has occurred. 2 Dialysis or a transplant is needed, because about 90% of renal function is lost. 3 Glomerular filtration rate has decreased more than 75%. 4 Renal function is less than 10%

1

A client has stress-induced muscle tension and is using biofeedback as a treatment. Which piece of equipment should the nurse obtain? 1 An electromyograph 2 An electrocardiograph 3 An electroencephalograph 4 An electrodiagnostograph

1

A client is suspected to have osteoarthritis. Which diagnostic test is used commonly to diagnose osteoarthritis? 1 Radiological studies (x-ray) 2 Computed tomography (CT) 3 Dual x-ray absorptiometry (DXA) 4 Magnetic resonance imaging (MRI

1

A client with diabetes mellitus has polyuria. Which process contributes to the polyuria? 1 Increased glucose in the urine 2 Production of abnormal ketones 3 Loss of protein across the glomerular membrane 4 Reduced antidiuretic hormone (ADH) response caused by insulin deficienc

1

A nurse is caring for a client with a condition that causes the left atrium to work harder during systole. The nurse is caring for which client? 1 A client with mitral stenosis 2 A client with aortic stenosis 3 A client with tricuspid stenosis 4 A client with pulmonary stenosis

1

A nurse is caring for a client with a greenstick fracture. The nurse is most likely caring for which client? 1 A boy, age 8 2 A young woman, age 19 3 A young woman, age 23 4 A man, age 34

1

Upon assessment, a client is experiencing difficulty in recognizing a pencil. Based on the nurse's observations, the client is exhibiting which condition? 1 Agnosia 2 Aphasia 3 Dysphasia 4 Dystoni

1

Which manifestation would the nurse expect to observe in the client diagnosed with rheumatoid arthritis? 1 Joint stiffness in the morning 2 Low body temperature 3 Increased appetite 4 Weight gain

1

A client has pancreatic insufficiency. Which assessment finding is typical for this client? 1 Steatorrhea 2 Bloody stools 3 Curling ulcer 4 Cushing ulcer

1 A large amount of fat in the stool (steatorrhea) is the most common sign of pancreatic insufficiency. Bloody stools occur in individuals with ulcerative colitis. Curling ulcers are stress ulcers that occur in individuals that are severely burned. Cushing ulcers are stress ulcers that occur in individuals that have severe head trauma or brain surgery.

A patient reports pain in the right shoulder and abdomen. Which organ does the nurse think is affected? 1 Liver 2 Heart 3 Appendix 4 Rectum

1 A patient with liver or biliary disorder will experience referred pain in the right shoulder and right abdomen. Cardiac disorders are associated with pain in the left shoulder and arm. Presence of pain in the right shoulder does not indicate that the patient has appendicitis (right lower quadrant) or rectal diseases (lower quadrant, midline).

client has a subarachnoid hemorrhage with a positive Kernig sign. How should the nurse interpret these findings? 1 The client has meningeal irritation. 2 The client has sciatic pain. 3 The client has photophobia. 4 The client has an aura

1 A positive Kernig sign (straightening the knee with the hip and knee in a flexed position produces pain in the back and neck regions) may appear, indicating meningeal irritation. Sciatic pain can occur with a herniated intervertebral disk. Photophobia is sensitivity to light. An aura can occur in migraines or seizures.

nurse is using the R-I-F-L-E acronym as classification criteria for which client? 1 A client with acute kidney injury 2 A client with urinary tract infection 3 A client with chronic pyelonephritis 4 A client with nephrotic syndrom

1 Classification criteria have been developed to guide the diagnosis of acute kidney injury and are described by the acronym RIFLE (R = risk, I = injury, F = failure, L = loss, and E = end-stage kidney disease [ESKD]), representing three levels of renal dysfunction of increasing severity. R-I-F-L-E is used for acute kidney injuries, not for urinary tract infections, chronic pyelonephritis, or nephrotic syndrome.

The nurse should consider which pathophysiologic process when planning care for a client with rheumatoid arthritis (RA)? 1 T cell abnormalities 2 IgE-mediated antibodies 3 Slower aging of telomeres 4 Immune complex nonbindin

1 A surprising new discovery is the presence of T cell abnormalities in individuals with RA, indicating a defect in telomere repair that may result in faster aging, not slower, of telomeres and consequent less efficient immune function. Rheumatoid factors (RFs) bind with their target self-antigens in blood and synovial membrane, forming immune complexes (antigen-antibody complexes). The RFs usually consist of two classes of immunoglobulin antibodies (antibodies for IgM and IgG) but occasionally involve antibodies for IgA (not IgE).

A client has an epidural hematoma. While planning care, the nurse recalls that the bleeding occurs between the dura mater and which other structure? 1 Skull 2 Brain 3 Pia mater 4 Arachnoid mate

1 An epidural hematoma occurs between the dura mater and the skull. Bleeding between the dura mater and the brain is a subdural hematoma. Bleeding between the dura mater and the pia mater or arachnoid mater are not termed epidural hematoma.

nurse is assessing a client for a suspected diagnosis of fibromyalgia. Which clinical manifestation will help establish a diagnosis for fibromyalgia? 1 Pain that has lasted for 3 months or more 2 Pain located above the waist 3 Tenderness in the groin area 4 Tenderness in 5 of the 18 "trigger" points

1 An individual must report pain that has been present for 3 months or more. Tenderness in 11 of the 18 "trigger" or "tender" points occurs when pressure is applied to those areas. Tender points include areas on both sides of the spine and both above and below the waist. There are no tender points located in the groin area.

Which statement by the client indicates teaching was successful regarding ankylosing spondylitis? 1 The disease is more common in men. 2 The disease in women is usually more severe. 3 The disease in women is predominantly in the spine. 4 The disease in men affects predominantly synovial joint

1 Ankylosing spondylitis is more common in men than in women. This disease is less severe in women. In women it is predominant in the peripheral joints. Rheumatoid arthritis affects the synovial joints, and ankylosing spondylitis causes excessive bone formation.

Which olfactory dysfunction does the nurse suspect in a child whose parent reports that the child does not react to the smell of any aromatic substances such as perfumes or flowers? 1 Anosmia 2 Parosmia 3 Hyposmia 4 Olfactory hallucination

1 Anosmia is a state of olfactory dysfunction in which the patient completely loses the ability to smell. Parosmia is characterized by abnormal sense of smell. Hyposmia is a condition characterized by impaired sense of smell. Olfactory hallucination is a condition in which the patient smells odors that are actually not present in the surroundings.

Which client is at highest risk of pyelonephritis and therefore needs close monitoring for signs of its occurrence? 1 A woman who has cystitis 2 A woman who is pregnant 3 A man who is a paraplegic 4 A man who has glomerulonephritis

1 Anyone who has cystitis is at high risk of developing pyelonephritis. The infection is probably usually spread by ascending uropathic microorganisms along the ureters, but spread also may occur by way of the bloodstream. Most cases of pyelonephritis occur in women. Although pregnancy does increase the risk of pyelonephritis, the client with cystitis has an even higher risk due to an active infection. Although stasis of urine with paraplegia does increase the risk of pyelonephritis, the client with cystitis has an even higher risk due to an active infection. Glomerulonephritis is not an infection and does not present the highest risk in this situation.

A client has fibromyalgia. Which information should the nurse include in the teaching? 1 Aerobic exercise is helpful. 2 Infection is usually the cause. 3 This is a deteriorating condition. 4 Stress can make this life threatening

1 Assist the individual to use aerobic exercise to reduce stress and increase rapid eye movement sleep. Explain that fibromyalgia presumably is not caused by infection, is not a deforming or deteriorating condition, and is neither life-threatening nor markedly debilitating. Widespread muscle pain and tenderness.

A client has osteoporosis. Which treatment will the nurse prepare to administer? 1 Bisphosphonates 2 Chelation 3 Antibiotics 4 Hyperbaric oxygen therapy

1 Bisphosphonates are first-line medications for treating osteoporosis; they primarily work by inhibiting hydroxyapatite breakdown, reducing bone resorption. Chelation of bone aluminum can be used in osteomalacia, but not in osteoporosis. Antibiotics and hyperbaric oxygen therapy are given for osteomyelitis, not osteoporosis.

A client has inflammation of the sacs that overlie bony prominences. The nurse is caring for which client? 1 A client with bursitis 2 A client with tendonitis 3 A client with epicondylitis 4 A client with osteomyelitis

1 Bursitis is inflammation of the bursae, which are sacs that protect the skin over bony protuberances. Tendonitis is inflammation of a tendon. Epicondylitis is not inflammation of the sacs that overlie bony prominences; an epicondyle is the bony prominence at the end of a bone where tendons or ligaments attach. Osteomyelitis is infection and inflammation in a bone.

Which client is most predisposed to superior vena cava syndrome? 1 A client with bronchogenic cancer 2 A client with lymphoma 3 A client with tuberculosis 4 A client with metastasi

1 Causes of superior vena cava syndrome include bronchogenic cancer (75% of cases), followed by lymphomas and metastasis of other cancers. Other less common causes include tuberculosis, mediastinal fibrosis, cystic fibrosis, and invasive therapies (pacemaker wires, central venous catheters, and pulmonary artery catheters).

A client has cholangiocellular carcinoma. The cancer is located in which area? 1 Bile ducts 2 Hepatocytes 3 Esophagus 4 Stomach

1 Cholangiocellular carcinoma develops in the bile ducts. Hepatocellular carcinoma develops in the hepatocytes. Esophageal cancer develops in the esophagus. Gastric adenocarcinoma develops in the stomach.

A nurse is caring for a client with chronic pyelonephritis. Which pathophysiologic mechanism should the nurse consider when planning care? 1 Diffuse scarring of one or both kidneys 2 Inflammation of the renal capsule 3 Hypertrophy of the renal tubules 4 Fibrosis around the kidne

1 Chronic pyelonephritis is a persistent or recurrent infection of the kidney leading to scarring of one or both kidneys. Inflammation and fibrosis occur in the interstitial spaces between the tubules, not in the capsule or around the kidney. Destruction, not hypertrophy, of the renal tubules occurs in chronic pyelonephritis.

An adult client presents with weakness of the facial muscles, impaired articulation of r, n, l, and raspy voice. On physical examination, the pharyngeal reflexes are diminished. Which diagnosis is supported by the assessment data? 1 Bulbar palsy 2 Tardive dyskinesia 3 Tourette syndrome 4 Locked-in syndrome

1 Clinical manifestations of bulbar palsy include paresis or paralysis of the jaw, face, pharynx, and tongue musculature, impaired articulation of r, n, l, and raspy voice with diminished pharyngeal reflexes. Clinical manifestations of tardive dyskinesia include involuntary muscle movements, not muscle weakness. Clinical manifestations of Tourette syndrome include multiple motor and vocal tics, not muscle weakness. Locked-in syndrome involves complete paralysis of voluntary muscles except for eye movement.

The nurse is evaluating a client with nephrotic syndrome. The nurse assesses for which clinical manifestation of protein loss related to nephrotic syndrome? 1 Edema 2 Bleeding 3 Low urine protein 4 Hyperthyroidism

1 Decreased oncotic pressure from protein loss leads to edema. Nephrotic syndrome is associated with hypothyroidism and hypercoagulability, leading to clotting, not bleeding. Nephrotic syndrome causes increased levels of urine protein, not low levels.

A client abuses alcohol and has cardiomyopathy. The nurse is caring for which client? 1 A client with dilated cardiomyopathy 2 A client with restrictive cardiomyopathy 3 A client with obstructive cardiomyopathy 4 A client with hypertrophic cardiomyopath

1 Dilated cardiomyopathy typically occurs from alcohol abuse. Restrictive, obstructive, and hypertrophic cardiomyopathy typically have other causes.

A nurse is caring for a client with cystitis and a client with pyelonephritis. Which assessment finding will be typical of both clients? 1 Dysuria 2 Fever and chills 3 Costovertebral angle tenderness 4 Suprapubic pain

1 Dysuria, or painful urination, is present with both disorders. Fever and chills do not usually occur with cystitis. Costovertebral angle tenderness occurs only with pyelonephritis, because the kidneys are located at the costovertebral angle. Suprapubic pain occurs only with cystitis, because the bladder is located just behind the pubic bone.

A client is admitted with suspected acute pancreatitis. Which elevated laboratory result is most diagnostic for acute pancreatitis? 1 Lipase 2 Amylase 3 Aminotransferase 4 Alanine transaminas

1 Elevated serum lipase level is the primary diagnostic marker for acute pancreatitis. Although amylase rises in response to pancreatitis, it is less specific. Aminotransferase and alanine transaminase levels are more diagnostic for hepatic (liver) disease.

"The amylase is sky-high," says a nurse in morning report. "Better go see this client first to check the pain level." Pain management is likely an issue for this client because of which pathophysiologic process? 1 Elevated amylase indicates pancreatic enzymes are autodigesting the pancreas. 2 Elevated amylase leads to secretion of histamine that causes itching and severe burning. 3 Elevated amylase is a false neurotransmitter that indicates end-stage cirrhosis with hepatic encephalopathy. 4 Elevated amylase is an enzyme that normally is secreted as an inactive proenzyme and digests fat, causing pain.

1 Elevation of amylase is characteristic of acute pancreatitis, which causes pancreatic enzymes to autodigest the pancreas. Amylase does not cause histamine release. Amylase is not a false neurotransmitter. Amylase does not digest fat, nor is it secreted as a proenzyme.

A client with gout has an acute attack. What is the pathophysiologic process for the development of this acute attack? 1 Formation of crystals 2 Formation of pannus 3 Release of pannus into the synovial fluid 4 Release of crystals into the synovial fluid

1 Evidence suggests that an acute attack of gout is the result of the formation of crystals rather than the release of crystals from connective tissues into the synovial fluid. Pannus occurs with rheumatoid arthritis, not gout.

Which type of fracture would most likely occur in an adult client that ran every day? 1 Fatigue fracture 2 Pathologic fracture 3 Greenstick fracture 4 Transchondral fracture

1 Fatigue fractures are caused by abnormal stress or torque applied to a bone with normal ability to recover and usually occur in joggers, skaters, dancers, or military recruits. Pathologic fractures generally are a result of bone weakness caused by another disease like cancer or infection or osteoporosis. A greenstick fracture perforates one cortex and splinters the spongy bone. The name is derived from the damage sustained by a young tree branch (a green stick) when it is bent sharply. The outer surface is disrupted, but the inner surface remains intact. This type of fracture commonly occurs in children and the elderly. Transchondral fractures consist of fragmentation and separation of articular cartilage that covers the end of the bone at a joint and are most prevalent in adolescents.

A client has diverticulosis. When the nurse takes the history, which dietary situation may be expected? 1 Low residue 2 Iron deficiency 3 Lactose deficiency 4 High intake of dietary fiber

1 Habitual consumption of a low-residue diet reduces fecal bulk, thus reducing the diameter of the colon and promoting diverticulosis. Iron deficiency can lead to anemia, not diverticulosis. There is no such clinical or dietary condition as "lactose deficiency," because lactose (a sugar found in milk) is not a mandatory component of a healthy diet. Lactase deficiency, on the other hand (lack of the enzyme that enables the digestion of lactose), is the cause of lactose intolerance. But it is not associated with diverticulosis. The treatment for diverticulosis is a high intake of dietary fiber; therefore a diet high in fiber would not lead to diverticulosis.

A nurse is talking about a hepatitis that can be acquired from ingesting contaminated food or water and may cause acute fever, abdominal pain, and jaundice but does not lead to chronic hepatitis. Which hepatitis is the nurse describing? 1 Hepatitis A 2 Hepatitis B 3 Hepatitis C 4 Hepatitis D

1 Hepatitis A is acquired from ingesting contaminated food or water and may cause acute fever, abdominal pain, and jaundice. Individuals with hepatitis A generally make a full recovery. Hepatitis B, C, and D can cause chronic liver disease.

The nurse is assessing a patient who has entered anaphylactic shock. Which physiologic condition does the nurse expect to see in the patient? 1 Hypotension 2 Hyperglycemia 3 Bronchodilation 4 Urinary retention

1 Histamine levels are high in a patient in patients who have entered anaphylactic shock, resulting in vasodilation of blood vessels, which in turn increases the blood flow to the area and leads to hypotension. Elevated histamine levels do not decrease insulin levels and do not cause hyperglycemia. Histamine causes constriction of the bronchial smooth muscles and results in bronchospasm. Histamine does not cause urinary retention.

Which pathophysiologic process should the nurse remember when planning care for a client with a herniated intervertebral disk? 1 Nucleus pulposus extrudes and compresses the nerve root. 2 Gelatinous material inside the cord hardens and obstructs blood flow. 3 Degeneration of the spine involves the lamina or neural arch of the vertebra. 4 Vertebra slides forward possibly causing a fracture of the pars interarticularis

1 In a herniated disk, the ligament and posterior capsule of the disk are usually torn, allowing the gelatinous material (the nucleus pulposus) to extrude and compress the nerve root. The gelatinous material is the nucleus pulposus. Spondylolysis is a structural defect (degeneration or developmental defect) of the spine involving the lamina or neural arch of the vertebra. Spondylolisthesis occurs when a vertebra slides forward and onto the vertebra below it, and may include a fracture of the pars interarticularis.

During the planning of care for a pregnant woman with acute pyelonephritis, which pathologic mechanisms will the nurse expect to occur? 1 Ureteral relaxation caused by higher progesterone levels 2 Neurologic impairment interfering with normal bladder contraction 3 Deposition of circulating antigen-antibody immune complexes into the glomerulus 4 Persistent or recurrent infection of the kidney leading to scarring of one or both kidneys

1 In pregnancy the pathologic mechanisms involve dilation and relaxation of the ureters with hydroureter and hydronephrosis; these are partly caused by obstruction from the enlarged uterus and partly by higher progesterone levels. Neurologic impairment interfering with normal bladder contraction, with residual urine and ascending infection, is common in neurogenic bladder. Deposition of circulating antigen-antibody immune complexes into the glomerulus occurs in glomerulonephritis. Persistent or recurrent infection of the kidney leading to scarring of one or both kidneys is chronic pyelonephritis.

arents of an infant with which condition will need the most complex teaching about protecting their infant from infection? 1 Severe combined immunodeficiencies (SCIDs) 2 DiGeorge syndrome 3 Selective IgA deficiency 4 Bruton agammaglobulinemia

1 Individuals who have SCIDs have defective cell-mediated and humoral immunity, the most severe immunodeficiency. Individuals who have DiGeorge syndrome have defective cell-mediated immunity; antibody production may be diminished, but antibody response is normal. Individuals who have selective IgA deficiency have partially defective humoral-mediated immunity, but cell-mediated immunity is not impaired. Individuals who have Bruton agammaglobulinemia have defective humoral-mediated immunity, but cell-mediated immunity functions normally.

A client has diastolic heart failure. Which clinical manifestation will the nurse expect to find upon assessment? 1 Pulmonary edema 2 Elevated ejection fraction 3 Frequent infections 4 Bloody stools

1 Individuals with diastolic dysfunction present with dyspnea on exertion, fatigue, and evidence of pulmonary edema (inspiratory crackles on auscultation, pleural effusions). The ejection fraction is normal in diastolic heart failure. Frequent infections and bloody stools are not associated with diastolic heart failure.

A nurse reads in the chart that a client has asthenia. How should the nurse interpret this finding? 1 The client tires easily. 2 The client has paresis. 3 The client withdraws. 4 The client is feverish

1 Individuals with hypertonia or hypotonia tire easily, which is termed asthenia. Paresis is partial paralysis with incomplete loss of muscle power. Withdrawal can occur in depression or when drug use is ceased. Fever occurs when the client has an elevated temperature.

A nurse plans to help a client with diabetes mellitus to reduce the risk of coronary artery disease. What is the rationale for the nurse's action? 1 Diabetes causes endothelial damage. 2 Diabetes causes thinning of vessel walls. 3 Diabetes causes hyperhomocysteinemia. 4 Diabetes causes decreased inflammation

1 Insulin resistance and diabetes have multiple effects on the cardiovascular system, including endothelial damage, thickening of the vessel wall, and increased inflammation. Hyperhomocysteinemia occurs because of a genetic lack of the enzyme that metabolizes homocysteine (an amino acid) or because of a nutritional deficiency of folate, cobalamin (vitamin B12), or pyridoxine (vitamin B6).

A nurse plans to help a client with diabetes mellitus to reduce the risk of coronary artery disease. What is the rationale for the nurse's action? 1 Diabetes causes endothelial damage. 2 Diabetes causes thinning of vessel walls. 3 Diabetes causes hyperhomocysteinemia. 4 Diabetes causes decreased inflammation.

1 Insulin resistance and diabetes have multiple effects on the cardiovascular system, including endothelial damage, thickening of the vessel wall, and increased inflammation. Hyperhomocysteinemia occurs because of a genetic lack of the enzyme that metabolizes homocysteine (an amino acid) or because of a nutritional deficiency of folate, cobalamin (vitamin B12), or pyridoxine (vitamin B6).

Which client should the nurse monitor for possible upper urinary tract obstruction? 1 A client with a kidney stone 2 A client with a urethral stricture 3 A client with prostate enlargement 4 A client with bladder neck dyssynergi

1 Kidney stones may cause obstruction of the upper urinary tract. Urethral stricture, prostate enlargement, and bladder neck dyssynergia cause lower urinary tract obstruction.

Which client should the nurse monitor for possible upper urinary tract obstruction? 1 A client with a kidney stone 2 A client with a urethral stricture 3 A client with prostate enlargement 4 A client with bladder neck dyssynergia

1 Kidney stones may cause obstruction of the upper urinary tract. Urethral stricture, prostate enlargement, and bladder neck dyssynergia cause lower urinary tract obstruction.

While planning care for a client in chronic renal failure, what should the nurse consider about the client's calcium level? 1 Calcium will be abnormally low in the blood serum. 2 Calcium will be abnormally high in the blood serum. 3 Calcium will be abnormally high due to acidosis. 4 Calcium will be absorbed in larger amounts from the gastrointestinal tract.

1 Low serum calcium results from decreased absorption from the gastrointestinal tract. Hypocalcemia develops in chronic renal failure, not high levels. Acidosis also contributes to a negative calcium balance.

Which clinical manifestation should indicate to the nurse that the client is experiencing early intracranial hypertension? 1 Confusion 2 Bounding pulse 3 Small, reactive pupils 4 Increased pulse pressure

1 Mental status changes (e.g., confusion) are early indicators of intracranial hypertension (stage 2). Bounding pulse, small, reactive pupils, and increased pulse pressure are clinical manifestations of stage 3 intracranial hypertension.

Which information indicates the nurse has an accurate understanding of the pathophysiology of neurogenic shock? 1 Parasympathetic overstimulation and sympathetic understimulation 2 Sympathetic overstimulation and parasympathetic understimulation 3 Massive vasodilation and increased systemic vascular resistance 4 Massive vasoconstriction and decreased systemic vascular resistance

1 Neurogenic shock (sometimes called vasogenic shock) is the result of widespread and massive vasodilation that results from parasympathetic overstimulation and sympathetic understimulation. Neurogenic shock pathophysiology also includes decreased systemic vascular resistance (corresponding with the massive vasodilation).

A client has just developed acute kidney injury. Which assessment finding is common in the early stages? 1 Oliguria 2 Diuresis 3 Hematuria 4 Proteinuria

1 Oliguria begins within 1 day after a hypotensive event and lasts 1 to 3 weeks, but it may regress in several hours or extend for several weeks, depending on the duration of ischemia or the severity of injury or obstruction. Diuresis occurs in later stages of healing. Hematuria can occur with glomerulonephritis. Proteinuria occurs with nephrotic syndrome.

client has just developed acute kidney injury. Which assessment finding is common in the early stages? 1 Oliguria 2 Diuresis 3 Hematuria 4 Proteinuria

1 Oliguria begins within 1 day after a hypotensive event and lasts 1 to 3 weeks, but it may regress in several hours or extend for several weeks, depending on the duration of ischemia or the severity of injury or obstruction. Diuresis occurs in later stages of healing. Hematuria can occur with glomerulonephritis. Proteinuria occurs with nephrotic syndrome.

A nurse is teaching about oxytocin. Which information from the client indicates teaching was successful? 1 It causes the let-down reflex. 2 It causes the uterus to relax. 3 It causes hypoglycemia. 4 It causes hypocalcemia

1 Oxytocin binds to its receptors on myoepithelial cells in the mammary tissues and causes contraction of those cells, which increases intramammary pressure and milk expression (let-down reflex). Oxytocin also acts on the uterus to stimulate contractions. Insulin, not oxytocin, produces hypoglycemia. Calcitonin, not oxytocin, lowers serum calcium levels by inhibition of bone-resorbing osteoclasts.

A client has difficulty writing and using tools or utensils. Which term will the nurse use to describe this finding? 1 Apraxia 2 Dystonia 3 Akathisia 4 Hypomimesis

1 People with dyspraxia/apraxia have difficulty performing tasks requiring motor skills, including speaking, writing, using tools or utensils, playing sports, following instructions, and focusing. Hypomimesis manifests as aprosody—the loss of emotional language. Dystonia is the maintenance of an abnormal posture through muscular contractions. Akathisia is a special type of hyperactivity, causing a compulsion to move (usually the legs).

A client has atherosclerosis. Which clinical manifestation will the nurse find upon assessment? 1 Bruits 2 Varicose veins 3 Respiratory acidosis 4 Increased blood flow to extremities

1 Physical examination of a client with atherosclerosis may reveal bruits. Varicose veins are palpable tortuous veins that do not result from atherosclerosis. Respiratory acidosis occurs with respiratory problems that cause retention of carbon dioxide. Evidence of decreased (not increased) blood flow is typical in atherosclerosis.

A nurse is caring for a client that has the most common classification of acute kidney injury. The nurse is caring for which client? 1 One with prerenal injury 2 One with postrenal injury 3 One with intrarenal injury 4 One with ultrarenal injur

1 Prerenal is the most common type of acute kidney injury and is caused by renal hypoperfusion. Intrarenal acute kidney injury results from ischemia within the kidney and is not as common as prerenal. Postrenal acute kidney injury is rare. There is no such classification as ultrarenal.

A nurse is caring for clients with various gastrointestinal symptoms. Which assessment finding will the nurse expect to observe? 1 Projectile vomiting in a client with pyloric stenosis 2 Hyperkalemia in a client with vomiting 3 Numerous hard stools in a client with diarrhea 4 Diarrhea in a client with Hirschsprung disease

1 Projectile vomiting can occur in pyloric stenosis. Vomiting will lead to hypokalemia, not hyperkalemia. Liquid stools, not hard, occur with diarrhea. Constipation occurs with Hirschsprung disease.

A nurse is caring for a child who has a disease similar to osteomalacia. The nurse is caring for which client? 1 A client with rickets 2 A client with osteopenia 3 A client with Paget disease 4 A client with osteosarcoma

1 Rickets is similar to osteomalacia in that it is caused by vitamin D deficiency and leads to soft, deformable bones in children. Osteopenia is a loss of bone density. Paget disease is a state of increased metabolic activity in bone characterized by abnormal and excessive bone remodeling. Osteosarcoma is a malignant neoplasm in bone, that is, a bone cancer.

A client is diagnosed with a kidney stone that is composed of magnesium, ammonium, and phosphate. What type of urinary calculus is this? 1 Struvite 2 Calcium 3 Cystine 4 Uric aci

1 Struvite stones are composed of magnesium, ammonium, and phosphate. Calcium oxalate or calcium phosphate stones account for 70% to 80% of stones. Less common stone elements include cystine and xanthine. Uric acid stones occur in persons who excrete excessive uric acid in the urine.

ent with acquired immunodeficiency syndrome (AIDS) has a decreasing CD4+ count. How should the nurse interpret this finding? 1 The number of helper T cells is decreasing. 2 Antibodies to HIV are being produced. 3 The client is having a positive response to antiviral drug therapy. 4 Neutrophils are being released in response to the HIV infection

1 The CD4+ cell count measures the number of helper T cells in a blood sample. Thus the major immunologic finding in AIDS is the striking decrease in the number of CD4-positive (CD4+) Th cells. A decreasing CD4+ count indicates progression of the disease process. The CD4+ count increases as antiviral drug therapy controls HIV infection. The CD4+ count does not measure antibodies to HIV or the number of neutrophils.

Which assessment finding leads the nurse to conclude that the client is experiencing early increased intracranial pressure? 1 Restlessness 2 Unresponsiveness 3 Inability to move 4 Inability to verbalize

1 The client may have episodes of confusion, restlessness, drowsiness, and slight pupillary and breathing changes during early increased intracranial pressure. Unresponsiveness, inability to move, and inability to verbalize are signs that occur later.

An older adult diagnosed with osteoporosis lives alone and falls in the bathtub. The person experiences tremendous left hip/leg pain with any movement and is not found until two days later. Initial laboratory work reveals: serum creatine kinase (CK) = 6000 units/l and potassium = 7.0. What is the highest priority? 1 Assist with a hemodialysis catheter insertion 2 Notify family members who do not live nearby 3 Consult with an orthopedic surgeon to stabilize hip/leg 4 Decrease intravenous fluids and monitor urinary output

1 The highest priority listed is to assist with a hemodialysis catheter insertion so hemodialysis may be performed immediately for the hyperkalemia and the serum CK level, indicating rhabdomyolysis. Once CK levels exceed 5000 units/l, acute renal failure is likely. Rapid intravenous, not decreased, hydration maintains adequate kidney flow. Consulting the orthopedic surgeon is correct, but is not the highest priority. Notifying the family is also correct, but not the highest priority at this time.

Which organism is the most common cause of osteomyelitis in clients with sickle cell disease? 1 Salmonella 2 Group B streptococci 3 Haemophilus influenzae 4 Staphylococcus aureus

1 The most common cause of osteomyelitis in clients with sickle cell disease is Salmonella. Group B streptococci and H influenzae tend to affect young children. The most common hematogenous cause overall is Staphylococcus aureus.

A nurse is caring for a client that has retching and vomiting. Which assessment finding will the nurse observe when the parasympathetic system is activated during the vomiting? 1 Copious salivation 2 Tachycardia 3 Diaphoresis 4 Tachypnea

1 The parasympathetic system mediates copious salivation, increased gastric motility, and relaxation of the upper and lower esophageal sphincters. A diffuse sympathetic discharge causes the tachycardia, tachypnea, and diaphoresis that accompany retching and vomiting.

Which pathophysiologic process should the nurse remember when planning care for a woman with gestational diabetes? 1 Insulin resistance combined with inadequate insulin secretion in relation to hyperglycemia 2 Genetic abnormalities that decrease the functioning of alpha cells in the pancreas 3 Cellular-mediated autoimmune destruction of pancreatic beta cells 4 Absolute insulin deficiency

1 The pathophysiology of gestational diabetes is insulin resistance combined with inadequate insulin secretion in relation to hyperglycemia. Genetic abnormalities occur with genetic disorders, not gestational diabetes. Autoimmune destruction and absolute insulin deficiency occur with type 1 diabetes.

A client has acute pancreatitis. When the nurse is planning care, which pathophysiologic process should the nurse remember? 1 Autodigestion of the pancreas 2 Walled-off cysts in the pancreas 3 Decreased serum lipase levels 4 Destruction of hepatocytes

1 The pathophysiology of pancreatitis is autodigestion of the pancreas. Walled-off cysts occur in chronic hepatitis. Elevated serum lipase level, not decreased, is the primary diagnostic marker for acute pancreatitis. Hepatocytes are found in the liver, not in the pancreas.

A nurse is caring for a client with osteoarthritis. What is the primary pathophysiologic feature of osteoarthritis? 1 Loss of articular cartilage 2 Thinning of the joint capsule 3 Increase of synovial fluid in the joint 4 Stress fractures at the end of the bone

1 The primary defect in osteoarthritis is loss of articular cartilage. There is thickening of the joint capsule, not thinning. There could be increase of synovial fluid during inflammation but it is not the primary defect. There are no stress fractures typically associated with osteoarthritis.

The nurse is caring for a client experiencing acute kidney injury (AKI). The chart notes that the client's urine output is increasing significantly and that the serum creatinine level has decreased and normalized. The client is in which phase of AKI? 1 Recovery 2 Initiation 3 Remediation 4 Maintenance

1 The recovery phase is the interval when kidney injury is repaired and normal renal function is reestablished. Diuresis (output is increasing significantly) is common during this phase, with a decline in serum creatinine and urea concentrations and an increase in creatinine clearance. Initiation is the time that the kidney injury is evolving. Maintenance occurs once the damaging incident is over and may last several weeks to months. During maintenance the urine output is the lowest and serum creatinine and blood urea nitrogen levels both increase. There is no remediation phase.

nurse is teaching an adult class about the major risk factor for developing bladder cancer. Which information should the nurse include? 1 Smoking 2 Bacteriuria 3 Urethral obstruction 4 Exposure to coal dus

1 The risk of primary bladder cancer is greater among people who smoke or are exposed to metabolites of aniline dyes or other aromatic amines. Bacteriuria and urethral obstruction are not major risk factors for developing bladder cancer. Exposure to coal dust is not a major risk factor for developing bladder cancer; coal dust can lead to respiratory problems.

A client has urinary calculi. While planning care, the nurse considers which most important factor in the formation of renal calculi? 1 Urine pH 2 Urine sodium levels 3 Serum lactate levels 4 Temperature of the urine

1 The temperature and pH of the urine influence the risk of precipitation and calculus formation, and pH is most important. Urine sodium levels and serum lactate levels do not have a major influence on formation of renal calculi.

A nurse is caring for a client who returned from a transsphenoidal surgical removal of a growth hormone (GH)-secreting adenoma. The nurse is caring for which client? 1 A client with acromegaly 2 A client with diabetes insipidus 3 A client with prolactinoma 4 A client with hypothyroidism

1 The treatment of choice in acromegaly is transsphenoidal surgical removal of the GH-secreting adenoma. Diabetes insipidus can be treated with oral hydration or antidiuretic hormone replacement, not transsphenoidal surgery. Dopaminergic agonists (bromocriptine and cabergoline) are the treatment of choice for prolactinomas. Hormone replacement therapy with the hormone levothyroxine is the treatment of choice for hypothyroidism.

client's serum sodium level is 150 mEq/L. The nurse expects which changes in the client's serum chloride and bicarbonate levels? Select all that apply. 1 Serum chloride, 115 mEq/L 2 Serum bicarbonate, 18 mEq/L 3 Serum chloride, 92 mEq/L 4 Serum bicarbonate, 30 mEq/L 5 Serum chloride, 100 mEq/L 6 Serum bicarbonate, 25 mEq/L

12 A serum sodium level greater than 145 mEq/L is hypernatremia. Increased serum chloride levels (greater than 105 mEq/L) and decreased serum bicarbonate levels (less than 24 mEq/L) occur in clients with hypernatremia. Chloride changes follow a direct relationship to changes in serum sodium. Bicarbonate levels are decreased in this situation, especially if the client is experiencing metabolic acidosis. Normal serum chloride is 103 mEq/L. Normal serum bicarbonate is 24 mEq/L.

A patient with cancer has a rapid pulse rate, blood pressure of 170/110 mm Hg, sweating, and tremors. Which tumor marker will help diagnose the patient's condition? 1 Blood catecholamine 2 Blood alpha-fetoprotein 3 Blood prostate-specific antigen 4 Blood carcinoembryonic antige

1 Tumor markers help to screen, identify, and follow the clinical course of a tumor. Pheochromocytoma is a benign tumor of the adrenal medulla. It produces an excess of adrenaline, which leads to a rapid pulse rate, hypertension, sweating, and tremors. The detection of elevated catecholamine in the blood and urine helps to confirm and treat the disease. The alpha-fetoprotein level helps to identify whether the patient has liver or germ cell cancer. In contrast, determining the level of prostate-specific antigen in the blood helps to identify whether the patient has a risk of prostate tumor. Tumors of the pancreas, lung, and breast produce carcinoembryonic antigens.

A client with glomerulonephritis has presence of blood with red cell casts, white cell casts, and very mild degrees of protein in the urine. How should the nurse describe this finding? 1 The client has nephritic sediment. 2 The client has nephrotic sediment. 3 The client has staghorn sediment. 4 The client has struvite sediment

1 Urine with a nephritic sediment is characterized by the presence of blood with red cell casts, white cell casts, and varying degrees of protein, which usually is not severe. Urine with nephrotic sediment contains massive amounts of protein and lipids and either a microscopic amount of blood or no blood. Struvite are stones that primarily contain magnesium-ammonium-phosphate as well as varying levels of matrix. Struvite calculi may grow quite large and branch into a staghorn configuration.

A client's glomerular filtration rate decreases by 28%. The nurse will report that the client is in which category for acute kidney dysfunction? 1 Risk 2 Loss 3 Injury 4 Failur

1 Using the RIFLE criteria (risk, injury, failure, loss, end-stage renal disease) for kidney dysfunction, a glomerular filtration rate (GFR) decrease of 25% up to 50% characterizes the "risk" category. The "injury" category indicates a decrease in GFR of 50% up to 75%; "failure" indicates a decrease in GFR of over 75%; and "loss" is complete loss of kidney function greater than 4 weeks.

A client has a central nervous system injury and starts to vomit. Which condition does the nurse suspect is causing the vomiting? 1 Injury involving the vestibular nuclei 2 Injury resulting in paraplegia of the extremities 3 Injury causing a decrease in intracranial pressure 4 Injury impinging directly on the floor of the third ventricle

1 Vomiting often accompanies central nervous system injuries that involve the vestibular nuclei. Vomiting occurs when there is increased, not decreased, intracranial pressure and when the floor of the fourth ventricle, not the third, is directly involved. Paraplegia does not necessarily lead to vomiting, like an injury to the vestibular nuclei would.

nurse is teaching an adult community group how to prevent West Nile virus infection. Which information should the nurse include? 1 Wear protective clothing when outside 2 Take the vaccine on a yearly basis 3 Drink bottled water when traveling 4 Avoid touching a person with the virus

1 West Nile virus is spread by infected mosquitoes, so protective clothing can help prevent the disease. No West Nile vaccine has been developed for humans. Bottled water does not help in prevention of West Nile virus transmission. West Nile virus is not spread through casual contact such as touching or kissing a person with the virus.

To reduce stress-induced muscle tension, a client is attempting to control maladaptive tension using integrated electromyography (EMG). What term should the nurse use to describe this treatment when giving report to the oncoming shift? 1 Yoga 2 Meditation 3 Biofeedback 4 Progressive relaxation training

3

Which measurement will alert the nurse that the client has developed portal hypertension? Record the answer as a whole number: At least _______________ mm Hg.

10 Portal hypertension is an increase to at least 10 mm Hg. Portal hypertension is abnormally high blood pressure in the portal venous system. Pressure in this system is normally 3 mm Hg.

Which age group should the nurse monitor that has the highest risk for bone tumors? 1 Infants 2 Children 3 Adolescents 4 Adults between ages 30 and 35 years

3

client with chronic kidney disease has anemia. While planning care, the nurse should consider whether there is a deficiency of which hormone? 1 Aldosterone 2 Angiotensin I 3 Erythropoietin 4 Parathyroid

3

A client experiences trauma and repetitive stress/overuse on the knee joint. Which conditions are associated with this repetitive stress or overuse? Select all that apply. 1 Bursitis 2 Osteoarthritis 3 Osteomyelitis 4 Pompe disease 5 Ankylosing spondylitis

12 Bursitis is an inflammatory response that may be seen after trauma and overuse. Osteoarthritis usually occurs in those persons who put exceptional stress on joints, such as obese persons, gymnasts, or long-distance runners (e.g., marathoners). Osteomyelitis is a bone infection most often caused by bacteria, not by stress. Ankylosing spondylitis occurs from a misfolding of histocompatibility antigen human leukocyte antigen, not from stress. Pompe disease is an infantile form of acid maltase deficiency, causing abnormal glycogen metabolism.

A nurse is preparing educational pamphlets about the differences between Crohn disease and ulcerative colitis. Which characteristics should be placed in a section that is specific to ulcerative colitis? Select all that apply. 1 Mucosal layer involved 2 Lesions usually continuous 3 High risk for fistula formation 4 Caused by Helicobacter pylori infection 5 Lesions usually discontinuous 6 Deep lesions involving the entire intestinal wa

12 Characteristics specific to ulcerative colitis include involvement of the mucosal layer and shallow lesions that are usually continuous. Characteristics specific to Crohn disease are a high risk of fistula formation, discontinuous lesions, and deep lesions involving the entire intestinal wall. Helicobacter pylori infection is not pertinent to ulcerative colitis or Crohn disease but to peptic ulcer disease.

Which diet would the nurse suggest for a client with nephrotic syndrome? Select all that apply. 1 Low-fat 2 Salt-restricted 3 Low-protein 4 High-protein 5 High-carbohydrate

12 Nephrotic syndrome is commonly treated by adhering to a normal-protein (i.e., 1 g/kg/day, where body weight is measured in kilograms), low-fat, salt-restricted diet. Protein should be neither high nor low. Carbohydrates should not be high.

hich principles should the nurse remember when caring for a client with spinal shock? Select all that apply. 1 Poor venous circulation occurs below the lesion. 2 It involves all skeletal muscles below the lesion. 3 No disruption of thermal control occurs below the lesion. 4 It causes increased muscle tone below the lesion. 5 It is characterized by an incomplete loss of reflex function below the lesion

12 Spinal shock does involve dysfunction of skeletal muscles, resulting in complete or incomplete paralysis below the level of injury, as well as poor venous circulation. Such an injury is characterized by impairment of control of thermal regulation, flaccidity, and a complete loss of reflex function below the level of the lesion. The hypothalamus cannot regulate body heat through vasoconstriction and increased metabolism; therefore the individual assumes the temperature of the air (poikilothermia).

The nurse admits a client with severe nausea, vomiting, and 10/10 pain radiating from the right upper abdominal quadrant to the back. A cholecystectomy was performed four years ago. Based upon clinical signs and laboratory results, a diagnosis of pancreatitis was made. The client has now developed tetany. What are the pathophysiologic mechanisms occurring in this client to produce tetany? Select all that apply. 1 Calcium is deposited in areas of fat necrosis 2 Decreased response to parathormone 3 Glucagon is released from damaged alpha cells 4 Bowel hypomotility with accumulation of fluids 5 Pancreatic edema pressing on the duct

12 Tetany may develop as a result of hypocalcemia, when calcium is deposited in areas of fat necrosis or as a decreased response to parathormone. Transient hyperglycemia, not tetany, can occur if glucagon is released from damaged alpha cells in the pancreatic islets. Abdominal distention, not tetany, accompanies bowel hypomotility and the accumulation of fluids in the peritoneal cavity. Jaundice, not tetany, can occur from obstruction of the bile duct (e.g., a gallstone) or from pancreatic edema pressing on the duct.

hich joints should the nurse assess because they are typically affected first in rheumatoid arthritis? Select all that apply. 1 Fingers 2 Wrists 3 Hips 4 Knees 5 Ankle

1212 Initially the joints most commonly involved are the metacarpophalangeal joints, proximal interphalangeal joints (fingers), and wrists, with later involvement of larger weight-bearing joints (hips, knees, ankles).

Which clinical manifestations will the nurse expect to find in a client that has rhabdomyolysis? Select all that apply. 1 Dark urine 2 Weakness 3 Muscle pain 4 Metabolic alkalosis 5 Decreased creatine kinase

123 A classic triad of muscle pain, weakness, and dark urine is considered typical of rhabdomyolysis, but those affected may have no complaint of pain or muscle weakness. Creatine kinase is released in massive quantities and can be severely elevated, not decreased. Metabolic alkalosis does not occur; rather, metabolic acidosis occurs from liberation of intracellular phosphorus and sulfate.

A nurse is teaching about the most common causes of cirrhosis. Which information should the nurse include? Select all that apply. 1 Alcoholism 2 Hepatitis C 3 Idiopathy 4 Hemachromatosis 5 Antibiotic overdose

123 A nurse is teaching about the most common causes of cirrhosis. Which information should the nurse include? Select all that apply. 1 Alcoholism 2 Hepatitis C 3 Idiopathy 4 Hemachromatosis 5 Antibiotic overdose

A nurse is teaching about the types of hepatitis that can be acquired sexually and cause chronic hepatitis. Which types should the nurse include? Select all that apply. 1 Hepatitis B 2 Hepatitis C 3 Hepatitis D 4 Hepatitis E 5 Hepatitis G

123 Hepatitis B, D, and C are acquired sexually and can lead to chronic hepatitis. Hepatitis E is spread by fecally contaminated waters in endemic areas. GB virus C, formerly known as hepatitis G virus, is a blood-borne virus and is not a significant cause of human liver disease.

A nurse monitors a client with malignant hypertension for which life-threatening complications? Select all that apply. 1 Cardiac failure 2 Encephalopathy 3 Cerebrovascular accident 4 First-degree heart block 5 Acute pericarditis

123 Malignant hypertension can cause cerebrovascular accident, encephalopathy, and cardiac failure due to high systolic and diastolic pressures and the resulting high hydrostatic pressures in the capillaries. Malignant hypertension is not associated with first-degree heart block or acute pericarditis.

A nurse is teaching about the risk factors for osteoarthritis. Which information should the nurse include in the teaching session? Select all that apply. 1 Steroids 2 Hemophilia 3 Ballet dancing 4 Low calcium intake 5 Weight below healthy rang

123 Risk factors for osteoarthritis are long-term mechanical stress like ballet dancing, hemophilia from chronic bleeding into the joints, and drugs like steroids. Weight below healthy range and low calcium intake are risk factors for osteoporosis, not osteoarthritis.

nurse is caring for a client with a giant cell tumor. Which information should the nurse consider when planning care for this client? Select all that apply. 1 It causes bone resorption. 2 It is a type of myelogenic tumor. 3 It affects females more than males. 4 It is a very fast growing type of cancer. 5 It has a very narrow age distribution

123 The giant cell tumor, a type of myelogenic tumor, is the sixth most common bone tumor and is more common in females. It is a solitary, circumscribed tumor that causes extensive bone resorption. It has a wide age distribution and has a slow, relentless growth rate.

A nurse is developing a plan of care for a client diagnosed with osteoarthritis. Which treatments should the nurse include in the client's plan of care? Select all that apply. 1 Range of motion exercises 2 Rest of the affected joint 3 Use of analgesics 4 Weight gain 5 Radiation

123 Treatments for osteoarthritis include range of motion exercises, rest of the affected joint, use of analgesics, and weight loss, not weight gain. Radiation is used for cancer, not osteoarthritis.

he nurse is caring for a client who is diagnosed with Alzheimer disease. Rank the clinical manifestations in the correct order from expected early clinical manifestations to late clinical manifestations. 1. Forgetfulness, loss of initiative, and impaired judgment 2. Memory loss increases, disorientation, and confusion 3. Incontinent, inability to perform self-care, and unclear language 4. Nonambulatory/bedbound, unable to eat, and no significant cognitive function

1234 Early stage clinical manifestations include forgetfulness, loss of initiative, and impaired judgment. Middle stage clinical manifestations include increased memory loss, disorintation, and confusion. Late stage clinical manifestations include incontinence, inability to perform self-care, and unlcear language. End-stage clinical manifestations include being nonambulatory/bedbound, being unable to eat, and having no significant cognitive function.

Which information from a client indicates teaching by a nurse was successful about risk factors for liver cancer? Select all that apply. 1 Exposure to mycotoxins 2 Obesity 3 Alcohol abuse 4 Cirrhosis 5 Hepatitis A virus

1234 Liver cancer, hepatocellular carcinoma, has the following risk factors: exposure to mycotoxins, alcohol abuse, obesity, cirrhosis, and hepatitis B, C, and D viruses. Hepatitis A does not lead to liver cancer.

A nurse is teaching an adult community group about the risk factors of gastric cancer. Which information should the nurse include? Select all that apply. 1 Infection with Helicobacter pylori 2 Consumption of heavily salted foods 3 Low intake of fruits and vegetables 4 Use of tobacco and alcohol 5 Presence of polyps

1234 The most important environmental causative factors of gastric cancer are (1) infection with Helicobacter pylori that carries the CagA gene product cytotoxin-associated vacuolating antigen A (VacA), (2) consumption of heavily salted and preserved foods (e.g., nitrates in pickled or salted foods such as bacon), (3) low intake of fruits and vegetables, and (4) use of tobacco and alcohol. Presence of polyps is associated with colorectal cancer, not gastric cancer.

Which treatment options should the nurse include in the plan of care for clients with rheumatoid arthritis (RA)? Select all that apply. 1 Nonsteroidal anti-inflammatory drugs 2 Occupational therapy 3 Assistive devices 4 Physical therapy 5 Methotrexate 6 Antibiotics

12345 Treatment options for RA include nonsteroidal anti-inflammatory drugs, glucocorticoids, occupational and physical therapy, and assistive devices. Methotrexate remains the first line of treatment for RA. Antibiotics are given for osteomyelitis, not RA.

A client has an abnormality in endocrine functioning. Which factors may cause endocrine abnormalities? Select all that apply. 1 Faulty feedback system 2 Gland dysfunction 3 Altered metabolism of the hormones 4 Hormone production by nonendocrine tissues 5 Adequate quantity of hormone precursor

1234Elevated or depressed hormone levels result from (1) faulty feedback systems, (2) dysfunction of a gland or glands, (3) altered metabolism of hormones, or (4) production of hormones from nonendocrine tissues. An inadequate, not adequate, quantity of hormone precursors can cause an abnormality in endocrine functioning.

Which clinical manifestations will alert the nurse that the client with type 1 diabetes is experiencing hypoglycemia? Select all that apply. 1 Coma 2 Confusion 3 Tachycardia 4 Diaphoresis 5 Ketoacidosis 6 Kussmaul respiration

1234Hypoglycemia can cause the following symptoms for those with type 1 diabetes: pallor, tremor, anxiety, tachycardia, diaphoresis, dizziness, fatigue, poor judgment, confusion, and coma. Ketoacidosis most often occurs with type 1 diabetes and is from hyperglycemia. Kussmaul respirations occur with ketoacidosis.

A nurse is teaching about the causes of acute tubular necrosis. Which information should the nurse include? Select all that apply. 1 Severe burns 2 Antibiotics that are nephrotoxic 3 Disseminated intravascular coagulation 4 Contrast dyes used for radiologic studies 5 Increased ammonia levels from liver failur

1234Severe burns can be a cause of acute tubular necrosis. Certain antibiotics, including aminoglycosides, are nephrotoxic and can be a direct cause of acute tubular necrosis. Vascular disease such as disseminated intravascular coagulation can also lead to acute tubular necrosis. Radiocontrast media used for radiologic studies occasionally can be nephrotoxic, causing acute tubular necrosis. Liver failure with elevated ammonia levels causes toxicity to the central nervous system but does not cause acute tubular necrosis.

nurse is teaching about the possible causes of acute kidney injury. Which information should the nurse include? Select all that apply. 1 Cardiac failure 2 Neurogenic bladder 3 Renal artery occlusion 4 Chronic glomerulonephritis 5 Childbirth complications

1235 Cardiac failure, neurogenic bladder, renal artery occlusion, and childbirth complications are possible causes of acute kidney injury. Chronic glomerulonephritis is most often associated with chronic kidney disease.

A client has severe glomerulonephritis. Which findings are common when the nurse performs an assessment? Select all that apply. 1 Edema 2 Hematuria 3 Proteinuria 4 Hypotension 5 Smoky brown-tinged urin

1235 Edema, hematuria, proteinuria, and smoky brown-tinged urine are all common findings in severe glomerulonephritis. Hypertension, not hypotension, occurs in glomerulonephritis.

A client has cirrhosis. Which assessment findings will the nurse expect to observe? Select all that apply. 1 Dark urine 2 Spider angiomas 3 Light-colored stools 4 Increased body hair 5 Bleeding tendency

1235 Hepatic failure causes dark urine, spider angiomas, light-colored stools, and bleeding tendencies. There is loss of body hair in cirrhosis, not increased body hair.

A client has acute pancreatitis. Which findings will the nurse typically observe? Select all that apply. 1 Fever 2 Tetany 3 Jaundice 4 Bradycardia 5 Bowel hypomotility 6 Pain radiating to the groi

1235 Jaundice can occur from obstruction of the bile duct (e.g., a gallstone) or from pancreatic edema pressing on the duct. Fever and leukocytosis accompany the inflammatory response. Abdominal distention accompanies bowel hypomotility and the accumulation of fluids in the peritoneal cavity. Tetany may develop as a result of hypocalcemia when calcium is deposited in areas of fat necrosis or as a decreased response to parathormone. The cardinal manifestation of acute pancreatitis is epigastric or midabdominal constant pain ranging from mild abdominal discomfort to severe, incapacitating pain. The pain may radiate to the back, not the groin. Tachycardia occurs, not bradycardia.

A nurse is planning management goals for a client with acute kidney injury. Which goals should the nurse include? Select all that apply. 1 Treating infections 2 Maintaining nutrition 3 Correcting fluid disturbances 4 Monitoring the kidney transplant 5 Remembering that drug excretion is impaired

1235 The four main management principles of acute kidney injury are correcting fluid and electrolyte disturbances, treating infections, maintaining nutrition, and remembering that excretion of drugs or their metabolites is impaired. Kidney transplant is used in the management of chronic kidney disease.

A nurse is taking care of clients with fractures and monitors the "5 Ps" of compartment syndrome. Which areas did the nurse assess? Select all that apply. 1 Pain 2 Pallor 3 Paresis 4 Pannus 5 Paresthesia 6 Pulselessness

12356 The "5 Ps" of compartment syndrome are pain (out of proportion to expected injury), pallor, paresthesia, paresis (of the involved extremity), and pulselessness. Pannus is involved in the pathophysiology of rheumatoid arthritis.

A nurse is preparing a presentation on disorders commonly associated with damage from alcohol overuse. What disorders should be included? Select all that apply. 1 Gastritis 2 Cirrhosis 3 Pancreatitis 4 Appendicitis 5 Cholecystitis 6 Esophageal cance

1236 Overuse of alcohol causes damage that can lead to gastritis, cirrhosis, esophageal cancer, and pancreatitis. Appendicitis and cholecystitis are not commonly associated with alcohol overuse.

A nurse is preparing a presentation on disorders commonly associated with damage from alcohol overuse. What disorders should be included? Select all that apply. 1 Gastritis 2 Cirrhosis 3 Pancreatitis 4 Appendicitis 5 Cholecystitis 6 Esophageal cancer

1236 Overuse of alcohol causes damage that can lead to gastritis, cirrhosis, esophageal cancer, and pancreatitis. Appendicitis and cholecystitis are not commonly associated with alcohol overuse.

The nurse plans to teach a client who is diagnosed with type 1 diabetes mellitus about treatment regimens. Which treatments should the nurse include in the teaching plan? Select all that apply. 1 Insulin therapy 2 Meal planning 3 Exercise schedule 4 C-5 supplements Incorrect5 Oral hypoglycemic drugs

123All individuals with type 1 diabetes require some combination of insulin therapy, meal planning, and exercise regimen. C-peptide is a component of proinsulin and is indicative of residual beta-cell mass and function, but it is not currently a medication or supplement. Oral hypoglycemic drugs are used to treat type 2 diabetes, not type 1.

A client has migraines. The nurse collaborates with the primary healthcare provider for management of migraines by implementing which treatments? Select all that apply. 1 Allowing sleep 2 Applying ice 3 Administering drug prophylaxis 4 Encouraging physical activity 5 Using fluorescent ligh

123Allowing sleep, applying ice, darkening the room, and administering drug prophylaxis can be part of migraine management. Physical activity and fluorescent lights have not been found to manage migraines.

nurse is teaching an adult community health group about risk factors for renal cell carcinoma. Which information should the nurse include? Select all that apply. 1 Obesity 2 Cigarette smoking 3 Uncontrolled hypertension 4 Low dietary intake of sodium 5 Exposure to allergen

123Obesity, cigarette smoking, and uncontrolled hypertension all are risk factors for renal cell carcinoma. Low dietary intake of sodium is not a risk factor for renal cell carcinoma. Allergens do not cause renal cell carcinoma.

What clinical manifestations should the nurse expect to observe in a client with chronic pancreatitis? Select all that apply. 1 Abdominal pain 2 Hyperglycemia 3 Weight gain 4 Steatorrhea 5 Odynophagi

124 Abdominal pain is seen with chronic pancreatitis and may be intermittent or continuous. Hyperglycemia is seen with destruction of the islet of Langerhans cells, leading to insulin-dependent diabetes that requires treatment. Steatorrhea is seen in late-stage chronic pancreatitis. Weight loss, not weight gain, is seen in chronic pancreatitis. Odynophagia (pain on swallowing) may be initiated from cancer of the esophagus, not chronic pancreatitis.

What clinical manifestations should the nurse expect to observe in a client with chronic pancreatitis? Select all that apply. 1 Abdominal pain 2 Hyperglycemia 3 Weight gain 4 Steatorrhea 5 Odynophagia

124 Abdominal pain is seen with chronic pancreatitis and may be intermittent or continuous. Hyperglycemia is seen with destruction of the islet of Langerhans cells, leading to insulin-dependent diabetes that requires treatment. Steatorrhea is seen in late-stage chronic pancreatitis. Weight loss, not weight gain, is seen in chronic pancreatitis. Odynophagia (pain on swallowing) may be initiated from cancer of the esophagus, not chronic pancreatitis.

A nurse hears in report that a client with diabetes mellitus has polydipsia, polyuria, and polyphagia. Which findings does the nurse expect to find upon assessment of this client? Select all that apply. 1 Increased thirst 2 Increased hunger 3 Increased weight gain 4 Increased urine output 5 Increased blood pressure

124 Because of elevated blood glucose levels, water is osmotically attracted from body cells, resulting in intracellular dehydration and stimulation of thirst in the hypothalamus (polydipsia). Hyperglycemia acts as an osmotic diuretic; the amount of glucose filtered by the glomeruli of the kidneys exceeds that which can be reabsorbed by renal tubules; glycosuria results, accompanied by large amounts of water lost in the urine (polyuria). Depletion of cellular stores of carbohydrates, fats, and protein results in cellular starvation and a corresponding increase in hunger (polyphagia). Weight loss occurs because of fluid loss in osmotic diuresis and loss of body tissue as fats and proteins are used for energy. Increased blood pressure is called hypertension.

What clinical manifestations are associated with osteoporosis? Select all that apply. 1 Kyphosis 2 Decreased height 3 Frequent infections 4 Femoral neck fracture 5 Increased bone densit

124 Manifestations of osteoporosis include decreased (not increased) bone density, kyphosis, decreased height, and fractures of long bones including neck of the femur, vertebrae, ribs, and distal radius. Frequent infections do not occur with osteoporosis.

Which complications should the nurse monitor for in a client with appendicitis? Select all that apply. 1 Peritonitis 2 Perforation 3 Hiatal hernia 4 Abscess formation 5 Intestinal obstruction

124 Perforation, peritonitis, and abscess formation are the most serious complications of appendicitis. Hiatal hernia and intestinal obstruction are not results of appendicitis.

An older adult with osteoporosis lives alone. What are the risk factors for osteoporosis? Select all that apply. 1 Early menopause (natural or surgical) 2 Prolonged corticosteroid (prednisone) use 3 Exercised extensively during middle age 4 A sedentary smoker who drinks alcohol 5 Being outside in the sun for long periods of time

124 The risk factors for osteoporosis include: hormonal and metabolic (early menopause, natural or surgical); drugs (corticosteroids); lifestyle (sedentary behavior, smoking, and excessive alcohol consumption); genetic; anthropometric; and dietary (excessive protein). Excessive exercise is not a risk factor, but not exercising is. The sun provides vitamin D to the body, which helps with the absorption of calcium, decreasing the risk for osteoporosis. Sun exposure places a person at risk for skin cancer, not for osteoporosis.

A nurse is teaching about the pathophysiology of rheumatoid arthritis. Which cell types should the nurse include in the teaching session that are associated with the development of rheumatoid arthritis? Select all that apply. 1 B cells 2 T cells 3 Platelets 4 Neutrophils 5 Macrophages 6 Red blood cells

1245 Both T and B cells play a role in the autoimmune response, as well as neutrophils and macrophages. Red blood cells and platelets do not play a role.

A client is diagnosed with gout after presenting with pain in the big toe. Which clinical manifestations should the nurse expect to find upon assessment? Select all that apply. 1 Hyperuricemia 2 Tophi formations 3 Increased blood clotting 4 Formation of renal stones 5 Inflammation of a single join

1245 Gout is manifested by an increase in serum urate concentration (hyperuricemia), recurrent attacks of monarticular arthritis (inflammation of a single joint), deposits of monosodium urate monohydrate (tophi) in and around the joints, and the formation of renal stones. Increased coagulation is not associated with this disorder.

A nurse is teaching a women's group about the symptoms of coronary artery disease and myocardial infarction for women. Which clinical manifestations should the nurse include? Select all that apply. 1 Sudden fatigue 2 Mild back discomfort 3 Pain down the left arm 4 Sense of unease 5 Palpitation

1245 Sudden fatigue, mild back discomfort, a sense of unease, and palpitations are all typical symptoms for women. Atypical pain, not pain down the left arm, usually presents in women.

A urinary tract obstruction may affect which anatomic structures of a client's urinary tract? Select all that apply. 1 Urethra 2 Bladder 3 Prostate 4 Ureters 5 Renal pelvi

1245 The urinary tract consists of the renal parenchyma, including the renal pelvis, ureters, bladder, and urethra. Obstructions can develop at any point in the urinary system. Obstructions are often categorized as upper (renal parenchyma, ureters) or lower (bladder, urethra). The prostate is often a cause of urinary tract obstruction and is not considered a part of the urinary tract.

lient has suspected appendicitis. Which assessment findings would help confirm the diagnosis? Select all that apply. 1 Fever 2 Nausea 3 Vertigo 4 Diarrhea 5 Rebound tenderness present 6 Elevated white blood cell coun

12456

A nurse is caring for a client with acromegaly. Which clinical manifestations will the nurse observe upon assessment? Select all that apply. 1 Enlarged tongue 2 Interstitial edema 3 Fine body hair 4 Barrel-chested appearance 5 Diminished sebaceous and sweat glands

124With connective tissue proliferation, individuals with acromegaly have an enlarged tongue, interstitial edema, enlarged and overactive sebaceous and sweat glands (leading to increased body odor), and coarse (not fine) skin and body hair. Additionally, increased insulin-like growth factor 1 levels cause ribs to elongate at the bone-cartilage junction, leading to a barrel-chested appearance.

A nurse is caring for a client with Alzheimer disease. Which of these pathophysiologic changes are consistent with Alzheimer disease? Select all that apply. 1 Neuritic plaques 2 Acetylcholine loss 3 Epileptogenic focus 4 Focal cerebral edema 5 Neurofibrillary tangles 6 Excitation of neurons

125

A client has cholelithiasis. Which findings will the nurse typically find upon assessment? Select all that apply. 1 Jaundice 2 Biliary colic 3 Bloody diarrhea 4 Left lower quadrant pain 5 Fatty meals causing pain

125 Abdominal pain and jaundice are the cardinal manifestations of cholelithiasis. Vague symptoms include heartburn, flatulence, epigastric discomfort, and food intolerances, particularly to fats and cabbage. The pain (biliary colic) occurs 30 minutes to several hours after a fatty meal is eaten. Cholelithiasis pain usually occurs in the right upper quadrant, not the left lower.

A nurse is teaching the staff about types of shock that are characterized by generalized vasodilation and peripheral pooling of blood. Which types of shock should the nurse include? Select all that apply. 1 Septic 2 Neurogenic 3 Cardiogenic 4 Hypovolemic 5 Anaphylactic

125 Septic shock, neurogenic shock, and anaphylactic shock are characterized by excessive vasodilation and peripheral pooling of blood. Cardiogenic shock occurs primarily as a result of severe dysfunction of the left or right ventricles, or both, that results in inadequate cardiac pumping. Hypovolemic shock is a result of excessive blood loss; vasoconstriction is initially seen in this form of shock.

lient has cholelithiasis. Which findings will the nurse typically find upon assessment? Select all that apply. 1 Jaundice 2 Biliary colic 3 Bloody diarrhea 4 Left lower quadrant pain 5 Fatty meals causing pai

125 Abdominal pain and jaundice are the cardinal manifestations of cholelithiasis. Vague symptoms include heartburn, flatulence, epigastric discomfort, and food intolerances, particularly to fats and cabbage. The pain (biliary colic) occurs 30 minutes to several hours after a fatty meal is eaten. Cholelithiasis pain usually occurs in the right upper quadrant, not the left lower.

A nurse is teaching the staff about the pathologic features of osteoarthritis. Which information should the nurse include? Select all that apply. 1 Formation of bone spurs 2 Thickening of the joint capsule 3 Inflammation of the diaphysis 4 Stress fractures of the epiphysis 5 Loss and damage of articular cartilage 6 Changes in subchondral bone

1256 The pathologic features of osteoarthritis are formation of bone spurs, thickening of the joint capsule, degeneration of articular cartilage, and changes in subchondral bone tissue. Osteoarthritis does not cause inflammation of the diaphysis or stress fractures of the epiphysis.

A client is diagnosed with ankylosing spondylitis. Which assessment data support this diagnosis? Select all that apply. 1 Low back pain 2 A 20-year-old man 3 Normal back curvature 4 Pain lessens with rest 5 Daily bouts of indigestion 6 Restricted, painful forward flexion and rotatio

126 Ankylosing spondylitis presents with low back pain and has an onset in young men in their teens and early twenties. Forward flexion, rotation, and lateral flexion of the spine are restricted and painful. Physical examinations reveal a loss of the normal lordosis (curvature) of the spine. It is common for men to present with back pain relieved by exercise. Gastrointestinal involvement is not typical.

A nurse closely monitors a client with the syndrome of inappropriate antidiuretic hormone (SIADH) for which complications? Select all that apply. 1 Convulsions 2 Neurologic damage 3 Acute kidney injury 4 Panhypopituitarism 5 Myocardial infarction

12Severe hyponatremia in SIADH can cause convulsions and irreversible neurologic damage. Acute kidney injury (ischemia or damage to the kidney), panhypopituitarism (in which all hormones are deficient from the pituitary), and myocardial infarction (in which lack of oxygen to the heart causes muscle damage) are not complications of SIADH.

Which clinical manifestations will the nurse typically find upon assessment of a client with Paget disease? Select all that apply. 1 Headache 2 Increased lacrimation 3 Impaired motor function 4 Decreased density of skull 5 Increased sympathetic nervous system discharg

13 Headache is commonly noted, and there may be cranial nerve dysfunction with impaired motor function. Deafness, atrophy of the optic nerve, and obstruction of the lacrimal duct may occur. In Paget disease, bone remodeling occurs and is first evident in the thickening of the frontal and occipital regions of the skull. The sympathetic nervous system is not directly affected.

A client has dermatomyositis. The nurse is assessing for the presence of Gottron lesions. Which areas should the nurse check to determine their presence? Select all that apply. 1 Knees 2 Eyelids 3 Elbows 4 Ankles 5 Periorbital tissue

13 In dermatomyositis, Gottron lesions are erythematous, scaly lesions that cover joints such as the knees and elbows. A typical heliotrope (reddish purple) rash that generally covers the eyelids and periorbital tissue also occurs in dermatomyositis. The ankles are not a typical site for the lesions.

When a nurse is teaching about renal cell carcinoma, which information should the nurse include in the teaching session? Select all that apply. 1 It may cause flank pain. 2 It is usually bilateral. 3 Symptoms include hematuria. 4 Granular cell tumors have the best prognosis. 5 Early stages produce a large abdominal mass

13 Symptoms include hematuria, flank pain, and flank/abdominal mass. The tumors usually occur unilaterally, not bilaterally. Early stages are often silent, and clear cell tumors have a better prognosis than granular cell tumors.

A nurse is caring for clients that have obstructed urine flow. The nurse is caring for which clients? Select all that apply. 1 One with a urethral stricture 2 One with an overactive bladder 3 One with severe pelvic organ prolapse 4 One with prostate enlargement 5 One with low bladder wall compliance

134 Anatomic causes of obstructed urine flow include urethral stricture, pelvic organ prolapse, and prostate enlargement. Overactive bladder and low bladder wall compliance may develop if the blockage persists.

What physiologic processes must the nurse remember while planning care for a client with indirect healing of a fracture? Select all that apply. 1 Callus formation occurs. 2 Scar tissue is formed. 3 Solid bone is remodeled. 4 New bone is eventually formed. 5 Adjacent bone cortices are in direct contac

134 Indirect healing is characterized by callus formation, remodeling of solid bone, and eventual formation of new bone tissue. Direct healing occurs when adjacent bones' cortices are in contact with one another. Bone is one of the only tissue types that form new complete tissue, not scar tissue, when they heal. (Liver tissue is the other type with this capability.)

A nurse is caring for clients with edema. Which client conditions can lead to edema formation? Select all that apply. 1 Venous obstruction 2 Decreased circulatory blood volume 3 Increased salt and water retention 4 Decreased serum albumin production 5 Isotonic fluid los

134 Venous obstruction causes hydrostatic pressure to increase behind the obstruction, pushing fluid from the capillaries into the interstitial spaces (causing edema). Increased salt and water retention causes plasma volume overload, increased capillary hydrostatic pressure, and edema. Albumin is the primary plasma protein responsible for capillary oncotic pressure. Decreased albumin levels will lower capillary oncotic pressure and decrease reabsorption of fluid from the interstitial space into the vascular space, leading to edema. Isotonic fluid loss and decreased circulatory blood volume will decrease capillary hydrostatic pressure. Edema formation will not occur in these situations.

A nurse is teaching the staff about common causes of traumatic brain injury. Which causes should the nurse include in the teaching session? Select all that apply. 1 Falls 2 Surgery 3 Violence 4 Sports-related events 5 Transportation-related event

1345

A nurse is teaching about risk factors for urinary stones. Which information should the nurse include? Select all that apply. 1 Diet 2 Smoking 3 Occupation 4 Fluid intake 5 Geographic location

1345 The risk of urinary calculi formation is influenced by a number of factors, including age, gender, race, geographic location, seasonal factors, fluid intake, diet, and occupation. Smoking is not a risk factor for urinary stones. warm climate, metabolic disease that increase urine level of Ca, acid diet-lots of protein, fruit juice, tea, Ca genetic lifestyle-sdentary

A nurse is teaching about a fracture that is generally observed in children and the elderly. Which fracture is the nurse describing? 1 Pelvis 2 Stress 3 Greenstick 4 Transchondra

3 Greenstick fractures occur most often in the growing bones of children and in the elderly. Pelvis fractures occur most often in older adults. Stress fractures can occur at any age. Transchondral fractures are most prevalent in adolescents.

A client missed two dialysis sessions. Which assessment findings will the nurse most likely observe when the client attends the next dialysis session? Select all that apply. 1 Fatigue due to progressive accumulation of uremic toxins and low red blood cells 2 Positive Chvostek and Trousseau signs due to hypomagnesemia 3 Deep rapid breathing due to a compensatory mechanism for metabolic acidosis 4 Possible cardiac dysrhythmias due to hyperkalemia 5 Sallow skin color from retained urochrom

1345 A person who needs dialysis and has missed two sessions will have fatigue due to progressive accumulation of uremic toxins and low red blood cells. Deep rapid breathing occurs due to a compensatory mechanism for metabolic acidosis. Possible cardiac dysrhythmias may occur due to hyperkalemia. Sallow skin color results from retained urochrome. Positive Chvostek and Trousseau signs are due to calcium, not magnesium.

A nurse is presenting information about risk factors for colon and rectal cancer to an adult group at a community health clinic. Which factors should the nurse include? Select all that apply. 1 Aging 2 Gender 3 Inactivity 4 High-fat diet 5 Type 2 diabetes mellitu

1345 Aging increases the risk of colon cancer. Low levels of physical activity increase the risk. Dietary factors, such as a high-fat, low-fiber diet, increase the risk, as does type 2 diabetes mellitus. There is no current research to support a strong link between gender and the risk for colon cancer.

Upon assessment, the nurse determines a client with a spinal cord injury at T4 is experiencing autonomic hyperreflexia (dysreflexia). The nurse comes to this conclusion by observing which assessment findings? Select all that apply. 1 Headache 2 Tachycardia 3 Blurred vision 4 Upper body flushing 5 Paroxysmal hypertension 6 Paralysis of the extremitie

1345 Autonomic hyperreflexia (dysreflexia) is characterized by paroxysmal hypertension, blurred vision, body flushing, sweating above the level of injury, and pounding headache. The client may experience bradycardia, not tachycardia. Paralysis of the extremities is related to the spinal cord injury and is not a result of autonomic hyperreflexia (dysreflexia).

A nurse is conducting health screenings at a local clinic. Which risk factors will place the individuals at a higher risk for hypertension? Select all that apply. 1 Obesity 2 Blood pressure of 110/70 3 High dietary sodium 4 Glucose intolerance 5 Positive family history for hypertension

1345 Hypertension is associated with positive family history, gender, black race, high dietary sodium, glucose intolerance, cigarette smoking, obesity, heavy alcohol consumption, and low dietary intake of minerals. Blood pressure of 110/70 is not a risk factor for hypertension; it is normal.

A nurse is teaching an adult community group about the predisposing factors associated with pyelonephritis. Which factors should the nurse include? Select all that apply. 1 Pregnancy 2 Viral infection 3 Catheterization 4 Neurogenic bladder 5 Urinary obstruction

1345 Predisposing factors of pyelonephritis include urinary obstruction, neurogenic bladder, instrumentation (e.g., catheterization), and pregnancy. Viral infection is not known to be a predisposing factor of pyelonephritis; bacterial infections can be a common cause. kidney stone; vesicoureteral reflux; preg; neurogenic bladder; instrumentation; female sexual traumA

A client has a lower urinary tract obstruction. Which findings will the nurse typically observe upon assessment? Select all that apply. 1 Nocturia 2 Flank pain 3 Urinary urgency 4 Intermittent urinary stream 5 Increased voiding frequency

1345 Symptoms of obstruction are more common in men and include (1) frequent daytime voiding (urination more than every 2 hours while awake); (2) nocturia (awakening more than once each night to urinate for adults less than 65 years of age or more than twice for older adults); (3) poor force of stream; (4) intermittency of urinary stream; (5) bothersome urinary urgency, often combined with hesitancy; and (6) feelings of incomplete bladder emptying despite micturition. Flank pain is a manifestation of an upper urinary tract disorder.

A nurse is monitoring a client with chronic kidney failure for neurologic effects. Which effects should the nurse assess for in this client? Select all that apply. 1 Stupor 2 Nausea 3 Fatigue 4 Twitching 5 Chest pain 6 Loss of vibration sense

1346 Neurologic effects from chronic kidney failure can result in stupor, fatigue, twitching, and loss of vibration sense. Nausea is a gastrointestinal effect. Chest pain is a cardiovascular effect.

A client is diagnosed with hyperosmolar hyperglycemic nonketotic syndrome (HHNKS). Which clinical manifestations will the nurse expect? Select all that apply. 1 Stupor 2 Ketonuria 3 Hypokalemia 4 Severe dehydration 5 Kussmaul respiration

134Clinical manifestations of HHNKS include severe dehydration, loss of electrolytes, including potassium, and neurologic changes, such as stupor. Ketonuria and Kussmaul respirations are clinical manifestations of diabetic ketoacidosis.

Which clients are exhibiting manifestations of an extrapyramidal motor syndrome? Select all that apply. 1 A client with a tremor 2 A client with quadraplegia 3 A client with cogwheel rigidity 4 A client with normal tendon reflexes 5 A client with hypertonia in extremities

134Extrapyramidal motor syndrome manifestations include cogwheel rigidity, normal tendon reflexes, and presence of a tremor. Paralysis and hypertonia in extremities are manifestations of a pyramidal motor syndrome.

What pathophysiologic changes does the nurse expect in a client that has upper urinary tract obstruction within 7 days? Select all that apply. 1 Increased urinary stasis 2 Reduced size of the medulla 3 Tubulointerstitial fibrosis 4 Increased metabolic acidosis 5 Dilation of the upper urinary tract

135 If an upper urinary tract obstruction is not resolved within 7 days, increased urinary stasis, tubulointerstitial fibrosis, and dilation of the upper urinary tract occur. If the obstruction has still not been resolved after 28 days, the medulla decreases in size and metabolic acidosis occurs.

A client who has diabetes and arterial insufficiency in the lower extremities had a sudden loss of ability to speak and was unable to move the left arm and leg. This condition lasted for 9 minutes and then resolved, with no apparent lasting effects. What terms should a nurse use with another health professional to describe this situation? Select all that apply. 1 Aphasia 2 Paraplegia 3 Left hemiplegia 4 Cerebrovascular accident 5 Transient ischemic attack

135 Loss of ability to speak is aphasia; inability to move the left arm and leg is left hemiplegia; reversal of these symptoms in 9 minutes makes them transient; altogether these observations describe a transient ischemic attack. The client does not have paraplegia, which is paralysis of the lower extremities. This client did not have a cerebrovascular accident, which would show no reversal of symptoms, owing to permanent loss of perfusion in a region of the brain.

Which clients should the nurse monitor for severe postobstructive diuresis after relief of a complete obstruction of one kidney? Select all that apply. 1 One with edema and weight gain 2 One with dehydration 3 One with hypertension 4 One with a high fever 5 One with nephrogenic diabetes insipidus

135 Risk factors for severe postobstructive diuresis include chronic, bilateral obstruction, impairment of one or both kidneys' ability to concentrate urine or reabsorb sodium (nephrogenic diabetes insipidus), hypertension, edema and weight gain, congestive heart failure, and uremic encephalopathy. Edema, not dehydration, is a risk factor for postobstructive diuresis. High fever can lead to dehydration but not to postobstructive diuresis.

client has chronic pancreatitis. Which actions will the nurse take? Select all that apply. 1 Offer pain relief 2 Reduce pancreatic enzyme 3 Encourage cessation of alcohol intake 4 Drain biliary tree secretions 5 Administer oral enzyme replacements

135 The treatment of acute pancreatitis includes the cessation of alcohol intake, the replacement of oral enzymes, and pain control. Pancreatic enzyme replacement, not reduction, is a component of care. Biliary tree drainage is not a component of care for this condition.

A nurse is assessing a client with acute kidney injury with oliguria. What are the possible mechanisms that could account for the decreased urine output? Select all that apply. 1 Back leak 2 Nerve damage 3 Tubular obstruction 4 Decreased fluid intake 5 Alterations in renal blood flow

135 Three mechanisms have been proposed to account for the decrease in urine output in acute kidney injury: back leak, tubular obstruction, and alterations in renal blood flow. In back leak, glomerular filtration remains normal, but tubular reabsorption of filtrate is accelerated as a result of permeability caused by ischemia. Necrosis of the tubules causes sloughing of cells, cast formation, or ischemic edema that results in tubular obstruction, increasing pressure and reducing glomerular filtration rate (GFR). Efferent arteriolar vasoconstriction may be produced, or there may be redistribution of blood flow from the cortex to the medulla. Autoregulation of blood flow may be impaired, resulting in decreased GFR. Nerve damage and decreased fluid intake are not considered mechanisms that may cause a decrease in urine output in acute kidney injury.

Which information should the nurse include when discussing attention deficit hyperactivity disorder (ADHD)? Select all that apply. 1 Impulsivity is often a characteristic. 2 It does not affect working memory. 3 It is a common childhood disorder. 4 The disorder generally goes into remission by adulthood. 5 Inability to maintain sustained attention is common.

135 Working memory, the ability to remember instructions and information needed to guide behavior, is affected by ADHD, and impulsivity is often a characteristic. ADHD is a common childhood disorder. It typically begins in childhood, and for 50% to 75% of those diagnosed, it continues throughout adolescence and adulthood. ADHD causes an inability to maintain sustained attention.

An obese adult client reports chronic fatigue and the urge to move legs at night. The partner reports excessive snoring with periods of not breathing at all. Because of severe claustrophobia, the client is unable to tolerate nasal continuous positive airway pressure devices. The nurse should monitor for which conditions in the client? Select all that apply. 1 Stroke 2 Hypocapnia 3 Peripheral edema 4 Systemic hypotension 5 Low oxygen saturation 6 Right-sided congestive heart failure

1356 Sleep apnea produces hypercapnia and low oxygen saturation and eventually leads to polycythemia, pulmonary hypertension, systemic hypertension, stroke, right-sided congestive heart failure, dysrhythmias, liver congestion, cyanosis, and peripheral edema. Sleep apnea does not lead to hypocapnia or systemic hypotension.

A nurse is teaching a women's community group about the risk factors for osteoporosis. Which information should the nurse include? Select all that apply. 1 Obesity 2 Ovarian cysts 3 Endometriosis 4 Early menopause 5 Early menarche

14

Which clients will be predisposed to cardiomyopathies as a result of remodeling? Select all that apply. 1 A woman with hypertension 2 A man with enlarged prostate 3 An infant with truncus arteriosus 4 A teenager with ischemic heart disease 5 An older person with a hip replacement

14

A client has a significant water deficit. Which assessment findings of dehydration will the nurse observe? Select all that apply. 1 Headache 2 Decreased heart rate 3 Increased skin turgor 4 Decreased urine volume 5 Increased blood pressure

14 Clinical manifestations of dehydration include headache and decreased urine output. Heart rate increases with water deficits. Skin turgor decreases in dehydration. Blood pressure decreases in dehydration from a water deficit.

A nurse is teaching a senior citizens' group about risk factors that predispose elderly persons to urinary tract infections. Which factors should the nurse include? Select all that apply. 1 Prostatitis 2 Sexual inactivity 3 Urinary incontinence 4 Decreased estrogen in women 5 Chronic illness such as diabete

145 Factors that predispose elderly individuals to urinary tract infections include chronic illness such as diabetes (whose effects tend to compromise immune response), decreased estrogen in women, and prostatitis. Urinary incontinence is not a contributing factor to the development of urinary tract infections. Sexual activity, not inactivity, is a contributing factor to urinary tract infections.

pg 11 ch 9 19/11A nurse is teaching about characteristics of malignant tumors. Which information should the nurse include in the teaching session? Select all that apply. 1 Metastasis 2 Slow growth 3 Low mitotic index 4 Invasion of surrounding tissues 5 Self-stimulation with growth factors

145 Malignant tumors often metastasize and invade surrounding tissues, and their cells often have autocrine (self-stimulating) growth factor stimulation. Cells in malignant tumors divide and proliferate rapidly. Benign tumors have slow growth and low mitotic index.

A nurse is teaching the staff about physiologic mechanisms that protect against urinary tract infections. Which information should the nurse include in the teaching session? Select all that apply. 1 Prostatitis 2 Acidic urine 3 Hypercalcemia 4 Tamm-Horsfall protein 5 Uroepithelial secretions

145 Several factors normally combine to protect against urinary tract infections. Most bacteria are washed out of the urethra during micturition. The low pH (acidic urine) and high osmolality of urea, the presence of Tamm-Horsfall protein, and secretions from the uroepithelium provide a bactericidal effect. Inflammation of the prostate (prostatitis) causes urinary stasis and increases the risk of urinary tract infection, whereas hypercalcemia promotes renal calculi and urinary stasis.

Which factors are required before a client forms a kidney stone? Select all that apply. 1 Aggregation 2 Low concentration of salt 3 Supersaturation of one of the salts 4 Precipitation from a liquid to a solid 5 Absence of Tamm-Horsfall protein

1456 Kidney stone formation is complex and related to supersaturation (not low concentration) of one or more salts in the urine, precipitation of the salts from a liquid to a solid state, growth through aggregation (crystallization), and the presence or absence of stone inhibitors (e.g., Tamm-Horsfall protein, if present, decreases stone formation).

A nurse is assessing a client with acute cystitis. Which assessment findings are typical for this client? Select all that apply. 1 Dysuria 2 Colic pain 3 Urinary stones 4 Urinary urgency 5 Urinary frequency 6 Pain in the suprapubic area

1456 Most clients with acute cystitis experience dysuria, urinary frequency and urgency, and pain in the suprapubic area or lower back, or both. Colic pain usually occurs with urinary stones, not with acute cystitis. Stones are not typical of acute cystitis.

hich terms should the nurse use to describe types of hypertonia? Select all that apply. 1 Dystonia 2 Asthenia 3 Flaccidity 4 Spasticity 5 Paratonia

145Dystonia, spasticity, and paratonia are types of hypertonia. Asthenia means to tire easily and can occur with both hypertonia and hypotonia. Flaccidity is decreased muscle tone or hypotonia.

What classic clinical manifestations will the nurse observe in a client diagnosed with Parkinson disease? Select all that apply. 1 Rigidity 2 Paralysis 3 Flaccidity 4 Bradykinesia 5 Resting tremor 6 Urinary incontinenc

145The classic manifestaions of Parkinson disease are resting tremor, rigidity, and bradykinesia/akinesia. Clients with Parkinson disease do not develop true paralysis or flaccidity; urinary retention occurs, not incontinence.

Which information from the staff indicates teaching by the nurse was successful for possible causes of upper urinary tract obstruction? Select all that apply. 1 Renal calculi 2 Hypertension 3 Hypocalcemia 4 Ureteral blockage 5 Renal artery thrombosis 6 Retroperitoneal fibrosis

146 Renal calculi, ureteral blockage, and the pressure from abdominal inflammation or scarring (retroperitoneal fibrosis) can impede urine flow, causing upper urinary tract obstruction. Renal artery thrombosis and hypertension can impede blood flow, but not urine flow. Hypocalcemia would cause a decrease in stone formation, whereas hypercalcemia could lead to renal calculi.

A client has been diagnosed with a pericardial effusion resulting in the presence of frank blood. The nurse considers which conditions could cause bloody effusion? Select all that apply. 1 Aneurysm 2 Neoplasm 3 Tuberculosis 4 Left heart failure 5 Coagulation defect

15 Effusions of frank blood are generally related to aneurysms, trauma, or coagulation defects. A serous effusion develops with overhydration and left heart failure. Serosanguineous effusion occurs with tuberculosis or neoplasm.

ient is diagnosed with Huntington disease. Which information will the nurse teach the client? Select all that apply. 1 It is a type of chorea. 2 It is a nonhereditary disease. 3 It is asymptomatic until the sixth decade of life. 4 It is a commonly diagnosed neurologic disorder. 5 Its principal pathologic feature is severe degeneration of the basal ga

15 Huntington disease is a type of chorea. The principal pathologic feature is severe degeneration of the basal ganglia, particularly the caudate nucleus. It is a relatively rare hereditary, degenerative disorder. It most commonly affects those between the ages of 25 and 45.

Which prescriptions from the primary healthcare provider should the nurse expect to receive for a client with chronic pancreatitis? Select all that apply. 1 Insulin 2 Radiation 3 Chemotherapy 4 Ursodeoxycholic acid 5 Oral enzyme replacement

15 To correct enzyme deficiencies and prevent malabsorption, oral enzyme replacements are taken before and during meals. Loss of islet cells can cause insulin-dependent diabetes and requires treatment. Radiation and chemotherapy are treatments for cancer, not for chronic pancreatitis. Ursodeoxycholic acid slows disease progression of primary biliary cirrhosis, not chronic pancreatitis.

A client has a transchondral fracture. Which information should the nurse consider when planning care for this client? Select all that apply. 1 They are most prevalent in adolescents. 2 They run parallel to the long axis of the bone. 3 They are a type of complete break in a bone. 4 They occur in normal or abnormal bone that is subjected to repeated stress. 5 They involve fragmentation and separation of a portion of the articular cartilage.

15 Transchondral fractures, most prevalent in adolescents, consist of a fragmentation and separation of a portion of the articular cartilage that covers the end of a bone or joint. A linear fracture runs parallel to the long axis of the bone. A transchondral break is a typical incomplete, not a complete, fracture. Stress fractures occur in normal or abnormal bone that is subjected to repeated stress.

Which conditions should the nurse assess for in a client with upper motor neuron lesions? Select all that apply. 1 Dyssynergia 2 Atonic bladder 3 Flaccid bladder 4 Detrusor areflexia 5 Overactive bladder 6 Loss of bladder sensation

15 Upper motor neuron lesions result in an overactive (hyperreflexive) bladder and dyssynergia (loss of coordinated neuromuscular contraction). Lesions in the sacral area of the spinal cord or peripheral nerves result in detrusor areflexia, underactive, hypotonic, or atonic (flaccid) bladder function, often with loss of bladder sensation.

The nurse admits a client with severe nausea, vomiting, and 10/10 pain radiating from the right upper abdominal quadrant to the back. A cholecystectomy was performed four years ago. The client developed chronic pancreatitis and cysts in the pancreas. When the family asks what is in the cysts, how should the nurse respond? Select all that apply. 1 The cysts can be filled with blood. 2 The cysts can be filled with calcium. 3 The cysts can be filled with leptin. 4 The cysts can be filled with adiponectin. 5 The cysts can be filled with pancreatic juice. 6 The cysts can be filled with necrotic debris

156 The cysts are walled-off areas or pockets of pancreatic juice, necrotic debris, or blood within or adjacent to the pancreas. Low calcium can lead to tetany, but it is not contained in a cyst. Leptin, a product of the obesity gene (Ob gene) and expressed primarily by adipocytes, acts on the hypothalamus to suppress appetite and functions to regulate body weight within a fairly narrow range. Adiponectin is a cytokine from adipose cells that promotes insulin sensitivity, whereas a lack can lead to insulin resistance.

A nurse is teaching a class about Alzheimer disease. Which information should the nurse include? Select all that apply. 1 It is the leading cause of dementia. 2 It is an uncommon neurologic disorder. 3 Its pathogenesis does not have a genetic component. 4 It causes plaques that increase nerve impulse transmissions. 5 It is a result of neuronal proteins becoming distorted and tangled

15Alzheimer disease is a common neurologic disorder and is the leading cause of dementia. Neurofibrillary tangles occur when the proteins in neurons become tangled and distorted. There is a genetic relationship in late-onset and familial Alzheimer dementia. With the formation of plaques, there is decreased, not increased, nerve impulse transmission.

After assessing a patient with herniated intervertebral disk (HID), the nurse suspects that the primary health care provider will recommend surgery for the patient. Which findings support the nurse's conclusion? Select all that apply. 1 The patient has weak bowel reflexes. 2 The patient has strong bladder reflexes. 3 The patient has weak Babinski reflexes. 4 The patient has strong ankle jerk reflexes. 5 The patient has weak deep tendon reflexes.

15Weak bowel and deep tendon reflexes indicate that the patient has severe nerve damage. Therefore, surgery may be necessary to repair the herniated disk. A strong bladder reflex indicates that the patient is normal and does not have risk of bladder distention. A patient with HID will have Babinski reflexes due to sensory impairment. Weak Babinski reflexes indicate that the patient is improving and does not require surgery. Presence of strong ankle jerk reflexes indicates that the patient does not have motor weakness in the lower limbs.

After assessing a patient with herniated intervertebral disk (HID), the nurse suspects that the primary health care provider will recommend surgery for the patient. Which findings support the nurse's conclusion? Select all that apply. 2 The patient has weak bowel reflexes. 2 The patient has strong bladder reflexes. 3 The patient has weak Babinski reflexes. 4 The patient has strong ankle jerk reflexes. 5 The patient has weak deep tendon reflexes

15Weak bowel and deep tendon reflexes indicate that the patient has severe nerve damage. Therefore, surgery may be necessary to repair the herniated disk. A strong bladder reflex indicates that the patient is normal and does not have risk of bladder distention. A patient with HID will have Babinski reflexes due to sensory impairment. Weak Babinski reflexes indicate that the patient is improving and does not require surgery. Presence of strong ankle jerk reflexes indicates that the patient does not have motor weakness in the lower limbs.

client with Graves disease has goiter. Which pathophysiologic mechanism for goiter does the nurse consider when planning care for this client? 1 Autoantibodies activation of the thyroid-stimulating hormone (TSH) receptors in the gland 2 Functional abnormalities resulting from hyperactivity of the sympathetic nervous system 3 Infiltrative changes involving the orbital contents with enlargement of the ocular muscles 4 Pretibial myxedema preventing formation of thyroxine hormone in the thyroi

1A goiter is formed when autoantibodies, called the thyroid-stimulating immunoglobulins, override the normal regulatory mechanisms, which results in a stimulation of the TSH receptors, causing an enlargement of the thyroid (goiter). Functional abnormalities resulting from hyperactivity of the sympathetic nervous system and infiltrative changes involving the orbital contents with enlargement of the ocular muscles describe the two mechanisms for ophthalmopathy, not goiter, development. Pretibial myxedema is characterized by subcutaneous swelling on the anterior portions of the legs and by indurated and erythematous skin.

A severely hypertensive client presents with headache, tachycardia, and diaphoresis. Laboratory tests demonstrate excessive circulating catecholamine levels. The nurse suspects the client is experiencing which adrenal medulla disorder? 1 Pheochromocytoma 2 Hypersecretion of adrenal androgens 3 Congenital adrenal hyperplasia 4 Adrenocortical hypofunction

1A pheochromocytoma is a catecholamine-secreting tumor that is usually located in the adrenal medulla and presents with headache, tachycardia, and diaphoresis that result from effects of the massive circulating catecholamine levels on the sympathetic nervous system. Hypersecretion of androgens causes virilization. Congenital adrenal hyperplasia results from the deficiency of an enzyme that is critical in cortisol biosynthesis, resulting in overproduction of either mineralocorticoids or androgens. Adrenocortical hypofunction can lead to Addison disease.

A nurse is caring for a client with amyotrophic lateral sclerosis. Which information should the nurse remember when planning care for this client? 1 Commonly, the arms and legs are affected first. 2 Most clients die within 1 year from cardiac failure. 3 Most clients lose their sensory and cognitive functions. 4 Early symptoms include speech abnormalities and respiratory failure

1About 60% of individuals have a spinal form of the disease with focal muscle weakness beginning in the arms and legs and progressing to muscle atrophy, spasticity, and loss of manual dexterity and gait. Progressive muscle atrophy and paralysis lead to respiratory failure and death within 2 to 5 years, although a small percentage of individuals may live 10 years or longer. No associated mental, sensory, or autonomic symptoms are present.

A client has a collection of blood that is located between the skull and the dura. What condition will the nurse observe written in the chart? 1 Epidural hematoma 2 Subdural hematoma 3 Intracerebral hematoma 4 Subarachnoid hematoma

1An epidural hematoma is a collection of blood between the skull and the dura. A subdural hematoma is a collection of blood between the dura and the surface of the brain. Intracerebral hematomas are blood collecting within the brain. Subarachnoid hemorrhage, a collection of blood in the subarachnoid space, is most often associated with a blood vessel rupture due to aneurysm, but it may be due to severe hypertension or head injury.

Which type of movement would the nurse expect to observe in a client diagnosed with hepatic encephalopathy? 1 Asterixis 2 Athetosis 3 Essential 4 Bradykinesia

1Asterixis is an irregular flapping movement of the hands seen in clients with disorders affecting the liver with excess ammonia levels in the blood (hepatic encephalopathy). Athetosis occurs most commonly as a result of injury to putamen of basal ganglion. Essential is a familial tremor and is not associated with hepatic encephalopathy. Bradykinesia is slowness of voluntary movements and is associated with Parkinson disease.

nurse is taking the history of a client with an intracranial aneurysm. There is no history of arteriosclerosis, trauma, or inflammation. Which area should the nurse ask about next? 1 Drug use 2 Alcohol use 3 Sexual activity 4 Cigarette smoking

1Cocaine abuse has been linked to aneurysm formation. Alcohol, sexual activity, and cigarette smoking have not been linked to aneurysm formation.

The nurse is caring for a client who has been diagnosed with end-stage renal failure. The nurse understands that this client's kidneys are functioning at about what percentage of normal? 1 5 2 25 3 50 4 75

1End-stage renal failure refers to a renal function of less than 10% requiring dialysis or transplant. Renal insufficiency generally refers to a decline in renal function to about 25% of normal or a glomerular filtration rate of 25 to 30 ml/min; 50% and 75% are too high.

nurse is reviewing targeted cancer screening results of various clients. Which client will be most at risk for adenomas of the colon? 1 A client with an APC mutation 2 A client with a WT1 mutation 3 A client with an NF1 mutation 4 A client with a BRCĀ mutatio

1Familial polyposis coli (adenomas of the colon) is associated with APC mutation. WT1 mutation is associated with Wilms tumor, a childhood cancer of the kidney. NF1 mutation is associated with neurofibromatosis. BRCĀ mutation is associated with inherited breast cancer.

Which clinical manifestation should the nurse expect to find in a client with Graves disease? 1 Double vision 2 Sunken eyeballs 3 Hyperthermia 4 Myxedema coma

1Graves disease is an autoimmune disease that can cause goiter (diffuse thyroid enlargement) and ophthalmopathy, which may lead to double vision (diplopia). The disease is manifested by protruding eyeballs (exophthalmos), not sunken eyeballs. Hyperthermia occurs in thyrotoxic crisis. Myxedema coma occurs in hypothyroidism, not Graves disease (hyperthyroidism).

Which client will the nurse most closely monitor for life-threatening impairment of the diaphragm? 1 A client with a cervical cord injury 2 A client with a thoracic cord injury 3 A client with a lumbar cord injury 4 A client with a sacral cord injury

1In the cervical region, cord swelling may be life-threatening because it may impair the diaphragm function (phrenic nerves exit at C3 to C5) and vegetative functions (mediated by the medulla oblongata). Thoracic, lumbar, and sacral are located further down in the spinal cord.

A nurse is administering recombinant tissue plasminogen activator (rtPA). The nurse is caring for which client? 1 A client with thrombotic stroke within 3 hours of symptom onset 2 A client with hemorrhagic stroke within 3 hours of symptom onset 3 A client with thrombotic stroke within 24 hours of symptom onset 4 A client with hemorrhagic stroke within 24 hours of symptom onse

1In thrombotic stroke, thrombolytic therapy for acute ischemia is within 3 hours (not 24) of the onset of symptoms. For a client with hemorrhagic stroke, rtPA is contraindicated.

A client has a pituitary adenoma. Which information should the nurse remember when planning care for this client? 1 It is usually microscopic. 2 It is generally metastatic. 3 It will experience rapid growth. 4 It will be mainly located in the posterior pituitary

1Most pituitary adenomas are microscopic. They arise from the anterior pituitary, are benign, and are usually slow growing in nature.

Which assessment finding is typical of a client with astrocytoma? 1 Focal seizure disorder with headache 2 Difficulty in balance with unsteady gait 3 Inability to control fine motor movement 4 Hearing loss and deafnes

1Onset of a focal seizure disorder between the second and sixth decades of life suggests an astrocytoma. Headache and subtle neurobehavioral changes may be early signs. Ependymomas of the fourth ventricle present with difficulty in balance, unsteady gait, and difficulty with fine motor movement. Schwannomas of the vestibular nerves present with hearing loss and deafness.

A patient with a vertebral fracture reports a pounding headache, blurred vision, nasal congestion, and sweating above the T3 level. Upon assessing the patient, the nurse finds that the patient has paroxysmal hypertension and bradycardia. Which nursing intervention will help to prevent a cerebrovascular accident in the patient? 1 Elevating the head of the patient's bed 2 Administering intravenous fluids 3 Providing potassium supplements 4 Teaching range-of-motion exercises

1Pounding headache, blurred vision, nasal congestion, sweating above T3 level, paroxysmal hypertension, and bradycardia are clinical manifestations of autonomic hyperreflexia. Elevating the head of the patient's bed decreases intracranial pressure and in turn, reduces the risk of cerebrovascular accident. Administering intravenous fluid increases fluid volume in the blood and may result in hypertension. Potassium supplements help to maintain fluid-electrolyte balance, but do not reduce the risk of cerebrovascular accident. Teaching range-of-motion exercises helps to reduce muscle flaccidity in the patient, but does not reduce the risk of cerebrovascular accident.

A nurse is caring for a client that has the most common classification of acute kidney injury. The nurse is caring for which client? 1 One with prerenal injury 2 One with postrenal injury 3 One with intrarenal injury 4 One with ultrarenal injury

1Prerenal is the most common type of acute kidney injury and is caused by renal hypoperfusion. Intrarenal acute kidney injury results from ischemia within the kidney and is not as common as prerenal. Postrenal acute kidney injury is rare. There is no such classification as ultrarenal.

A client has renal colic originating from the proximal ureters. What type of pain will the nurse find upon assessment? 1 In the flank and radiates to the groin 2 In the abdomen and radiates to the chest 3 In the right side and radiates to the left side 4 In the back and radiates retroperitoneall

1Renal colic, described as moderate to severe pain, often originating in the flank and radiating to the groin, usually indicates obstruction of the renal pelvis or proximal ureter. Pain in the abdomen that radiates to the bladder, as well as pain in the back that radiates retroperitoneally, usually indicates lower urinary tract symptoms. Colic pain does not move from the right to the left side or to the chest.

Which laboratory result for thyroid-stimulating hormone (TSH) would the nurse expect to find in a client with Graves disease? 1 Low level in the blood 2 High level in the blood 3 Normal level in the blood 4 Increased level in the urine

1TSH is low (suppressed) in the blood. In Graves disease, the presence of thyroid-stimulating immunoglobulins mimics the activity of TSH at the thyroid receptor, which causes increased thyroid hormone production that suppresses TSH production from the anterior pituitary. In Graves disease, TSH levels are not high in the blood, nor are they normal. TSH levels are not increased in the urine.

A client has osteoporosis. Which type of fracture does the nurse monitor for in this client? 1 Bowing 2 Reduced 3 Pathologic 4 Greenstick

3 Pathologic fractures occur in bones that have been weakened by disease, such as osteoporosis. Bowing fractures are common in children. A reduced fracture is one that has been corrected. Greenstick fractures are most common in children and elderly people.

Which finding in the patient with focal injury supports the nurse's conclusion that the patient has moderate blunt trauma? 1 The patient has paraplegia. 2 The patient has acute contusional swelling. 3 The patient has subacute subdural hematoma. 4 The patient has traumatic cerebral hemorrhage

1Traumatic injury results from an impact on the head that disturbs normal brain function. Paraplegia is associated with focal injury on top of the head. Therefore, the nurse expects moderate blunt trauma. The nurse expects severe blunt trauma if the patient has acute contusional swelling, subacute subdural hematoma, and traumatic cerebral hemorrhage.

Which term should the nurse use to describe the accumulation of nitrogenous wastes in a client with chronic kidney disease? 1 Uremia 2 Anuria 3 Oliguria 4 Hematuria

1Uremia and azotemia are terms for the accumulation of nitrogenous wastes. Anuria is the absence of urine. Oliguria is insufficient urine output (30 ml/hour or less). Hematuria is blood in the urine.

"Tell me the name of that chemical that makes crystals when my gout flares up," says a client. "I was hurting too much to remember it last time someone told me." What is the nurse's best response? 1 Urea 2 Uric acid 3 Calcium phosphate 4 Beta-hydroxybutyric acid

2

A client has delirium. When planning care, what should the nurse remember about the duration of this condition? 1 It lasts seconds to minutes. 2 It lasts hours to weeks. 3 It lasts months to years. 4 It is irreversible

2

A client has the most common form of acute glomerulonephritis. The nurse is caring for which client? 1 A client with lupus nephritis 2 A client with immunoglobulin A (IgA) nephropathy 3 A client with chronic glomerulonephritis 4 A client with anti-glomerular basement membrane disease

2

A client is experiencing the Somogyi effect. Which clinical findings will the nurse find upon assessment? 1 Hyperglycemia, followed by hypoglycemia 2 Hypoglycemia, followed by rebound hyperglycemia 3 Early morning rise in blood glucose with no hypoglycemia 4 Early morning hypoglycemia without rebound hyperglycemi

2

A nurse is asked how uric acid is eliminated from the body. How should the nurse respond? 1 Through sweat 2 Through the kidneys 3 Through crystals of tophi 4 Through metabolism of protein

2

A nurse is teaching a client diagnosed with gout about dietary modifications. Which nutritional information should the nurse include in the teaching session? 1 Restrict vegetable intake 2 Restrict meat intake 3 Restrict fluid intake 4 Restrict fruit intake

2

The nurse is caring for a client who recently underwent a release of a unilateral complete ureteral obstruction, resulting in postobstructive diuresis. The nurse anticipates which clinical manifestation? 1 Edema 2 Increased urine output 3 Increased blood pressure 4 Weight gain

2

What is the most common early clinical manifestation a nurse will observe in a client diagnosed with ankylosing spondylitis? 1 Kyphosis 2 Low back pain 3 Pain with sitting 4 Pleuritic chest pain

2

Which cells secrete amylin? 1 F cells 2 Beta cells 3 Delta cells 4 Alpha cells

2

Which client is predisposed to developing colon cancer? 1 A client with hiatal hernia 2 A client with ulcerative colitis 3 A client with duodenal ulcers 4 A client with dumping syndrom

2

Which information indicates the nurse has a correct understanding about the difference between a sprain and a strain? 1 A sprain is a tear in a tendon, whereas a strain is a tear in a ligament. 2 A strain is a tear in a tendon, whereas a sprain is a tear in a ligament. 3 A strain is a partial dislocation of a joint, whereas a sprain is a complete joint dislocation. 4 A sprain is a partial dislocation of a joint, whereas a strain is a complete joint dislocation.

2

Which term should the nurse use to describe the bone tissue union in a client with a fractured bone that is healing in an incorrect anatomic position? 1 Disunion 2 Malunion 3 Nonunion 4 Delayed union

2

nurse is teaching the client about causes of hyperglycemia in type 2 diabetes mellitus. Which information from the client indicates teaching was successful? 1 One cause is glycogen excess. 2 One cause is insulin resistance. 3 One cause is glucagon deficiency. 4 One cause is increased ghrelin

2

Which clinical finding in a client will help a nurse distinguish between pyramidal motor syndromes and extrapyramidal motor syndromes? 1 Extrapyramidal motor syndromes will have paralysis. 2 A Babinski sign occurs in pyramidal motor syndromes. 3 Pyramidal motor syndromes will have cogwheel rigidity with hypotonia. 4 Absence of involuntary movement occurs in extrapyramidal motor syndromes.

2 A Babinski sign is present in pyramidal motor syndromes. Pyramidal motor syndromes will have paralysis. Extrapyramidal motor syndromes will have cogwheel rigidity with hypotonia. Absence of involuntary movement occurs in pyramidal motor syndromes.

A client presents with a wide-based gait, in which the feet are turned outward. Upon assessment, the client staggers when walking and holds the pelvis stiff. Which condition do these assessment findings support? 1 Apraxia 2 Cerebellar gait 3 Basal ganglion gait 4 Decorticate postur

2 A cerebellar gait is wide-based with the feet apart and often turned outward or inward for greater stability. The pelvis is held stiff, and the individual staggers when walking. Apraxia is the inability to perform purposeful or skilled motor acts. Basal ganglion gait is a wide-based gait in which the person walks with small steps and decreased arm swing. Decorticate posture occurs when the upper extremities are flexed at the elbows and held close to the body while the lower extremities are externally rotated and extended.

A client has hyperthyroidism and develops a thyroid storm. The nurse will monitor the client for which type of heart failure? 1 Right heart failure 2 High-output failure 3 Systolic heart failure 4 Diastolic heart failure

2 A common cause of high-output failure is hyperthyroidism, especially if thyrotoxicosis or a thyroid storm develops. In high-output failure, the heart increases its output, but the body's metabolic needs are still not met. Right heart failure is defined as the inability of the right ventricle to provide adequate blood flow into the pulmonary circulation at a normal central venous pressure. Systolic heart failure is defined as an inability of the heart to generate an adequate cardiac output to perfuse vital tissues. Diastolic heart failure is also known as heart failure with preserved systolic function or heart failure with normal ejection fraction.

While examining a patient's eye, the nurse finds that the patient has a thin and curved cornea, increased pupil diameter, decreased opacity of lens, and a decreased size of the anterior chamber. What should the nurse infer from these findings? 1 The patient is at risk for cataracts. 2 The patient is at risk for glaucoma. 3 The patient is at risk for astigmatism. 4 The patient is at risk for senile miosis

2 A decrease in the size and volume of the anterior chamber of the eye causes an accumulation of aqueous humor and increases the intraocular pressure, which may result in glaucoma. An increase in the opacity of the lens decreases the refraction of light and causes a cataract to form. A reduction in pupil diameter causes persistent constriction and results in senile miosis. The presence of a thick, less-curved cornea is a clinical manifestation of astigmatism.

primary health care provider prescribes bone marrow transplantation to a patient with a leukocyte adhesion defect. Which treatment strategy will the nurse prepare the patient for before surgery can be performed? 1 Transfusing white blood cells 2 Administering corticosteroids 3 Transplanting fetal thymus tissue 4 Administering calcium supplements

2 A patient undergoing bone marrow transplantation has a risk of graft-versus-host disease (GVHD). Corticosteroids suppress the immune responses and reduce the risk of GVHD. White blood cells play a major role in immune responses. Transfusing white blood cells increases the risk of GVHD. Therefore, the nurse transfuses blood that has been irradiated to destroy white blood cells that would lead to a reaction. A patient with leukocyte adhesion defect does not have T cell deficiency. Therefore, transplanting fetal thymus tissue will not be beneficial. Administering calcium supplements will not reduce the risk of GVHD.

Which condition does the nurse suspect in a patient who reports polyuria, nocturia, and polydipsia? 1 Hypothyroidism 2 Diabetes insipidus 3 Pituitary adenomas 4 Syndrome of inappropriate antidiuretic hormone (SIADH

2 A patient with diabetes insipidus will exhibit polyuria, nocturia, and polydipsia due to a decrease in antidiuretic hormone levels. Therefore, the nurse suspects that the patient has diabetes insipidus. A patient with hypothyroidism will have cold intolerance, lethargy, and myxedema. A patient with pituitary adenomas will have headache, fatigue, neck stiffness, and seizures. SIADH causes impaired taste, anorexia, dyspnea, and dulled sensorium.

A nurse is caring for a client with colorectal cancer and learns that a point mutation occurred in the RAS gene. What pathophysiologic process should the nurse consider while planning care? 1 A tumor-suppressor gene became activated 2 A proto-oncogene converted to an oncogene 3 A tumor-suppressor gene became inactivated 4 A proto-oncogene converted to a gen

2 A point mutation in the RAS gene converts it from a regulated proto-oncogene to an unregulated oncogene, an accelerator of cellular proliferation. Activating point mutations in RAS are found in many cancers, especially pancreatic and colorectal cancer. A tumor-suppressor gene, if activated, helps prevent cancer, whereas its inactivation can lead to cancer; but those events are separate from a point mutation in the RAS gene. A proto-oncogene is a type of gene; it is not a nongene entity that is converted into a gene.

A client develops an acute arterial occlusion in an extremity from peripheral artery disease. Which finding will the nurse observe upon assessment? 1 The limb will be swollen, reddish, warm, and numb. 2 The limb will be pale, without pulses, and severely painful. 3 The limb will be normal except for dilated tortuous veins and aching. 4 The limb will be painful upon ambulation (intermittent claudication)

2 Acute arterial occlusion causes color change (pale from lack of blood), pulselessness, and severe pain. Swollen, reddish, warm, and numb extremities are not characteristic of acute arterial occlusion. Acute arterial occlusion does not refer to veins. Intermittent claudication occurs from gradually increasing obstruction to arterial blood flow, not from an acute arterial occlusion.

A client develops an acute arterial occlusion in an extremity from peripheral artery disease. Which finding will the nurse observe upon assessment? 1 The limb will be swollen, reddish, warm, and numb. 2 The limb will be pale, without pulses, and severely painful. 3 The limb will be normal except for dilated tortuous veins and aching. 4 The limb will be painful upon ambulation (intermittent claudication).

2 Acute arterial occlusion causes color change (pale from lack of blood), pulselessness, and severe pain. Swollen, reddish, warm, and numb extremities are not characteristic of acute arterial occlusion. Acute arterial occlusion does not refer to veins. Intermittent claudication occurs from gradually increasing obstruction to arterial blood flow, not from an acute arterial occlusion.

A nurse is teaching the staff about the role that increased sympathetic nervous system (SNS) activity plays in the development of hypertension. Which information should the nurse include? 1 Decreased heart rate 2 Increased renin levels 3 Decreased insulin resistance 4 Increased renal excretion of sodium

2 Additional mechanisms of SNS-induced hypertension include structural changes in blood vessels (vascular remodeling), renal sodium retention (shift in natriuresis curve), insulin resistance, increased renin and angiotensin levels, and procoagulant effects. SNS increases heart rate.

urse is reviewing the pathology report on a client's colon polyp. Which finding will most likely cause the nurse to monitor the client for colorectal cancer? 1 Non-neoplastic polyp 2 Adenomatous polyp 3 Hyperplastic polyp 4 Helicobacter pylori poly

2 Adenocarcinomas of the colon and rectum usually arise from adenomatous polyps. The most common types of polyps are hyperplastic polyps (which are non-neoplastic, that is, benign) and adenomatous polyps (which are neoplastic). Helicobacter pylori is not associated with colorectal cancer but with stomach cancer.

Which male client is most predisposed to an obstruction of the lower urinary tract? 1 A client with pelvic organ prolapse 2 A client with prostate enlargement 3 A client with urinary tract infection 4 A client with congenital stricture of a calyx

2 Anatomic causes of resistance to urine flow include urethral stricture, prostatic enlargement in men, and pelvic organ prolapse in women. A urinary tract infection does not lead to an obstruction of the lower urinary tract; however, a urinary tract obstruction can lead to a urinary tract infection. A congenital stricture of a calyx could lead to upper urinary tract obstruction, not lower.

Which change occurs in a patient's liver cells due to insulin? 1 Decreased glycolysis 2 Decreased glycogenesis 3 Decreased glycogenolysis 4 Decreased gluconeogenesis

3 Insulin decreases glycogenolysis in the liver cells. It increases glycolysis in liver cells. Glycogenesis is increased in liver cells due to insulin. Insulin increases gluconeogenesis in liver cells. Glycogenolysis is the biochemical breakdown of glycogen to glucose glycogenesis is the opposite,

Which information indicates the nurse has a correct understanding of the definition of cardiogenic shock? 1 Cardiogenic shock causes increased cardiac output related to severe vasodilation. 2 Cardiogenic shock is the presence of poor tissue oxygenation despite adequate blood volume. 3 Cardiogenic shock causes decreased catecholamine release, resulting in poor tissue perfusion. 4 Cardiogenic shock is the impaired release of renin and aldosterone from an overwhelming infection.

2 Cardiogenic shock is characterized by decreased cardiac output and tissue hypoxia despite adequate volume of blood. Vasodilation is seen in anaphylactic, neurogenic, and septic shocks. Catecholamine production is increased in cardiogenic shock. Renin and aldosterone levels increase in cardiogenic shock.

A client has amyotrophic lateral sclerosis (ALS). Which assessment findings will be typical for the nurse to observe in this client? 1 Writhing movements and progressive dementia 2 Progressive muscle weakness and eventually paralysis 3 Muscle rigidity, difficulty initiating movement, drooling 4 Progressive loss of memory, judgment, and self-care ability

2 Clinical manifestations of ALS include progressive muscle weakness and eventual paralysis. Writhing movements and progressive dementia characterize Huntington disease. Muscle rigidity, difficulty initiating movement, and drooling characterize Parkinson disease. Progressive loss of memory, judgment, and self-care ability characterize Alzheimer disease.

A client's arms are flexed at the elbows and held close to the body, and the legs are externaly rotate and extended. What term will the nurse use to document this type of dystonia? 1 Senile posture 2 Decorticate posture 3 Decerebrate posture 4 Basal ganglion postu

2 Decorticate posture is characterized by upper extremities flexed at the elbows and held close to the body and by lower extremities that are externally rotated and extended. Senile posture is characterized by an increasingly flexed posture similar to that caused by basal ganglion dysfunction. Decerebrate posture is characterized by all four limbs rigidly extended. Basal ganglion posture refers to a stooped, hyperflexed posture with a narrow-based, short-stepped gait.

A client has hypokinesia. The nurse will observe which finding upon assessment? 1 Lip smacking 2 Expressionless face 3 Varied speech inflections 4 Numerous spontaneous gestures

2 Decreased associated movements accompanying emotional expression cause an expressionless face, a statue-like posture, absence of speech inflection, and absence of spontaneous gestures for an individual with hypokinesia. Lip smacking occurs in hyperkinesia.

Which client is at highest risk for osteomyelitis and therefore needs careful assessment for development of the condition? 1 A client who has a pustule on the arm 2 A client who has an infected diabetic foot ulcer 3 A client who has widespread psoriasis 4 A client who has a closed oblique tibial fracture

2 Diabetic foot ulcers confer a high risk of osteomyelitis because they easily become infected and the infection may spread to nearby bone. Although pustules carry some risk of osteomyelitis, a foot ulcer is a higher risk. Psoriasis does not create high risk for osteomyelitis. A closed fracture does not create as high a risk for osteomyelitis as does a current infection.

A client is brought to the emergency room after a motor vehicle accident in which the client sustained diffuse brain (axonal) injury. Which assessment findings would the nurse expect to accompany this injury? 1 Weakness and no coma 2 Agitation and spastic paralysis 3 Memory deficits but good judgment skills 4 Long attention span and no loss of consciousness

2 Diffuse brain injury can produce agitation, spastic paralysis, memory deficits, dysphagia, and short (not long) attention span. Diffuse brain injury leads to a coma that lasts at least 6 hours or longer. Poor (not good) judgment skills occur.

Which term should the nurse use to describe the consequence of an upper urinary tract obstruction in one of a client's ureters? 1 Anuria 2 Hydronephrosis 3 Dilation of the urethra 4 Kidney stone formatio

2 Dilation of the urinary tract occurs proximal to the obstruction. In this case, the proximal ureter and renal pelvis would enlarge, causing hydronephrosis. Urine production will continue to occur if one ureter is blocked, so anuria would not occur. Dilation of the urinary tract occurs proximal to the obstruction, so the urethra would not be dilated. Kidney stones are causes, rather than consequences, of an upper urinary tract obstruction.

The nurse is caring for a client who recently underwent a release of a unilateral complete ureteral obstruction, resulting in postobstructive diuresis. The nurse anticipates which clinical manifestation? 1 Edema 2 Increased urine output 3 Increased blood pressure 4 Weight gaiN

2 Diuresis is an increase in urine output. Relief of bilateral, partial urinary tract obstruction or complete obstruction of one kidney is usually followed by a brief period of diuresis (commonly called postobstructive diuresis). Increased blood pressure, edema, and weight gain are indications of increased fluid volume, not decreased volume.

A client has dyssynergia. The nurse is caring for which client? 1 One with lower motor neuron disorder 2 One with upper motor neuron lesions 3 One with lesions in the sacral area of the spinal cord 4 One with lesions above the pontine micturition center

2 Dyssynergia is a loss of coordinated neuromuscular contraction, resulting in an overactive or hyperreflexive bladder from lesions that develop in upper motor neurons of the brain or spinal cord. Neurologic disorders that develop above the pontine micturition center result in detrusor hyperreflexia, also known as an uninhibited or reflex bladder. Lesions in the sacral area of the spinal cord or peripheral nerves result in underactive, hypotonic, or atonic (flaccid) bladder function. A lower motor neuron disorder results in detrusor areflexia.

A nurse checks a woman on estrogen replacement therapy for deep vein thrombosis. What is the rationale for the nurse's action? 1 Estrogen causes endothelial injury. 2 Estrogen causes hypercoagulability. 3 Estrogen causes stasis of blood flow. 4 Estrogen causes a reduction in platelets.

2 Estrogen therapy causes hypercoagulability of the blood, one of the three factors of the triad of Virchow. The other two factors, endothelial injury (for example, trauma) and stasis of blood flow (for example, immobility), are not caused by estrogen. A reduction in platelets would lead to bleeding and is not an effect of estrogen.

A nurse is assessing a client with suspected primary hyperparathyroidism. Which assessment findings would support this diagnosis? Select all that apply. 1 Tetany 2 Renal stones 3 Hypercalcemia 4 Hypophosphatemia 5 Pathologic fractures

2345Hypercalcemia and hypophosphatemia are the hallmarks of primary hyperparathyroidism. Renal stones from the increased calcium may occur. Bone resorption may cause pathologic fractures, kyphosis, and compression fractures. Tetany occurs in hypoparathyroidism.

40-year-old woman has diffuse pain, fatigue, and increased sensitivity to touch without signs of inflammation. Which condition will the nurse observe written on the chart? 1 Contracture 2 Fibromyalgia 3 Disuse atrophy 4 Stress-induced muscle tension

2 Fibromyalgia is a chronic musculoskeletal syndrome characterized by diffuse pain, fatigue, increased sensitivity to touch (i.e., tender points), the absence of systemic or localized inflammation, and the presence of fatigue and nonrestorative sleep; anxiety and depression also are frequently present. Of affected individuals, 80% to 90% are women, and the peak age is 30 to 50 years. Contracture is a contraction of muscle fibers caused by phenomena secondary to the muscle (spasms or weakness) and sometimes without a muscle action potential. Disuse atrophy is a pathologic reduction in normal size of muscle fibers after prolonged inactivity or local nerve damage. Stress-induced muscle tension is caused by chronic anxiety and includes neck stiffness, back pain, and headache.

For a client with Graves disease, which type of hypersensitivity reaction should guide nursing care? 1 Type I 2 Type II 3 Type III 4 Type IV

2 Graves disease is an example of a type II tissue-specific hypersensitivity reaction that causes the cell to malfunction. Type I is an IgE allergic response. Type III is a complex-mediated hypersensitivity reaction. Type IV is a type of delayed hypersensitivity response.

Two siblings are diagnosed with a thyroid disorder due to destruction of thyroid tissue by lymphocytes and circulating thyroid autoantibodies. This pathophysiology will lead the nurse to suspect which condition? 1 Painless thyroiditis 2 Hashimoto disease 3 Subacute thyroiditis 4 Postpartum thyroiditis

2 Hashimoto disease is also called autoimmune thyroiditis. It results in the gradual destruction of thyroid tissue by infiltration of lymphocytes and circulating thyroid autoantibodies, and it has several genetic risk factors. Painless thyroiditis has a similar course to subacute thyroiditis but is pathologically identical to Hashimoto disease; however, spontaneous recovery occurs. Subacute thyroiditis is a nonbacterial inflammation of thyroid often preceded by a viral infection. Postpartum thyroiditis generally occurs within 6 months of delivery.

A nurse is planning care for a client with ischemic acute tubular necrosis. Which pathophysiologic process should the nurse remember? 1 Antigen-antibody complexes attack tubular epithelial cells 2 Oxygen free radicals cause tubular cell damage 3 Increased intraluminal pressure within the glomerulus 4 Enlarged pore size of the glomerular filtration membran

2 Hypotension associated with hypovolemia produces ischemia, generating toxic oxygen free radicals that cause cellular swelling, injury, and necrosis. Increased intraluminal pressure results from urinary obstructive disease. Structural alterations in the glomerular filtration membrane are associated with nephrotic syndrome. Membranous nephropathy, a type of glomerulonephritis, results from the antigen-antibody complexes activating C5b-C9 fragments of complement on glomerular (not tubular) epithelial cells with injury and release of inflammatory mediators.

A client has the most common form of acute glomerulonephritis. The nurse is caring for which client? 1 A client with lupus nephritis 2 A client with immunoglobulin A (IgA) nephropathy 3 A client with chronic glomerulonephritis 4 A client with anti-glomerular basement membrane diseas

2 IgA nephropathy (also called Berger disease) is the most common form of acute glomerulonephritis. Lupus nephritis is a form of acute glomerulonephritis but is not the most common, and anti-glomerular basement membrane disease is a rare form of glomerulonephritis. Chronic glomerulonephritis is not a form of acute glomerulonephritis.

client has Wiskott-Aldrich syndrome. Which nursing goal takes priority? 1 Protection from an anaphylactic response 2 Protection from IgM deficiency infections 3 Protection from thrombus formation 4 Protection from facial scarring

2 In Wiskott-Aldrich syndrome, an X-linked recessive disorder, IgM antibody production is greatly depressed. IgE hypersensitivity response causes anaphylactic reactions or allergic reactions. This syndrome causes bleeding because of decreases in circulating platelets; it does not cause thrombus (clot) formation. Facial scarring is not associated with this syndrome.

While planning care for a client with rheumatoid arthritis, which pathophysiologic process should the nurse consider? 1 Excessive wear and tear and microtrauma damaging articular cartilage 2 Immune cell accumulation in pannus and destruction of articular cartilage 3 Cysts developing in subchondral bone and creating fissures in articular cartilage 4 Free radicals attaching to the synovial membrane and tunneling into articular cartilage

2 In rheumatoid arthritis, immune cells accumulate in pannus and destroy articular cartilage. Rheumatoid arthritis is not caused by excessive wear and tear; it is an autoimmune disease. Subchondral cysts and fissures in articular cartilage are not characteristic of rheumatoid arthritis. Free radicals do not attach to membranes; they damage them by removing electrons from them.

A client has rheumatoid arthritis. Which clinical manifestation will the nurse observe upon assessment? 1 Stiffness worse at night 2 Symmetric joint swelling 3 Multiple abscesses 4 Enlarged jaw

2 Joint swelling that is widespread and symmetric is caused by increasing amounts of inflammatory exudate. Stiffness usually lasts for about 1 hour after rising in the morning. Multiple abscesses can occur with osteomyelitis. Enlarged jaw occurs with Paget disease, whereas in rheumatoid arthritis the joints affected are the metacarpophalangeal, proximal interphalangeal, or wrists.

A client is diagnosed with third-degree block. A nurse categorizes this block as which disorder? 1 Disorder of impulse formation 2 Disorder of impulse conduction 3 Disorder of bacterial infection 4 Disorder of bacterial overgrowth

2 Left bundle branch block is classified as abnormal conduction within the heart. Disorders of impulse formation include sinus bradycardia, atrial flutter, and ventricular fibrillation. Left bundle branch block is not classified as a bacterial infection or overgrowth.

Which nursing goal will take priority for a client with a slow-growing aortic aneurysm? 1 Expansion of blood volume 2 Reduction of blood pressure 3 Administration of beta stimulants 4 Prevention of postural hypotension

2 Medical treatment is indicated for slow-growing aortic aneurysms, particularly in early stages, and includes cessation of smoking, reduction of blood pressure and blood volume, and implementation of β-adrenergic blockade. Reducing postural hypotension is not a priority in this situation.

Which response by the nurse best explains agnosia to a nursing assistant personnel? 1 "The client would have an altered sense of smell." 2 "The client would not be able to understand words." 3 "This occurs when a client cannot remember recent events." 4 "A client would not be able to identify a pencil by touching it.

4

client with cancer has received treatment to shrink the cancer before surgery. Which term should the nurse use to describe the treatment to shrink the cancer? 1 Adjuvant 2 Neoadjuvant 3 Brachytherapy 4 Alternative

2 Neoadjuvant chemotherapy is given before localized (surgical or radiation) treatment of a cancer. Neoadjuvant therapy can shrink a cancer so that surgery may spare more normal tissue. Adjuvant chemotherapy is given after surgical excision of a cancer. Brachytherapy uses radiation sources (typically a capsule filled with a radionuclide) to be temporarily placed into body cavities that have cancer. Alternative therapies can be biologically harmless or harmful; rarely is there any evidence these approaches are medically effective, and in the worst cases they can be expensive, delay the use of effective therapies, and produce toxic side effects.

A client has nephrotic syndrome. Which substance is the client losing? 1 Cholesterol 2 Albumin 3 Phospholipids 4 Triglycerides

2 Nephrotic syndrome is the excretion of 3.5 g or more of protein in the urine per day and is characteristic of glomerular injury. Albumin is lost in the greatest quantity because of its high plasma concentration and low molecular weight. Serum albumin level decreases (to less than 3 g/dl), and concentrations of serum cholesterol, phospholipids, and triglycerides increase.

client has neurogenic shock. What is a primary pathophysiologic response in the client's body? 1 Decreased volume 2 Massive vasodilation 3 Sympathetic overstimulation 4 Increased systemic vascular resistance

2 Neurogenic shock (sometimes called vasogenic shock) is the result of widespread and massive vasodilation that results from parasympathetic overstimulation and sympathetic understimulation, not overstimulation. Decreased (not increased) systemic vascular resistance occurs, resulting in "relative hypovolemia." The amount of blood has not changed, but the space that carries blood has increased.

A client has a broken bone in which the bone ends have failed to grow together. Which term should the nurse use to describe this condition? 1 Malunion 2 Nonunion 3 Dislocation 4 Delayed union

2 Nonunion is failure of the bone ends to grow together. A malunion is the healing of a bone in an incorrect anatomic position. Dislocation is the displacement of one or more bones in a joint in which the opposing joint surfaces lose contact entirely. A delayed union is a union that takes 8 to 9 months to heal.

A nurse is caring for a client who has diabetic neuropathy and develops an infectious foot ulcer. The nurse closely monitors the client for osteomyelitis. What is the rationale for the nurse's actions? 1 Viral proliferation within cells 2 Extension of infection into bone 3 Hypoperfusion due to vasculopathy 4 Autoimmune infection by macrophages

2 Osteomyelitis in people who have a diabetic neuropathic foot ulcer most commonly arises when the ulcer becomes infected and microorganisms spread through the nearby tissue to a bone. Virus infection is not characteristic of diabetic neuropathic foot ulcers. Vasculopathy can occur in diabetes, but it is not directly related to the nurse's expectation that the infection may spread from ulcer to bone. Autoimmune infection by macrophages does not occur in osteomyelitis.

A nurse is caring for a woman diagnosed with rheumatoid arthritis. Which assessment finding does the nurse recognize as a major reason for the primary healthcare provider to prescribe a bone density test (dual x-ray absorptiometry)? 1 Ulnar drift 2 Kyphotic posture 3 "My hands and knees hurt a lot." 4 Subcutaneous nodules on elbows

2 Osteoporosis causes vertebral collapse, leading to kyphosis; a bone density test is prescribed to determine osteoporosis. Subcutaneous nodules, hurting hands and knees, and ulnar drift are characteristic of rheumatoid arthritis and would not require a bone density test.

A nurse is caring for a client with a bone tumor that demonstrates a moth-eaten pattern of bone destruction. The nurse is caring for which client? 1 A client with fibrosarcoma 2 A client with osteosarcoma 3 A client with endochondroma 4 A client with chondrosarcoma

2 Osteosarcoma has a moth-eaten pattern and is the most aggressive type of bone tumor. Fibrosarcoma is a solitary tumor that most frequently affects the metaphyseal region of the femur or tibia; it demonstrates a permeative growth pattern. Chondrosarcoma (cartilage-forming tumor) is the second most common primary malignant bone tumor and infiltrates the trabeculae of spongy bone. A secondary chondrosarcoma is derived from an endochondroma.

A teenager has the most common malignant bone-forming tumor in the metaphyseal region of the femur. During an interdisciplinary team meeting, which term will the primary healthcare provider use to describe this condition? 1 Chondroma 2 Osteosarcoma 3 Osteoid osteoma 4 Giant cell tumor

2 Osteosarcoma is the most common malignant bone-forming tumor. It occurs in puberty in persons younger than 20 years or in adults 50 to 60 years of age. It occurs in the metaphyseal region of long bones. Chondroma is a cartilage-forming tumor located within the bone that is benign. Osteoid osteoma is benign; these arise from the osteoblast. Giant cell tumors are benign but aggressive. They commonly occur between the ages of 20 and 40 years.

The nurse teaches a coworker about pericardial effusions. Which information from the coworker indicates teaching was successful? 1 Only a large amount of fluid can cause tamponade. 2 Pericardial effusion is not always clinically significant. 3 Serous effusions are generally related to aneurysms. 4 Fluid accumulates in the myocardial region.

2 Pericardial effusion is not always clinically significant if the fluid accumulates gradually. However, if fluid, in either large or small amounts, accumulates rapidly, a serious condition called tamponade may result. Effusions of frank blood are generally related to aneurysms, whereas serous effusion develops with left heart failure or overhydration. A pericardial effusion is the accumulation of fluid in the pericardial cavity, not in the myocardial region.

Which complication should the nurse monitor for in a client with acute otitis media? 1 Influenza 2 Meningitis 3 Sudden ear pain 4 Inflamed tympanic membrane

2 Possible complications of acute otitis media include chronic otitis media, brain abscess, and meningitis. Influenza is not associated with acute otitis media. Sudden ear pain, inflamed tympanic membrane, and fever are symptoms of acute otitis media.

A client has atherosclerosis. Which complication does the nurse monitor for in this client? 1 Formation of arteriovenous shunts 2 Obstruction of the blood vessel lumen 3 Autoimmune destruction of the vessel wall 4 Increased blood flow through the diseased vessels

2 Smooth muscle proliferation, plaque formation, and thrombosis from rupture of an atherosclerotic plaque all can reduce the size of the blood vessel lumen. Arteriovenous shunts and autoimmune factors are not a common consequence of atherosclerosis. Blood flow is reduced with severe atherosclerosis.

A client has prerenal acute kidney injury. Which information from the history is most consistent with this disorder? 1 Client had glomerulonephritis. 2 Client had severe hypotension. 3 Client had acute tubular necrosis. 4 Client had bilateral kidney stones

2 Prerenal acute kidney injury occurs when blood flow to the kidneys is compromised. Severe hypotension from heart failure, hypovolemia, or shock is a leading cause. Glomerulonephritis does not cause prerenal acute kidney injury, because the glomeruli are located within the kidneys, but it does cause intrarenal dysfunction. Acute tubular necrosis does not cause prerenal acute kidney injury, because the renal tubules are located within the kidneys; it is classified as intrarenal. Bilateral kidney stones do not cause prerenal acute kidney injury, because they are located distal to the nephrons; they are classified as postrenal.

A client has prerenal acute kidney injury. Which information from the history is most consistent with this disorder? 1 Client had glomerulonephritis. 2 Client had severe hypotension. 3 Client had acute tubular necrosis. 4 Client had bilateral kidney stones.

2 Prerenal acute kidney injury occurs when blood flow to the kidneys is compromised. Severe hypotension from heart failure, hypovolemia, or shock is a leading cause. Glomerulonephritis does not cause prerenal acute kidney injury, because the glomeruli are located within the kidneys, but it does cause intrarenal dysfunction. Acute tubular necrosis does not cause prerenal acute kidney injury, because the renal tubules are located within the kidneys; it is classified as intrarenal. Bilateral kidney stones do not cause prerenal acute kidney injury, because they are located distal to the nephrons; they are classified as postrenal.

patient with arthritis has a malar rash, oral ulcers, seizures, and a discoid rash. Which class of medication should be included in the patient's prescriptions to provide effective treatment? 1 Tetracycline 2 Corticosteroids 3 Aminoglycosides 4 Protease inhibitors

2 Presence of arthritis, malar rash, oral ulcers, seizures, and discoid rash indicates that the patient has systemic lupus erythematosus (SLE), an autoimmune disorder. Corticosteroids have antiinflammatory and immunosuppressant action, so these drugs will help to provide effective treatment to the patient. Tetracycline and aminoglycosides are antibiotics that help to alleviate symptoms of bacterial infections. Protease inhibitors are antiretroviral medications that help to alleviate symptoms of AIDS, not SLE.

A client was found at home in the fetal position complaining of a severe, sudden onset headache. Assessments in the emergency room reveal confusion, disorientation, and one dilated pupil. What reflex needs to be evaluated initially? 1 Agnosia 2 Pupillary reaction 3 Epileptogenic focus 4 Oculovestibular reflex

2 Pupillary reaction is one of five assessments to determine cerebral function. Agnosia is a defect of pattern recognition in which there is failure to recognize the form and nature of an object; this is not a reflex. Epileptogenic focus occurs in epilepsy when a group of neurons exhibit a paroxysmal depolarization shift and function; it is not a reflex. Oculovestibular reflex is used primarily for comatose clients.

A client who reports abdominal and back pain has been diagnosed with bacteria in the urine. What is the appropriate term for involvement of the upper urinary tract that is likely to cause such symptoms? 1 Cystitis 2 Pyelonephritis 3 Nephrotic syndrome 4 Bladder tumor

2 Pyelonephritis indicates involvement of the upper tracts. Cystitis is inflammation of the bladder. Nephrotic syndrome is the excretion of 3.5 g or more of protein in the urine per day and is characteristic of glomerular injury. Gross painless hematuria is the archetypal clinical manifestation of bladder cancer, not bacterial infection.

A nurse is assessing a client with suspected kidney stone obstruction of the ureter. Which assessment finding will support this diagnosis? 1 Pyuria 2 Renal colic 3 Urge incontinence 4 Abdominal mass

2 Renal colic, described as moderate to severe pain often originating in the flank and radiating to the groin, usually indicates obstruction of the renal pelvis or proximal ureter. Pyuria is not a sign of ureter obstruction by a kidney stone; hematuria may be present with a kidney stone. Urge incontinence normally occurs when a kidney stone is obstructing the lower urinary tract. Kidney stones are usually small, so they generally do not form a palpable mass.

Which symptom would the nurse expect to find when assessing a client who is atopic? 1 Dehydration, anxiety, and tremors 2 Rhinitis, urticaria, and wheezing 3 Jaundice, fatigue, and decreased urine output 4 Seizures, shortness of breath, and hemorrhage

2 Rhinitis, urticaria, and bronchial asthma are all common manifestations of an atopic, type I allergic reaction. A person who is atopic has a genetic predisposition to allergies. The airways and skin of the atopic individual have increased responsiveness to a wide variety of allergens. Dehydration, anxiety, and tremors are not characteristic of type I hypersensitivity reactions; neither are jaundice, fatigue, and decreased urine output. Individuals with type I hypersensitivity reactions do not usually experience seizures or hemorrhage.

A nurse is teaching a women's group about the risk factors for osteoporosis. Which information about risk factors from the women indicates teaching was successful? 1 Dark skinned 2 Thin build 3 Large stature 4 Increased calcium intake

2 Risk factors for osteoporosis include: thin build, fair or pale skinned (not dark), small stature (not large), and decreased calcium intake, not increased.

Which information indicates the nurse has an accurate understanding of the difference between sequestrum and involucrum bone tissue? 1 Sequestrum is new bone, whereas involucrum is mature bone. 2 Sequestrum is necrotic bone, whereas involucrum is new bone that is formed. 3 Sequestrum bone is the immune system response to infection, whereas involucrum involves edema in the bone marrow. 4 Sequestrum is osteoblasts laying down new bone, whereas involucrum leads to deprivation of blood to the bones.

2 Sequestrum is necrotic bone, whereas involucrum is new bone that is formed in osteomyelitis. Sequestrum is not new bone; involucrum is. Sequestrum describes necrotic bone, not the immune system response. Deprivation of the bone's blood supply leads to sequestrum, and the new bone that forms nearby is involucrum.

Which presurgical intervention should the nurse expect to implement on a child with a femoral fracture? 1 Dislocation 2 Skin traction 3 Open reduction 4 Internal fixation

2 Skin traction is used when only a few pounds of pulling force are needed to realign the fragments or when the traction will be used for brief times only, such as before surgery or, for children with femoral fractures, for 3 to 7 days before applying a cast. Dislocation is the displacement of one or more bones in a joint in which the opposing joint surfaces entirely lose contact; it is not an intervention. Open reduction and internal fixation are surgical procedures, not presurgical interventions.

Which information should the nurse include in a teaching session for a client with hypokalemic periodic paralysis? 1 This condition will last about 5 to 10 minutes. 2 Avoid eating sodium- or carbohydrate-rich foods. 3 Try to exercise strenuously at least twice a week. 4 Cold can help decrease the pain in the muscles

2 Sodium- or carbohydrate-rich foods should be avoided. Unlike with hyperkalemic periodic paralysis, eating sodium- or carbohydrate-rich foods also can cause an attack. Hypokalemic periodic paralysis also can be triggered by exposure to cold or rest after strenuous exercise. This condition can last hours to days, not 5 to 10 minutes.

A client is brought to the hospital after twisting an ankle while running around the bases in a baseball game. On examination, the ligaments have sustained tears. Which is the best term to describe this injury? 1 Strain 2 Sprain 3 Avulsion 4 Reduction

2 Sprains involve tears of the ligaments; strains are tears in tendons or muscles. An avulsion is a complete separation of a tendon from its bony attachment site. Treatment of a displaced fracture involves realigning the bone fragments (reduction) close to their normal or anatomic position and holding the fragments in place.

What client statement regarding treatment options for osteoarthritis indicates the client needs more teaching? 1 "I need to lose weight." 2 "Thank goodness I will never need surgery." 3 "Thank goodness I have my cane to help me walk." 4 "I should take anti-inflammatory medications for the pain."

2 Surgery is used to improve joint movement, correct deformity or malalignment, or create a new joint with artificial implants. Clients with osteoarthritis need to lose weight if they are obese. Use of canes, crutches, or walkers is used to decrease weight bearing. Anti-inflammatory medications can help reduce pain and inflammation.

A client has compartment syndrome. The nurse should teach the client about which treatment? 1 Dialysis 2 Fasciotomy 3 Bicarbonate 4 Osmotic diuresis

2 Surgical intervention consists of performing a fasciotomy of the affected area to decompress the compartment and allow return of normal blood supply. Dialysis, bicarbonate, and osmotic diuresis may be needed for rhabdomyolysis.

A client with type 1 diabetes has morning fasting blood glucose levels between 200 and 240 mg/dl with no hypoglycemia during the night. Which acute complication of diabetes is this client experiencing? 1 Somogyi effect 2 Dawn phenomenon 3 Diabetic ketoacidosis (DKA) 4 Hyperosmolar hyperglycemic nonketotic syndrome (HHNKS

2 The dawn phenomenon is an early morning rise in blood glucose concentration with no hypoglycemia during the night. The Somogyi effect is a unique combination of hypoglycemia followed by rebound hyperglycemia. Clients with DKA will have blood glucose levels > 250 mg/dl. Clients with HHNKS will have blood glucose levels > 600 mg/dl.

Which information indicates the nurse has an accurate understanding of the major pathophysiologic process of glomerulonephritis? 1 Hydronephrosis from kidney stones 2 Immune system damage to glomeruli 3 Overactive bladder syndrome 4 Interstitial cystitis

2 The major cause of glomerulonephritis is immune system damage to glomeruli. Hydronephrosis does not cause glomerulonephritis. Overactive bladder syndrome is a syndrome of detrusor overactivity characterized by urgency with involuntary detrusor contractions during the bladder filling phase that may be spontaneous or provoked. Interstitial cystitis is a condition that includes nonbacterial infectious cystitis and noninfectious cystitis; it does not affect the glomerulus.

A client asks what causes portal hypertension. What is the nurse's best response? 1 Protein-energy malnutrition 2 Cirrhosis of the liver 3 Pancreatitis 4 Varices

2 The most common cause of portal hypertension is fibrosis and obstruction caused by cirrhosis of the liver. Protein-energy malnutrition in the presence of carbohydrate intake is called kwashiorkor. Pancreatitis is inflammation of the pancreas and does not cause portal hypertension. Varices are a consequence of portal hypertension, not the cause.

client is diagnosed with portal hypertension due to liver disease. Which is the most common assessment finding to support this diagnosis? 1 Ascites 2 Hematemesis 3 Altered mental status 4 Pressure of 5 mm Hg

2 The most common clinical manifestation of portal hypertension is vomiting of blood (hematemesis) from bleeding esophageal varices. Ascites can occur but is not the most common sign. Altered mental status occurs with encephalopathy. Portal hypertension is an increase to at least 10 mm Hg, not 5.

While planning care for a client with glomerulonephritis, the nurse considers that which type of hypersensitivity reaction is most responsible for the immune mechanisms associated with glomerulonephritis? 1 Type I hypersensitivity 2 Type III hypersensitivity 3 Delayed hypersensitivity 4 Immediate hypersensitivity

2 The most common types of immune injury are (1) deposition of circulating antigen-antibody immune complexes into the glomerulus (type III hypersensitivity) and (2) antibodies reacting in situ against planted antigens within the glomerulus (type II hypersensitivity). Type I or immediate is associated not with glomerulonephritis but with allergic responses. Type IV or delayed hypersensitivity is associated not with glomerulonephritis but with poison ivy or tuberculosis skin test.

nurse is performing a postvoid urine test on a client with urinary retention. Which piece of equipment should the nurse obtain? 1 Otoscope 2 Catheterization kit 3 Blood pressure cuff 4 Needle and syringe to withdraw urine

2 The postvoid urine is measured by catheterization within 5 to 15 minutes of urination or through a bladder ultrasound machine that measures bladder height and width to provide an approximation of urine within the vesicle. An otoscope (for the ear) and blood pressure cuff would be ineffective. A needle and syringe is not used to withdraw urine for a postvoid test.

A nurse encourages ambulation for a client to assist with venous return. What is the rationale for the nurse's action? 1 Uses gravity 2 Activates the muscle pump 3 Enhances the action of two-way valves 4 Promotes vascular smooth muscle contractions

2 The pumping action of skeletal muscles assists venous return when a person walks. When a person is in an upright position, such as when standing or walking, gravity opposes venous return in most parts of the body. Veins have only a thin layer of vascular smooth muscle, which does not assist venous return. Veins have one-way valves.

A nurse is teaching a wellness class risk factors for cerebrovascular accidents (CVAs). Which information should the nurse include? Select all that apply. 1 Nonsmoker 2 Polycythemia 3 Lipoprotein-A 4 Insulin resistance 5 Hyperhomocysteinemia 6 Nonrheumatic atrial fibrillation

23456Persons with polycythemia, lipoprotein-A, insulin resistance, hyperhomocysteinemia, and nonrheumatic atrial fibrillation are at increased risk for CVAs. Smokers, not nonsmokers, are at increased risk.

Which type of complication of rheumatoid arthritis is seen in clients taking steroids? 1 Pulmonary changes 2 Vascular changes 3 Skeletal changes 4 Lymph changes

2 The vascular changes (thrombosis, myocardial infarctions, cerebrovascular occlusions) are primarily noted in individuals receiving steroid therapy. Lymphadenopathy of the nodes close to the affected joints may develop, but it is not from taking steroids. Changes in skeletal muscle are often noted in the form of nonspecific atrophy secondary to joint dysfunction, not from steroids. Diffuse pulmonary fibrosis may occur because of immunologically mediated immune complex deposition, not from steroids.

A client has a seizure disorder. When the client experiences symptoms such as headache, malaise, and a sense of depression, which term should the nurse use to describe these symptoms? 1 Complex 2 Prodroma 3 Postictal 4 Grand ma

2 These symptoms can be exhibited as part of the prodromal phase, and often are indicative of a pending seizure. The postictal phase follows a seizure. Complex is a type of seizure that may have impaired consciousness. Grand mal is the traditional terminology for a tonic-clonic seizure.

The nurse will teach a client with cancer of the colon about which type of treatment that is most common? 1 Herbal therapy 2 Surgery 3 Radiation 4 Chemotherap

2 Treatment for cancer of the colon is surgical resection or removal of the rectum with formation of a permanent colostomy. Radiation and chemotherapy are of less use, although they can be adjuvant or neoadjuvant. Herbal therapy is not a common treatment for colon cancer.

A client experiences spinal cord damage after a motorcycle accident. The nurse will monitor the client for which type of shock? 1 Septic 2 Vasogenic 3 Anaphylactic 4 Hypovolemic

2 Vasogenic shock results from an increase in parasympathetic nervous stimulation and a decrease in sympathetic nervous stimulation, usually from spinal cord or medulla trauma. Septic shock is due to infection. Anaphylactic shock is a hypersensitivity reaction usually due to penicillin, latex, or food allergies. Hypovolemic shock results from a decrease in blood volume due to hemorrhage.

A client has end-stage chronic renal disease. The nurse monitors the client for renal osteodystrophy and spontaneous bone fractures. What is the rationale for the nurse's actions? 1 Erythropoietin secretion is impaired. 2 Deficiency of active vitamin D occurs. 3 Urea causes demineralization of bone. 4 Excess potassium leaches calcium from bone

2 Vitamin D, required for calcium absorption in the digestive tract, is activated in the kidneys. With chronic renal failure, vitamin D is not activated and calcium is not deposited in the bones. Erythropoietin does not influence bone mineralization; it affects red blood cells. Urea does not cause renal osteodystrophy; it causes azotemia. Hyperkalemia does not influence bone mineralization but does affect cardiac functioning.

A client has systolic left heart failure. Which hemodynamic change does the nurse monitor for in this client? 1 Increased ejection fraction 2 Increased left ventricular preload 3 Decreased ventricular stretching 4 Decreased white blood cell levels

2 When contractility is decreased, stroke volume falls and left ventricular end-diastolic volume increases. This causes stretching of the myocardium and an increase in preload. Ejection fraction decreases in systolic heart failure. Left heart failure does not decrease the right ventricular preload. Left heart failure does not decrease white blood cell levels.

A nurse is teaching a client with Raynaud disease to prevent vasospastic attacks. Which conditions will the nurse advise the client to avoid? Select all that apply. 1 Tropical areas 2 Cold exposure 3 Emotional stress 4 Wearing gloves 5 Warm water immersion

23 Clients with Raynaud disease experience vasospasm in the small arteries of the fingers during cold exposure or emotional stress. Tropical areas, wearing gloves, and warm water do not trigger vasospasms in Raynaud disease; in fact, gloves and warm water can help manage the symptoms.

A football player presents to the emergency department reporting right thigh pain that began after an injury to that area a few weeks earlier. A diagnosis of myositis ossificans is made. Which information should the nurse consider when planning care for this client? Select all that apply. 1 Myositis ossificans occurs only in the thigh. 2 Myositis ossificans can cause deformities. 3 Myositis ossificans is a late complication of muscle injuries. 4 Myositis ossificans is also known as heterotopic ossification. 5 Myositis ossificans is a late complication of a stress fracture

234 Myositis ossificans is also known as heterotopic ossification. It may involve the muscle or tendons, ligaments, or bones near the muscle, causing stiffness or deformity of an extremity. The etiology is not well understood, and it is a late complication of any muscle injury, but not from a fracture.

Which prescriptions from the primary healthcare provider should the nurse expect for a client recently admitted with acute pancreatitis? Select all that apply. 1 Regular diet 2 Continuous gastric suction 3 Parenteral fluids 4 Narcotic medications 5 Blood transfusions

234 Narcotic medications may be needed to relieve pain. To decrease pancreatic secretions and "rest the gland," oral food and fluids may be withheld initially and continuous gastric suction instituted. Parenteral fluids are essential to restore blood volume and prevent hypotension and shock. A regular diet is contraindicated for acute pancreatitis. Blood transfusions are not needed in acute pancreatitis.

Which assessment findings in a client would support a diagnosis of stable angina pectoris rather than an acute myocardial infarction? Select all that apply. 1 Chest pain that is severe, crushing, and prolonged 2 Chest pain relieved by nitrates 3 Chest pain associated with exercise 4 Chest pain associated with stress 5 Chest pain not relieved with res

234 Stable angina is caused by gradual luminal narrowing and hardening of the arterial walls, so that affected vessels cannot dilate in response to increased myocardial demand associated with physical exertion or emotional stress. The pain is usually relieved by rest and nitrates; lack of relief indicates an individual may be developing infarction.

nurse is teaching about the causes of chronic kidney disease. Which information should the nurse include? Select all that apply. 1 Infection 2 Hypertension 3 Diabetes mellitus 4 Chronic pyelonephritis 5 Toxic injury to kidney cells 6 Acute bladder neck obstruction

234 Systemic diseases, such as hypertension and diabetes mellitus, or intrinsic kidney diseases, such as chronic pyelonephritis, are causes of chronic kidney disease. Infection, acute bladder neck obstruction, and toxic injury to kidney cells are more likely associated with acute kidney injury than with chronic kidney disease. q

Which information indicates the nurse has a correct understanding of the causes for type II hypersensitivity tissue injury? Select all that apply. 1 IgE production increases, causing sensitization. 2 Antibody opsonization of cells leads to subsequent phagocytosis. 3 Antibodies stimulate cell receptors, causing cells to malfunction. 4 Antibodies activate complement, and subsequent lysis of target cells occurs. 5 Cytotoxic T lymphocytes destroy target cells directly

234 Type II hypersensitivities involve production of tissue-specific antibodies that may target opsonization of cells, leading to subsequent phagocytosis; stimulate cell receptors, causing cells to malfunction; and activate complement, causing subsequent lysis of target cells. Type I hypersensitivities involve increased levels of IgE. Type IV hypersensitivity reactions are mediated by the action of cytotoxic T cells or T-helper type 1 cells on target cells.

A client has seizures. The nurse can classify the seizures according to which criteria? Select all that apply. 1 Age of onset 2 Site of origin 3 Electroencephalogram (EEG) correlates 4 Response to therapy 5 Clinical manifestation

2345

Which clinical manifestations and complications will the nurse closely monitor for in a client with chronic gastritis? Select all that apply. 1 Obesity 2 Achlorhydria 3 Gastric cancer 4 Pernicious anemia 5 Gastrointestinal bleeding

2345 Chronic damage to the gastric mucosa can cause cellular changes that lead to achlorhydria, gastric cancer, and mucosal erosions that can cause gastrointestinal bleeding. Less available intrinsic factor can lead to pernicious anemia. Individuals with chronic gastritis usually are not obese.

A client has osteoarthritis in the hands. Which clinical manifestations will the nurse observe upon assessment? Select all that apply. 1 Ulnar drift 2 Joint swelling 3 Joint stiffness 4 Joint tenderness 5 Heberden nodes 6 Subcutaneous nodule

2345 Clinical manifestations of osteoarthritis include joint stiffness, increased pain with movement, joint tenderness and swelling as well as joint deformity, and Heberden nodes. Ulnar drift and subcutaneous nodules are associated with rheumatoid arthritis of the hand.

client has osteoarthritis in the hands. Which clinical manifestations will the nurse observe upon assessment? Select all that apply. 1 Ulnar drift 2 Joint swelling 3 Joint stiffness 4 Joint tenderness 5 Heberden nodes 6 Subcutaneous nodule

2345 Clinical manifestations of osteoarthritis include joint stiffness, increased pain with movement, joint tenderness and swelling as well as joint deformity, and Heberden nodes. Ulnar drift and subcutaneous nodules are associated with rheumatoid arthritis of the hand.

A nurse is teaching the staff about incontinence. Which information should the nurse include? Select all that apply. 1 Urge incontinence is a normal part of aging. 2 Stress incontinence can affect men after prostate surgery. 3 Urge incontinence can be a result of involuntary detrusor contractions. 4 Stress incontinence is caused by increased intra-abdominal pressure. 5 Functional incontinence is often the result of dementia or immobilit

2345 Functional incontinence is incontinence related to dementia or immobility. Stress incontinence is caused by increased intra-abdominal pressure and is most common in women over age 60 and in men who have had prostate surgery. Urge incontinence is caused by involuntary detrusor contractions. Incontinence is never normal under any circumstances, and it is not a normal part of aging.

A nurse is teaching the staff about incontinence. Which information should the nurse include? Select all that apply. 1 Urge incontinence is a normal part of aging. 2 Stress incontinence can affect men after prostate surgery. 3 Urge incontinence can be a result of involuntary detrusor contractions. 4 Stress incontinence is caused by increased intra-abdominal pressure. 5 Functional incontinence is often the result of dementia or immobility

2345 Functional incontinence is incontinence related to dementia or immobility. Stress incontinence is caused by increased intra-abdominal pressure and is most common in women over age 60 and in men who have had prostate surgery. Urge incontinence is caused by involuntary detrusor contractions. Incontinence is never normal under any circumstances, and it is not a normal part of aging.

Which information indicates a nurse has a correct understanding about prevention of a urinary tract infection in a normal male bladder environment? Select all that apply. 1 A short urethra 2 Low pH of urine 3 High osmolarity of urea 4 Flushing effects of voiding 5 Presence of Tamm-Horsfall protein

2345 Most bacteria are washed out of (flushed from) the urethra during micturition (voiding). The low pH and high osmolality of urea, the presence of Tamm-Horsfall protein, and secretions from the uroepithelium provide a bactericidal effect. A shorter urethra, as well as a colonization route from both the rectum and vagina to the urethra, is thought to explain the increased rate of infections in women.

The nurse is caring for a client diagnosed with osteoarthritis of the knee. Which clinical manifestations should the nurse expect related to the diagnosis? Select all that apply. 1 Joint stiffness that is worse after movement 2 Pain that gets worse with exercise 3 Joint enlargement 4 Joint deformity 5 Joint swelling

2345 Pain that gets worse with exercise, joint deformity, joint swelling, and joint enlargement are all clinical manifestations of osteoarthritis. Joint stiffness occurs as joint movement begins (not after movement), and it dissipates rapidly after a few minutes.

Which information should the nurse include when teaching a client about cirrhosis? Select all that apply. 1 Cirrhosis develops rapidly. 2 Portal hypertension can develop. 3 The pathology is not clearly understood. 4 Severity is determined by the amount of toxin. 5 Removal of the toxin slows the progression of liver damag

2345 Portal hypertension can develop from fibrosis or obstruction of biliary channels from cirrhosis. The pathology is not clearly understood, but alcohol and other toxins have been linked to cirrhosis. The severity is determined by the amount of toxin. Removal of the toxin/contaminant slows the progression of liver damage and enhances the process of regeneration. Cirrhosis is a process of cellular injury that develops over years; it does not develop rapidly.

hen planning care for a client with rheumatoid arthritis, which pathophysiological processes should the nurse remember? Select all that apply. 1 Infection 2 Joint deformity 3 Inflammation 4 Immune response 5 Pannus formation

2345 Rheumatoid arthritis is an inflammatory and immune process that leads to pannus formation on the bone, which causes joint deformity. There is no infection present.

A client with type 2 diabetes has hyperosmolar hyperglycemic nonketotic syndrome. Which findings would the nurse observe upon assessment? Select all that apply. 1 Mild hyperglycemia 2 Neurologic changes 3 Severe dehydration 4 No or slight ketosis 5 Poor skin turgor

2345Hyperosmolar hyperglycemic nonketotic syndrome is characterized by severe dehydration with poor skin turgor, severe hyperglycemia, no or slight ketosis, and a high mortality rate. Neurologic changes, such as stupor, are involved.

A nurse is caring for a client with a spinal cord injury. Which assessment findings will cause the nurse to conclude the client is in spinal shock? Select all that apply. 1 Blurred vision 2 Flaccid muscles 3 Absence of sensation 4 Loss of bladder control 5 Transient drop in blood pressure

2345Spinal shock involves all skeletal muscles; bladder, bowel, and sexual function; paralysis and flaccidity in muscles; absence of sensation; and transient drop in blood pressure. Spinal shock does not involve blurred vision; autonomic hyperreflexia does.

A client has rhabdomyolysis. The nurse should monitor this client for which complications? Select all that apply. 1 Aprotinin 2 Renal failure 3 Hyperkalemia 4 Respiratory acidosis 5 Disseminated intravascular coagulatio

235 Although relatively rare, rhabdomyolysis has many causes and can result in serious complications, including hyperkalemia (because of the release of intracellular potassium into the circulation), metabolic acidosis (from liberation of intracellular phosphorus and sulfate), acute renal failure (myoglobin precipitates in the tubules, obstructing flow through the nephron and producing injury), and even disseminated intravascular coagulation (likely caused by activation of the clotting cascade by sarcolemma damage and release of intracellular components from the damaged muscles). Respiratory acidosis is not a complication of rhabdomyolysis. Aprotinin is a protease inhibitor that blocks matrix metalloproteinases, thereby reducing the activity of enzymes such as collagenases; it is for tendinopathy, not rhabdomyolysis.

A nurse is teaching a group of adults about risk factors for ulcerative colitis. Which information should the nurse include? Select all that apply. 1 Diet 2 Genetics 3 White ethnicity 4 Native American ethnicity 5 Immunologic disorders

235 The familial tendency to develop ulcerative colitis and the occurrence of disease in identical twins support a genetic theory of causation. Perhaps most significant are humoral immunologic factors such as type 2 T-helper cell (Th2) response and activated macrophages associated with the disease. An immunologic basis for the disease is supported by the fact that ulcerative colitis often accompanies other autoimmune conditions, such as systemic lupus erythematosus. The disease is more prevalent among white populations. Jewish descent, not Native American, is a risk factor. Diet has not been linked as an increased risk for developing ulcerative colitis.

A client has bone destruction. Which bone destruction patterns are typical? Select all that apply. 1 Circular 2 Permeative 3 Moth-eaten 4 Asymmetric 5 Geographic

235 The three patterns of bone destruction include the geographic, moth-eaten, and permeative patterns. Circular and asymmetric patterns are not recognized destruction patterns.

A nurse is describing the classification criteria for the severity of renal dysfunction in acute kidney injury. Which order from least severe to most severe indicates the nurse has a correct understanding? 1. Loss 2. Risk 3. Injury 4. End-stage kidney disease 5. Failure

23514 RIFLE

What clinical findings in a client suggest to the nurse a diagnosis of nephritic syndrome? Select all that apply. 1 Edema 2 Proteinuria 3 Hypertension 4 Hyperlipidemia 5 Hypothyroidism 6 Microscopic hematuria

236 Clinical manifestations of nephritic syndrome include proteinuria, hypertension, and microscopic (or gross) hematuria. Edema, hypothyroidism, and hyperlipidemia are associated with nephrotic syndrome.

A nurse is teaching the client about cholelithiasis. Which information should the nurse include in the teaching session? Select all that apply. 1 Risk factors include male gender. 2 Cholesterol stones are the most common. 3 This means inflammation of the gallbladder. 4 Cholelithiasis is more prevalent in developed countries. 5 Pigmented stones are made primarily of cholestero

24 Two types of gallstones exist: cholesterol stones and pigmented stones; cholesterol stones are the most common and are made of cholesterol. Cholelithiasis is most prevalent in developed countries. Risk factors include obesity, middle age, female gender, American Indian ancestry, oral contraceptive use, and gallbladder disease. Inflammation of the gallbladder or cystic duct is known as cholecystitis, not cholelithiasis. Pigmented stones form from increased levels of unconjugated bilirubin (not cholesterol), which binds with calcium.

A nurse is teaching a health and wellness class about nutritional deficiencies associated with osteoporosis. Which information from the class members indicates teaching was successful for nutritional deficiencies associated with osteoporosis? Select all that apply. 1 Iron deficiency 2 Calcium deficiency 3 Sodium deficiency 4 Vitamin D deficiency 5 Magnesium deficienc

245 Osteoporosis is linked to a decreased or inadequate dietary calcium intake and lack of vitamin D, as well as possibly decreased magnesium. Iron and sodium deficiencies have not been linked to this disorder. However, excessive sodium intake has been associated with osteoporosis.

A nurse is teaching the client about overactive bladder syndrome. Which information from the client indicates teaching was successful? Select all that apply. 1 Increases the risk of bladder cancer 2 Associated with urgency and frequency 3 Decreases the risk of urinary tract infections 4 Results in urinary retention with overflow 5 Most common treatment is antimuscarinic agents

245 Overactive bladder syndrome is associated with urgency and frequency, and it results in urinary retention with overflow. Antimuscarinic agents are the most common treatment. There is no associated increased risk of bladder cancer, but it can increase, not decrease, the risk for urinary tract infections.

A nurse is teaching an adult community group about the risk factors for esophageal cancer. Which information should the nurse include? Select all that apply. 1 Cirrhosis 2 Male gender 3 Low-fiber diet 4 Tobacco use 5 Chronic alcohol intak

245 The male gender, tobacco use, and chronic alcohol intake all increase the risk of esophageal cancer. Cirrhosis and a low-fiber diet are not risk factors for esophageal cancer.

A nurse is teaching about Crohn disease. Which information should the nurse include? Select all that apply. 1 Common stool pattern of bloody diarrhea about 4 times per day 2 Cobblestone appearance of the gastrointestinal tract 3 Involvement of only the mucosal layer 4 Presence of "skip" lesions 5 Increased cancer risk 6 Weight loss

2456 In Crohn disease, there is a cobblestone appearance of the gastrointestinal tract; the entire intestinal wall is involved; there are "skip" lesions involved; there is an increased cancer risk; and there is weight loss. Bloody diarrhea and involvement of only the mucosal layer occur with ulcerative colitis. In Crohn disease, diarrhea is common but blood therein is not.

A nurse is teaching the staff about physiologic mechanisms that protect against urinary tract infections. Which information should the nurse include in the teaching session? Select all that apply. 1 Prostatitis 2 Acidic urine 3 Hypercalcemia 4 Tamm-Horsfall protein 5 Uroepithelial secretion

245Several factors normally combine to protect against urinary tract infections. Most bacteria are washed out of the urethra during micturition. The low pH (acidic urine) and high osmolality of urea, the presence of Tamm-Horsfall protein, and secretions from the uroepithelium provide a bactericidal effect. Inflammation of the prostate (prostatitis) causes urinary stasis and increases the risk of urinary tract infection, whereas hypercalcemia promotes renal calculi and urinary stasis. q

An older adult with osteoporosis lives alone and falls in the bathtub. The client experiences tremendous left hip/leg pain with any movement and is not found until two days later. Laboratory results indicate creatine kinase = 6000 units/l and potassium = 7.0. The client is placed on hemodialysis and the family arrives from out of town. The family is visibly upset and wants to know what is happening. How should the nurse respond? Select all that apply. 1 Hemodialysis assists with rehydration. 2 The hemodialysis is temporary and removes the high levels of potassium. 3 The increased intravenous fluids are overloading the heart. 4 The kidneys are being damaged by myoglobin released from the muscles. 5 Hemodialysis is part of the protocol in preparing for a hip replacement. 6 The family member fell and is experiencing complications from the fall.

246 Temporary dialysis is used in rhabdomyolysis to remove the myoglobin and high potassium levels from the blood in an effort to preserve renal function. The myoglobin released from the damaged muscle after lying in one position for two days is being filtered by the kidney. The myoglobin precipitates in the tubules obstruct the flow through the nephron and produce injury. The client did fall and is experiencing complications from the fall. Hemodialysis does not rehydrate; there are other methods for rehydration, like intravenous fluids. Hemodialysis is for the kidneys, not the heart, and there is no data in the question to suggest heart problems. Hemodialysis is not part of the protocol for a hip replacement procedure.

hich clients are at risk for development of drug-induced parkinsonism? Select all that apply. 1 One taking antifungal drugs 2 One taking antiemetic drugs 3 One taking antibiotic medications 4 One taking antihypertensive medications 5 One taking nonsteroidal anti-inflammatory drugs

24Drug-induced parkinsonism may be caused by neuroleptics, antiemetics, and antihypertensives. Drug-induced parkinsonism is not associated with antifungal, antibiotic, or nonsteroidal anti-inflammatory drugs.

A nurse instructs the staff about cellular injury of the myocardium. Which information from the staff indicates teaching was effective? Select all that apply. 1 Electrocardiogram (ECG) changes due to hypoxia are visible after approximately 120 seconds. 2 Myocardial cells remain viable if blood flow returns within 20 minutes. 3 Cardiac cells can withstand ischemic conditions for about 40 minutes. 4 After 20 seconds of decreased blood flow, myocardial cells become cooler. 5 Within several minutes of coronary occlusion, the heart cells lose the ability to contract.

25 Cardiac cells remain viable for approximately 20 minutes under ischemic conditions, not 40. After several minutes, the heart cells lose the ability to contract, and cardiac output decreases. ECG changes are visible after 30 to 60 seconds of hypoxia. After 8 to 10 seconds of insufficient blood flow, the myocardial cells are already cooler.

Which clients have basal ganglia motor syndromes? Select all that apply. 1 A client who is diagnosed with poliomyelitis 2 A ciient who is diagnosed with Parkinson disease 3 A client who is diagnosed with a spinal cord injury 4 A client who is diagnsoed with Tourette syndrome 5 A client who is diagnosed with Huntington disease

25Symptoms of Parkinson disease and Huntington disease are caused by degeneration of the basal ganglia. Poliomyelitis is a lower motor neuron syndrome. Spinal cord injury, depending upon the break, can affect upper or lower motor neurons. Tourette syndrome could be a frontal-striatal-thalamic dysfunction, but the pathophysiology is unclear.

Which client will the nurse closely observe for the development of diabetic ketoacidosis? 1 A client with type 2 diabetes with an infection 2 A client with type 1 diabetes with an infection 3 A client with type 2 diabetes with excessive physical exercise 4 A client with type 1 diabetes with excessive physical exercise

2A client with type 1 diabetes with an infection is the most predisposed to develop diabetic ketoacidosis. A client with type 2 diabetes with an infection is predisposed to hyperglycemic nonketotic syndrome, not diabetic ketoacidosis. A client with type 2 or type 1 diabetes with excessive physical exercise may experience hypoglycemia, not ketoacidosis.

Which client meets the diagnostic criteria for diabetes mellitus? 1 A client with a glycosylated hemoglobin of 5.5% 2 A client with a fasting blood glucose of 128 mg/dl 3 A client with a random serum glucose of 197 mg/dl 4 A client with a serum glucose of 185 mg/dl two hours after taking 75 grams of glucose

2A fasting blood glucose of 128 mg/dl meets the criteria for diabetes mellitus. A fasting plasma glucose ≥ 126 mg/dl meets the criteria for a diagnosis of diabetes mellitus. A glycosylated hemoglobin of 5.5% does not meet the criteria of > 6.5%; a two-hour oral glucose tolerance test result of 185 mg/dl does not meet the criteria of > 200 mg/dl; or a random serum glucose of 197 mg/dl does not meet criteria for > 200 mg/dl with classic symptoms of hyperglycemia.

A client with meningitis has nuchal rigidity. How should the nurse interpret this finding? 1 Projectile vomiting that indicates brain abscess 2 Neck stiffness that indicates meningeal irritation 3 Difficulty moving that is diagnostic for parkinsonism 4 Mild frontal headache that is diagnostic for encephalit

2Nuchal rigidity is neck stiffness that indicates irritation of the meninges due to inflammation and infection. Nuchal rigidity is not projectile vomiting, difficulty moving, or mild frontal headache.

Which class of vertebral injury does the nurse expect in the patient with spinal cord trauma whose computed tomography (CT) scan shows that the patient's vertebral body has shattered into several fragments? 1 Dislocation 2 Burst fracture 3 Simple fracture 4 Wedge fracture

2Because the CT scan shows that the patient's vertebral body is shattered into fragments, the nurse expects that the patient has burst fracture or comminuted fracture. If the CT scan shows changes in the position of the vertebral column, then the nurse would expect that the patient has dislocation. If the CT scan shows a single break, then the nurse would expect a simple fracture. If the CT scan shows that the patient's vertebral body was compressed anteriorly, then the nurse would expect that the patient has compressed fracture or wedge fracture.

A client has nephrotic syndrome. When the nurse checks the 24-hour urine laboratory report, which substance will the nurse observe being lost in large amounts? 1 Blood 2 Protein 3 Sodium 4 Glucose

2By definition, nephrotic syndrome involves loss of large amounts of protein in the urine. Hematuria is not characteristic of nephrotic syndrome but is common in nephritic syndrome. Sodium and water are reabsorbed in nephrotic syndrome. Glucose loss in urine is not characteristic of nephrotic syndrome.

The nurse is caring for a client diagnosed with increased intracranial pressure. Which assessment finding should indicate to the nurse that the client's brain has experienced an uncal herniation? 1 Pinpoint pupils 2 Cheyne-Stokes breathing 3 Supraventricular tachycardia 4 Increased restlessness and agitatio

2Clinical manifestations of an uncal brain herniation include abnormal breathing patterns, such as Cheyne-Stokes breathing. Other clinical manifestations include dilated and fixed pupils and a decreasing level of consciousness and posturing. Supraventricular tachycardia and increased restlessness and agitation are not associated with this type of herniation.

A nurse is caring for a client with a tumor in the wall of the brain ventricle. The nurse is caring for which client? 1 A client with a neurofibroma 2 A client with an ependymoma 3 A client with a neurilemmoma 4 A client with a glioblastoma multiforme

2Ependymoma is located in the wall of the brain ventricles. Neurilemmoma is located in the cranial nerves, usually cranial nerve VIII. Neurofibroma is an extramedullary tumor in the spinal cord. A glioblastoma is predominantly located in the cerebral hemisphere.

A client is hospitalized with a malignant tumor that secretes parathyroid hormone. What signs or symptoms should the nurse monitor to detect the resulting electrolyte imbalance? 1 Serum sodium, renal stones, and urine pH 2 Serum calcium, renal stones, and urine pH 3 Serum sodium, Chvostek sign, and Trousseau sign 4 Serum calcium, Chvostek sign, and Trousseau sign

2Excess parathyroid hormone causes hypercalcemia, which manifests as hypercalciuria, alkaline urine, and hypercalcemia, predisposing to kidney stones. Parathyroid hormone does not cause a sodium imbalance. The Chvostek and Trousseau signs are related to hypocalcemia, not hypercalcemia.

client is 64 years old and has experienced a hemorrhagic cerebrovascular accident (CVA). Based on the pathophysiology of a hemorrhagic cerebrovascular disease, a nurse should anticipate finding that the client had which risk factors? 1 Polycythemia or thrombocythemia 2 Hypertension and cerebral aneurysm 3 Atrial fibrillation or infective endocarditis 4 Insulin resistance and atherosclerosis

2Hypertension and cerebral aneurysm are risk factors for hemorrhagic CVA. Polycythemia and thrombocythemia are risk factors for ischemic CVA. Atrial fibrillation and infective endocarditis are risk factors for ischemic (embolic) CVA. Insulin resistance and atherosclerosis are risk factors for ischemic CVA.

hich client is most at risk for developing chronic kidney disease? 1 A client with myocarditis 2 A client with hypertension 3 A client with pernicious anemia 4 A client with chronic obstructive pulmonary disease

2Hypertension is a major cause of chronic kidney disease, because organs can be damaged without obvious signs and symptoms. Myocarditis, pernicious anemia, and chronic obstructive pulmonary disease do not cause chronic kidney disease.

A patient with back pain reports pain when sneezing and coughing. During the assessment, the nurse finds that the patient has impaired sensation at L4-L5 levels. What other clinical manifestation should the nurse anticipate to find in the patient? 1 Decreased biceps reflex 2 Decreased ankle jerk reflex 3 Presence of paroxysmal hypertension 4 Presence of severe edema in the ches

2Impaired sensation at L4-L5 levels and pain while sneezing and coughing indicate that the patient has a herniated intervertebral disk (HID). HID is associated with motor weakness in the lower extremities and decreased ankle jerk reflex. Decreased biceps reflex is associated with cervical injury and HID of a lower cervical disk. A patient with autonomic hyperreflexia has paroxysmal hypertension. HID is not associated with accumulation of fluid in the chest cavity; therefore, the patient will not have edema in the chest.

A client fell from a bicycle while riding without a helmet and was confused with no loss of consciousness. The headache resolved by the next day. Which term will the nurse use to describe this type of brain injury? 1 Contusion 2 Concussion 3 Postconcussive syndrome 4 Epidural hematoma

2Mild concussion does cause confusion but involves no loss of consciousness and may cause head pain for up to a few days. A contusion may be evidenced by immediate loss of consciousness (generally accepted to last no longer than 5 minutes), loss of reflexes (the individual falls to the ground), transient cessation of respiration, a brief period of bradycardia, and a decrease in blood pressure (lasting 30 seconds to a few minutes). A loss of consciousness would occur with epidural hematoma. A postconcussive syndrome, including headache, nervousness or anxiety, irritability, insomnia, depression, inability to concentrate, forgetfulness, and fatigability, may exist after the injury.

Which information indicates the nurse has a correct understanding of the causes for the development of sensory and motor symptoms in clients with multiple sclerosis? 1 Loss of γ-aminobutyric acid (GABA) neurotransmitter function in the spinal cord 2 Immunologic demyelination of axons in the central nervous system 3 Receptor abnormalities at neuromuscular junctions and in the ganglia 4 Degeneration of sensory and motor neurons in the peripheral nervous system

2Multiple sclerosis is an autoimmune disease that causes axonal demyelination of central nervous system (brain and spinal cord) neurons. Increased glutamate, not GABA, is associated with multiple sclerosis. Myasthenia gravis results from a defect in nerve impulse transmission at the neuromuscular junction. Multiple sclerosis affects the central nervous system, not the peripheral nervous system.

A patient with a history of spinal shock was admitted to the emergency department. Upon assessing the patient, the nurse find that the patient's blood pressure is 210/120 mm Hg and heart rate is 30 beats/minute. What does the nurse anticipate finding in the patient's diagnostic reports? 1 Edema at C3 level 2 Lesions at T6 level 3 Fracture at L5 level 4 Inflammation at S2 leve

2Paroxysmal hypertension, bradycardia, and history of spinal shock are the clinical manifestations of autonomic hyperreflexia. Patients with autonomic hyperreflexia most likely have lesions in the spinal cord at the T6 level or above. Therefore, the nurse would find lesions at T6 level in the patient's diagnostic reports. Presence of edema at C3 level impairs diaphragm functioning. A fracture at the L5 level causes spondylolisthesis or herniation but not paroxysmal hypertension. Autonomic hyperreflexia is not associated with inflammation at the S2 level.

hich assessment finding will help a nurse determine the client may have metabolic syndrome? 1 Blood pressure 120/70 mm Hg 2 Plasma triglycerides 160 mg/dl 3 Decreased waist circumference 4 Fasting plasma glucose 85 mg/dl

2Plasma triglycerides equal to or greater than 150 mg/dl are characteristic of metabolic syndrome. A blood pressure greater than or equal to 130/85 mm Hg is indicative of metabolic syndrome. Increased waist circumference, not decreased, is indicative of metabolic syndrome. Fasting plasma glucose equal to or greater than 100 mg/dl is associated with metabolic syndrome.

A patient presents with quadriparesis, respiratory insufficiency with shortness of breath, and extreme difficulty swallowing. Which medication will be most beneficial for the patient? 1 Venlafaxine (Effexor) 2 Azathioprine (Imuran) 3 Acetaminophen (Tylenol) 4 Phenoxybenzamine (Dibenzaline)

2Quadriparesis, respiratory insufficiency with shortness of breath, and extreme difficulty swallowing indicate that the patient has myasthenia gravis, which is an autoimmune disorder, which has progressed to myasthenic crisis. Azathioprine (Imuran) is an immunosuppressant that helps reduce the symptoms of myasthenia gravis. Antidepressants, such as venlafaxine (Effexor), or analgesics, such as acetaminophen (Tylenol), do not alter cholinesterase levels and do not reduce the symptoms of myasthenia gravis. Phenoxybenzamine (Dibenzaline) causes bladder or bowel emptying and alleviates the symptoms of autonomic hyperreflexia syndrome.

A client who experienced a head injury to the frontal lobe develops gegenhalten (paratonia). Which clinical manifestation would the nurse expect to observe? 1 Limp, atrophied muscles 2 Resistance to passive movements 3 Decerebrate rigidity of lower extremeties 4 Writhing, twisted movements of both arms

2Resistance to passive movements is characteristic of gegenhalten (paratonia), a type of hypertonia. Limp, atrophied muscles occur with hypotonia. Decerebrate rigidity is termed gamma, not gegenhalten. Writhing, twisted movements is termed chorea, not gegenhalten.

An elderly male with long-term alcohol abuse falls and experiences a hematoma between the dura mater and the brain. Which type of hematoma will the nurse most likely observe documented on the chart? 1 Epidural 2 Subdural 3 Extradural 4 Intracerebral

2Subdural hematomas (between the dura mater and the brain) are commonly found in the elderly and persons who abuse alcohol (groups that are at elevated risk of falls), and they are generally from venous rather than arterial bleeding. Extradural or epidural bleeding (between the dura mater and the skull) and intracerebral hematomas (located within the brain) are injuries not generally observed in this population after an alcohol-induced fall, but they may be associated with younger clients and acute trauma.

An elderly male with long-term alcohol abuse falls and experiences a hematoma between the dura mater and the brain. Which type of hematoma will the nurse most likely observe documented on the chart? 1 Epidural 2 Subdural 3 Extradural 4 Intracerebra

2Subdural hematomas (between the dura mater and the brain) are commonly found in the elderly and persons who abuse alcohol (groups that are at elevated risk of falls), and they are generally from venous rather than arterial bleeding. Extradural or epidural bleeding (between the dura mater and the skull) and intracerebral hematomas (located within the brain) are injuries not generally observed in this population after an alcohol-induced fall, but they may be associated with younger clients and acute trauma.

nurse is caring for a client who has diabetes insipidus (DI) from damage to the posterior pituitary caused by a cerebrovascular accident (stroke). The nurse is caring for which type of DI? 1 Vascular 2 Central 3 Psychogenic 4 Nephrogenic

2When DI develops from injury to the central nervous system (CNS), it is called neurogenic or central DI. Vascular and psychogenic are not classifications of diabetes insipidus. Nephrogenic DI is not caused by injury to the CNS but rather results from an inadequate response of the renal tubules to antidiuretic hormone.

A client has chronic pancreatitis. Which common cause of chronic pancreatitis should the nurse consider when planning care for this client? 1 Gallstones 2 Viral infection 3 Alcohol abuse 4 Bacterial infection

3

A client has poor vitamin A absorption. What should the nurse monitor for in this client? 1 Petechiae 2 Osteoporosis 3 Night blindness 4 Neurologic defect

3

A coworker asks a nurse what the most common cause of chronic antral gastritis is. What is the nurse's best response? 1 It is a viral infection. 2 It is a fungal infection. 3 It is a bacterial infection. 4 It is a parasite infestation

3

A nurse is asked about age-related bone loss and when it begins. How should the nurse reply? 1 It begins at 20. 2 It begins at 30. 3 It begins at 40. 4 It begins at 50

3

A nurse is assessing a client with suspected appendicitis. Which assessment finding would support the diagnosis? 1 Pain starting in the periumbilical area and then recurring in the left lower quadrant 2 Pain starting in the periumbilical area and then recurring in the left upper quadrant 3 Pain starting in the periumbilical area and then recurring in the right lower quadrant 4 Pain starting in the periumbilical area and then recurring in the right upper quadrant

3

A nurse is caring for a client with postrenal acute kidney injury. The nurse is caring for which client? 1 A client with blood loss 2 A client with hypovolemia 3 A client with an obstruction 4 A client with nephron damage

3

A nurse is reviewing the chart of a client with bladder cancer and finds the following: T1, N1, and M0. How should the nurse interpret this finding? 1 Cancer has metastasized. 2 Cancer has not traveled to the lymph nodes. 3 Cancer has invaded the connective tissue. 4 Cancer has moved to stage IV

3

A client asks the nurse the difference between rheumatoid arthritis (RA) and ankylosing spondylitis (AS). What is the best response by the nurse? 1 AS affects the synovial membrane. 2 AS is a localized inflammation. 3 AS is an overformation of bone. 4 AS affects the hands firs

3 A key difference of AS is that there is an overformation of bone. RA affects the synovial membrane. They are both systemic inflammations. RA affects the hands and fingers first, whereas AS affects the spine and sacroiliac joints.

A client asks the nurse the difference between rheumatoid arthritis (RA) and ankylosing spondylitis (AS). What is the best response by the nurse? 1 AS affects the synovial membrane. 2 AS is a localized inflammation. 3 AS is an overformation of bone. 4 AS affects the hands first

3 A key difference of AS is that there is an overformation of bone. RA affects the synovial membrane. They are both systemic inflammations. RA affects the hands and fingers first, whereas AS affects the spine and sacroiliac joints.

A client has a torus fracture. The nurse is caring for which client? 1 A client in whom a long bone splintered into many fragments 2 A client in whom the injury occurred at the epiphyseal growth plate 3 A client in whom the cortex of the bone buckled but did not break 4 A client in whom spongy bone was damaged but not cortical bon

3 A torus fracture occurs when the cortex of the bone buckles but does not break. A comminuted fracture involves the bone splintering into many fragments. A torus fracture does not involve injury at the epiphyseal growth plate. A torus fracture does not involve exclusive damage to spongy bone.

When viewed on a radiograph, the client's bone has a linear fracture that occurs in a straight line across the bone. Which type of fracture will the nurse be caring for in this client? 1 Spiral 2 Oblique 3 Transverse 4 Comminuted

3 A transverse fracture occurs straight across the bone horizontally. Spiral fractures encircle the bone. Oblique fractures occur at an angle to the long axis of the bone. Comminuted fracture has multiple bone fragments.

Which finding indicates a client with gout had a therapeutic response to allopurinol (Zyloprim)? 1 Increased uricase levels 2 Decreased glucose levels 3 Decreased serum urate levels 4 Increased serum allantoin levels

3 Allopurinol and febuxostat are both used to lower serum urate levels by inhibiting the activity of xanthine oxidase. Most mammals, except humans, have the enzyme uricase, which catalyzes the conversion of uric acid to allantoin, thus preventing overproduction of uric acid. Allopurinol is not given to decrease glucose.

A nurse found during the history that a client has chronically overused magnesium-aluminum antacids for a long period of time. The nurse will expect to find which laboratory result? 1 Hypokalemia 2 Hyperkalemia 3 Hypophosphatemia 4 Hyperphosphatemia

3 Antacid overuse for a long time can cause hypophosphatemia from intestinal malabsorption. Magnesium-aluminum antacids do not cause hypokalemia (which can be caused by reduced potassium intake, increased entry of potassium into the cells, or increased losses of body potassium) or hyperkalemia (which can be caused by increased potassium intake, a shift of potassium from cells to the extracellular fluid, decreased renal excretion, or drugs). Antacids can cause intestinal malabsorption, resulting in too little phosphate, not too much.

A client has problems with proprioception. Which nursing goal is the priority? 1 Enhance the sense of taste 2 Maintain cognition 3 Protect from falls 4 Improve vision

3 As with tactile dysfunction, any factor that interrupts or impairs the reception, transmission, perception, or interpretation of proprioceptive stimuli also alters proprioception and increases the risk for falls and injury. Taste, cognition, and vision are not associated with proprioception.

A nurse is caring for a client with a disorder that is characterized by poor digestion of fats, proteins, and carbohydrates. The nurse will use which term to describe this condition? 1 Lactase deficiency 2 Bile salt deficiency 3 Pancreatic insufficiency 4 Vitamin K insufficiency

3 Because the pancreas secretes lipase, proteases, and amylase, a decrease in pancreatic enzymes (pancreatic insufficiency) causes poor digestion of fats, proteins, and carbohydrates. Lactase deficiency does not involve poor digestion of fats and proteins, just the breakdown of lactose, a carbohydrate. Bile salt deficiency does not involve poor digestion of proteins and carbohydrates, just fat. Vitamin K insufficiency prolongs prothrombin time, leading to spontaneous development of purpura and petechiae; it does not affect fat, protein, or carbohydrate digestion.

Which client is demonstrating behaviors most associated with a brain stem injury? 1 A client who has decorticate rigidity 2 A client who has small, reactive, and regular pupils 3 A client who has no eye movement during oculovestibular reflex testing 4 A client whose eyes turn in the opposite direction when the head is turned from side to side

3 Brain stem injury results in abnormal or absent response during oculovestibular reflex testing. Decorticate rigidity suggests hemispheric damage above the midbrain. Metabolic imbalances usually produce small, reactive, and regular pupils. Eyes turning in the opposite direction as the head is turned from side to side is a normal response to oculocephalic reflex testing.

Which information should the nurse consider when planning care for a client with chronic gastritis? 1 There are four types of gastritis. 2 It tends to occur in young individuals. 3 It causes atrophy of the gastric mucosa. 4 Larger meals may help manage symptoms

3 Chronic gastritis causes thinning and degeneration (atrophy) of the gastric mucosa. The two types of chronic gastritis are type A (fundal) or type B (antral). It occurs most frequently in the elderly. Clients can usually manage the symptoms by eating smaller meals in conjunction with a soft, bland diet and by avoiding alcohol and aspirin. q

A client has a recurrent infection of the kidney that results in progressive inflammation, leading to tubule destruction. The nurse monitors the client for possible chronic kidney failure based on which medical diagnosis? 1 Acute cystitis 2 Functional incontinence 3 Chronic pyelonephritis 4 Painful bladder syndrome

3 Chronic pyelonephritis is a persistent or recurrent infection of the kidney leading to scarring of one or both kidneys. This scarring leads to chronic urinary tract obstruction that starts a process of progressive inflammation, destruction of the tubules, and finally impaired urine-concentrating ability, leading to chronic kidney failure. Acute cystitis is usually cured with antibiotics. Painful bladder syndrome does not lead to kidney failure. Functional incontinence is not an infection but an involuntary loss of urine attributable to dementia or immobility.

The nurse teaches young adults about the harmful effects of cigarette smoking. Which information from the young adults indicates teaching was effective? 1 Cigarettes cause an increase in high-density lipoprotein (HDL) levels. 2 Smoking will reduce low-density lipoprotein (LDL) levels. 3 Clotting is increased because of smoking. 4 Epinephrine decreases in people who smoke cigarettes

3 Cigarette smoking produces an increased thrombotic (clotting) and inflammatory state, decreased HDL, and increased LDL. Nicotine stimulates the release of catecholamines (epinephrine and norepinephrine), which increase heart rate and peripheral vascular constriction.

Which statement by the client about the manifestations of cirrhosis requires follow-up by the nurse? 1 "I may have to have fluid removed from my abdomen." 2 "I am at increased risk of bleeding." 3 "If I stop drinking, I will be cured." 4 "I may develop swelling in my ankles.

3 Cirrhosis is an irreversible disease of the liver. Stopping drinking will not cure the disease; therefore, the nurse should realize the client needs follow-up to correct this misinformation. Clients with cirrhosis may develop ascites and require the fluid to be removed. Because the liver normally produces clotting factors, clients with cirrhosis are at an increased risk of bleeding. Edema (swelling) is caused by decreased albumin production by the liver.

A client has suspected osteoporosis. What diagnostic test is the gold standard used to diagnose osteoporosis? 1 Radiologic studies (x-ray) 2 Computed tomography (CT) 3 Dual x-ray absorptiometry (DXA) 4 Magnetic resonance imaging (MRI

3 DXA is the gold standard for detecting osteoporosis. While CT and MRI have shown osteoporotic changes, as well as x-rays showing bone fractures, DXA is still the diagnostic option of choice.

A client's arms and legs are held in rigid extension. What term will the nurse use to document this position? 1 Senile posture 2 Decorticate posture 3 Decerebrate posture 4 Basal gaglion posture

3 Decerebrate posture is characterized by all four limbs rigidly extended. Senile posture is characterized by an increasingly flexed posture similar to that caused by basal ganglion dysfunction. Decorticate posture is characterized by upper extremities flexed at the elbows and held close to the body and by lower extremities that are externally rotated and extended. Basal ganglion posture refers to a stooped, hyperflexed posture with a narrow-based, short-stepped gait.

A nurse is treating a patient with diffuse axonal injury. Which condition is associated with a grade I injury? 1 Anterograde amnesia 2 Retrograde amnesia after 5 to 10 minutes 3 Confusion and disorientation with momentary amnesia 4 Retrograde and anterograde amnesia with immediate loss of consciousness

3 Diffuse axonal injury occurs over a widespread area in the brain. A patient with confusion, disorientation, and momentary amnesia is considered to have grade I injury. A patient with anterograde amnesia is considered to have grade III injury. A patient with retrograde amnesia after 5 to 10 minutes has grade II injury. A patient with retrograde and anterograde amnesia that causes immediate loss of consciousness is considered to have grade IV mild concussion.

A client has a hydroureter. How should the nurse interpret this finding? 1 The client has excessive urine flow through one of the ureters. 2 The client has a congenital defect where one ureter is larger than the other. 3 The client has dilation of one of the ureters due to obstruction and urine backup. 4 The client has a structural anomaly where all urine is diverted through only one ureter

3 Dilation of the ureter is referred to as hydroureter (accumulation of urine in the ureter) from an obstruction in the upper urinary tract. A hydroureter does not cause excessive urine flow but a backup of urine. A hydroureter is not a congenital defect but occurs from an obstruction. A hydroureter does not cause urine to be diverted through only one ureter but causes a backup of urine and dilation.

`A client has a hydroureter. How should the nurse interpret this finding? 1 The client has excessive urine flow through one of the ureters. 2 The client has a congenital defect where one ureter is larger than the other. 3 The client has dilation of one of the ureters due to obstruction and urine backup. 4 The client has a structural anomaly where all urine is diverted through only one ureter.

3 Dilation of the ureter is referred to as hydroureter (accumulation of urine in the ureter) from an obstruction in the upper urinary tract. A hydroureter does not cause excessive urine flow but a backup of urine. A hydroureter is not a congenital defect but occurs from an obstruction. A hydroureter does not cause urine to be diverted through only one ureter but causes a backup of urine and dilation.

A client has a temporary but total displacement of a bone from its normal position. Which term should the nurse use to describe this type of injury? 1 Fracture 2 Nonunion 3 Dislocation 4 Subluxation

3 Dislocation is the displacement of one or more bones in a joint in which the opposing joint surfaces entirely lose contact. A fracture is a break in the bone, and nonunion is the failure of bone ends to grow together after a fracture. If the contact is only partially lost between the two surfaces, it is called a subluxation.

A nurse is asked by a client to describe diverticula. What is the nurse's best response? 1 Ulcers that form in the colon 2 Perforations of the colon wall 3 Outpouchings of the colon mucosa 4 Areas of muscle hypertrophy in the colon

3 Diverticula are herniations or saclike outpouchings of the mucosa and submucosa through the muscle layers, usually in the wall of the sigmoid colon. Diverticula are not ulcers (breaks in the protective mucosal lining), perforations of the colon wall (breaks through the entire gastrointestinal wall), or areas of hypertrophy (enlargement).

A client has severe osteoarthritis and is unable to walk without assistance. As a nurse enters the room in response to the call light, the client says, "Oh, how embarrassing! I just wet the bed. I know you are busy, so I held it as long as I could, but then it just came." What is the technical term that the nurse should use with another health professional to describe the client's incontinence? 1 Urge incontinence 2 Stress incontinence 3 Functional incontinence 4 Dyssynergia incontinence

3 Functional incontinence is secondary to immobility or dementia. Urge incontinence involves immediate voiding after the urge occurs. Stress incontinence occurs from increased abdominal pressure and sphincter laxity. Dyssynergia is loss of coordinated neuromuscular contraction from lesions in the upper motor neurons of the brain and spinal cord that may lead to incontinence.

ich information indicates the nurse has a correct understanding of the pathophysiologic mechanisms that lead to peptic ulcer formation from chronic infection with Helicobacter pylori (H. pylori )? 1 H. pylori inhibits the proton pumps in the gastric lining. 2 H. pylori inhibits prostaglandins, which causes decreased mucus production. 3 H. pylori produces toxins and enzymes that promote inflammation and ulceration. 4 H. pylori causes the pyloric sphincter to open prematurely, releasing acid into the duodenum.

3 H. pylori releases toxins and enzymes that promote inflammation and ulceration as well as causing death of mucosal epithelial cells and elevated levels of gastrin and pepsinogen. Inhibition of proton pumps would decrease secretion of gastric acid and protect against peptic ulcers. Long-term use of nonsteroidal anti-inflammatory drugs inhibits prostaglandins and results in reduced maintenance of the mucosal barrier and decreased bicarbonate secretion. Cigarette smoking stimulates acid production. Presence of H. pylori does not cause premature release of acid.

client has chronic Helicobacter pylori infection and is at most risk for which type of cancer? 1 Colon 2 Breast 3 Gastric 4 Bladde

3 Helicobacter pylori infection, which also causes peptic ulcer disease, is considered a primary risk factor for gastric cancer. It is not associated with colon cancer, breast cancer, or bladder cancer.

Which statement by a client with a new diagnosis of hepatitis A alerts the nurse to the need for more education? 1 "I need to wash my hands after using the restroom." 2 "I may feel tired for about a week." 3 "I am going to have flare-ups for the rest of my life." 4 "My skin may turn yellow in the next few weeks

3 Hepatitis A does not have chronic flare-ups; therefore the nurse should realize the client needs teaching to correct this misinformation. A method of preventing the transmission of hepatitis A is handwashing, because the most common transmission route is fecal-oral. The prodromal phase of hepatitis is characterized by fatigue and malaise. The icteric phase begins about 1 month after exposure and is characterized by jaundice.

A nurse is asked what could cause chronic glomerulonephritis. How should the nurse reply? 1 Pneumonia 2 No urine protein 3 Hypercholesterolemia 4 Kidney stones that are passed

3 Hypercholesterolemia and proteinuria (protein in urine) have been associated with progressive glomerular and tubular injury. Pneumonia is a respiratory disease and is not associated with chronic glomerulonephritis. Kidney stones that are passed would not cause chronic glomerulonephritis. Absence of protein from urine is normal and would not lead to chronic glomerulonephritis.

A nurse is caring for a client who has end-stage kidney disease and needs dialysis. Which assessment finding does the nurse recognize as indicative of frost? 1 Serum potassium increased 2 Arterial blood pH decreased 3 Uremic skin residues 4 Pallor skin tone

3 Hyperparathyroidism and uremic skin residues (known as uremic frost) are associated with inflammation, irritation, and pruritus, with scratching, excoriation, and increased risk for infection. Increased serum potassium is hyperkalemia, not frost. Arterial blood pH decrease is metabolic acidosis, not frost. Pallor skin tone is associated with anemia, not frost.

A client develops pulmonary congestion soon after a myocardial infarction. Which pathophysiologic mechanism should the nurse consider when planning care for this client? 1 The client has inflammation of the peripheral veins. 2 The client has inflammation of the carotid veins. 3 The client has impaired left ventricular contractility. 4 The client has impaired right ventricular contractility.

3 If the coronary obstruction involves the perfusion to the left ventricle, pulmonary venous congestion ensues; if the right ventricle is ischemic, increases in systemic venous pressures occur. Inflammation of the peripheral veins or carotid veins do not cause pulmonary congestion after a myocardial infarction.

Which information indicates the nurse has an accurate understanding of the primary pathophysiologic process involved in a client with glomerulonephritis? 1 Trauma to the kidney 2 Obstructive pathology 3 Antigen-antibody complexes 4 Infection by Streptococcus species

3 Immune mechanisms are a major cause of injury for both primary and secondary forms of glomerulonephritis. The most common types of immune injury are (1) deposition of circulating antigen-antibody immune complexes into the glomerulus (type III hypersensitivity) and (2) antibodies reacting in situ against planted antigens within the glomerulus (type II hypersensitivity, cytotoxic). Trauma to the kidney and obstructive pathology are not responsible for glomerulonephritis. Streptococcus species are a cause of pyelonephritis.

A healthy individual stands up suddenly. What effect does the nurse expect when the baroreceptor reflex is activated? 1 Increased heart rate and vasodilation 2 Decreased heart rate and vasodilation 3 Increased heart rate and vasoconstriction 4 Decreased heart rate and vasoconstriction

3 In a healthy individual the baroreceptor reflex causes increased heart rate and vasoconstriction to maintain the blood pressure and thus adequate perfusion to the brain. Vasodilation would decrease the blood pressure. Decreased heart rate and vasodilation both would decrease the blood pressure. In orthostatic hypotension, the baroreceptor reflex is not necessarily absent or pathologically inverted—it may simply not be strong enough to fully compensate for the sudden postural change.

A nurse is teaching the staff about the cause of primary Addison disease. Which information should the nurse include in the teaching session? 1 A secreting adrenal cancer 2 Viral infection of the posterior pituitary 3 Autoimmune injury to the adrenal cortex 4 Bacterial infection of the adrenal medulla

3 In primary Addison disease, immune system attack on the adrenal cortex damages the gland. Secreting tumors cause hypersecretion of hormones; Addison disease causes a lack of secretions. Viral infection of the posterior pituitary would not cause Addison disease. Infection of the adrenal medulla primarily would affect the release of catecholamines.

A client has hypovolemic shock. Which early compensation mechanisms does the nurse expect? 1 Decreased heart rate 2 Decreased systemic vascular resistance 3 Release of epinephrine from adrenal glands 4 Release of antidiuretic hormone, causing water excretion

3 In the early stage of shock, the adrenal medulla is stimulated to release increased amounts of the catecholamines epinephrine and norepinephrine, which circulate to the heart and stimulate contractility and heart rate. Heart rate initially increases. Systemic vascular resistance is not decreased in early hypovolemic shock. Antidiuretic hormone is released from the posterior pituitary gland and causes water retention.

Which client should the nurse most closely monitor for the development of myxedema coma? 1 One with parathyroid adenoma 2 One with thyroid autoantibodies 3 One with untreated hypothyroidism 4 One with subclinical hyperthyroidism

3 Myxedema coma is caused by untreated hypothyroidism or occurs after overuse of narcotics/sedatives or an acute illness in hypothyroid individuals. Parathyroid adenoma will lead to hyperparathyroidism, not myxedema coma. Thyroid autoantibodies lead to hyperthyroidism, not hypothyroidism. Subclinical hyperthyroidism if not treated may lead to thyroid storm but not to myxedema coma.

hat is a similarity between a type I hypersensitivity reaction and a type IV hypersensitivity reaction? 1 Both develop immediately. 2 Both involve immunoglobulin G (IgG). 3 Neither involves the participation of a complement. 4 Neither involves macrophages as the principal effector cells

3 Neither type I nor type IV hypersensitivity reactions involve the participation of a complement. Type I reactions develop immediately, while type IV reactions are delayed, taking two to three days to develop. Both type II and type III hypersensitivity reactions involve immunoglobulin G (IgG), neither type I nor type IV reactions do. Type II and type IV reactions involve macrophages as the principal effector cells; type I and type III reactions do not.

A diabetic child with 4.0 grams of protein in the urine each day is experiencing edema and vitamin D deficiency. Which is the most likely diagnosis the nurse will observe written on the chart? 1 Acute renal failure 2 Nephritic syndrome 3 Nephrotic syndrome 4 Rapidly progressive glomerulonephritis

3 Nephrotic syndrome is characterized by excretion of 3.5 grams or more of protein in the urine per day due to glomerular injury. Acute renal failure presents with oliguria and a reduction in glomerular filtration rate and blood urea nitrogen. Nephritic syndrome is characterized by blood in the urine with red and white cell casts and varying degrees of protein. Rapidly progressive glomerulonephritis usually affects adults in their 50s and 60s and presents with hematuria; nephrotic syndrome is more common in children.

A nurse is assessing a client with suspected nephrotic syndrome. Which finding is consistent with nephrotic syndrome? 1 Hyperglycemia 2 Decreased serum triglycerides 3 Decreased serum albumin 4 Hematuria with red blood cell cast

3 Nephrotic syndrome is diagnosed when the protein level in a 24-hour urine collection is greater than 3.5 g. Serum albumin level decreases (to less than 3 g/dl), and concentrations of serum cholesterol, phospholipids, and triglycerides increase. Hyperglycemia does not occur as blood glucose levels are not affected. Hematuria with red blood cell casts is an indication of nephritic syndrome, not nephrotic syndrome.

A client has a disorder characterized by a neurologic lesion that affects bladder control. The nurse will report to the oncoming shift that the client has which disorder? 1 Cystitis 2 Bladder prolapse 3 Neurogenic bladder 4 Detrusor overactivity

3 Neurogenic bladder is a condition in which neurologic lesions cause bladder dysfunction. Neurologic lesions are not characteristic of cystitis, although stasis of urine due to neurologic lesions may increase the risk of cystitis. Neurologic lesions are not characteristic of bladder prolapse (in which the bladder falls from its normal anatomic position) or of detrusor overactivity (overactive bladder syndrome).

A nurse is caring for an older client that has bone spurs with damage to the articular cartilage of the synovial joints. The nurse is caring for which client? 1 One with osteoporosis 2 One with osteomyelitis 3 One with osteoarthritis 4 One with osteomalacia

3 Osteoarthritis is a common, age-related disorder of synovial joints. A client with osteoarthritis features damage of the articular cartilage, bone spurs, and subchondral bone changes. In osteoporosis, old bone is being resorbed faster than new bone is being made, causing the bones to lose density, becoming thinner and more porous. Osteomyelitis is a bone infection most often caused by bacteria. Osteomalacia is a metabolic bone disease characterized by inadequate bone mineralization.

nurse is caring for an older client that has bone spurs with damage to the articular cartilage of the synovial joints. The nurse is caring for which client? 1 One with osteoporosis 2 One with osteomyelitis 3 One with osteoarthritis 4 One with osteomalacia

3 Osteoarthritis is a common, age-related disorder of synovial joints. A client with osteoarthritis features damage of the articular cartilage, bone spurs, and subchondral bone changes. In osteoporosis, old bone is being resorbed faster than new bone is being made, causing the bones to lose density, becoming thinner and more porous. Osteomyelitis is a bone infection most often caused by bacteria. Osteomalacia is a metabolic bone disease characterized by inadequate bone mineralization.

Which term will the nurse use in report to describe a client who has skeletal pain with significant bone demineralization from vitamin D deficiency? 1 Osteopenia 2 Osteoporosis 3 Osteomalacia 4 Osteomyelitis

3 Osteomalacia is inadequate mineralization of bone tissue, most commonly caused by vitamin D deficiency, and it usually causes skeletal pain. Although low vitamin D can be a risk factor for osteoporosis, the situation described here, with bone pain, corresponds to osteomalacia, not osteoporosis. Osteoporosis leads to porous bone and loss of density. Osteopenia is bone density between 1.5 and 2.5 standard deviations below normal. Osteoporosis is a score of 2.5 or more standard deviations below normal bone density. Osteomyelitis is a bone infection with inflammation.

A nurse is caring for a client with osteoporosis. Which type of fracture does the nurse monitor for in this client? 1 Osteomalacia 2 Osteomyelitis 3 Insufficiency 4 Rhabdomyolysis

3 Osteoporosis weakens the bone structure and increases the risk for insufficiency fractures. Osteomalacia is a metabolic disease characterized by soft osteoid bone instead of rigid bone. Osteomyelitis is a bone infection with inflammation. Rhabdomyolysis is the rapid breakdown of muscle that causes the release of intracellular contents, including the protein pigment myoglobin, into the extracellular space and bloodstream.

nurse is asked by a coworker what causes overactive bladder syndrome. How should the nurse respond? 1 It is a result of increased serum calcium. 2 It is a result of a loss of sensation of bladder fullness. 3 It is a result of involuntary detrusor muscle contractions. 4 It is a result of failure of the external sphincter to fully open

3 Overactive bladder syndrome is a syndrome of detrusor overactivity characterized by urgency associated with spontaneous or provoked involuntary detrusor contractions during the bladder filling phase. The contractions occur during bladder filling, not from a loss of sensation of bladder fullness. There is coordination between the contracting bladder and the external sphincter, but it is the detrusor that is too weak, not the external sphincter. Increased serum calcium can lead to stones, not to overactive bladder syndrome.

A nurse is caring for a client with postrenal acute kidney injury. The nurse is caring for which client? 1 A client with blood loss 2 A client with hypovolemia 3 A client with an obstruction 4 A client with nephron damage

3 Postrenal failure is due to urinary tract obstruction. Nephron damage is characteristic of intrarenal failure. Prerenal failure is usually caused by hypoperfusion of the kidney owing to decreased systemic blood volume (hypovolemia), usually from blood loss.

A nurse is teaching a client about the cause of Prinzmetal angina. Which information from the client indicates teaching was successful? 1 It is from a deep vein clot. 2 It is from a blood clot in a lung artery. 3 It is from a spasm of a heart artery. 4 It is from low oxygen due to respiratory disease.

3 Prinzmetal angina is a special type of chest pain that occurs transiently at rest, most likely owing to vasospasm of coronary arteries (that is, in the simpler terms that the client used, a spasm of a heart artery). Deep vein thrombosis occurs in the limbs and does not affect coronary artery blood flow. Blood clots in a coronary artery cause acute coronary syndrome rather than Prinzmetal angina. Respiratory disease does not cause Prinzmetal angina.

A nurse is teaching a client about the cause of Prinzmetal angina. Which information from the client indicates teaching was successful? 1 It is from a deep vein clot. 2 It is from a blood clot in a lung artery. 3 It is from a spasm of a heart artery. 4 It is from low oxygen due to respiratory disease.

3 Prinzmetal angina is a special type of chest pain that occurs transiently at rest, most likely owing to vasospasm of coronary arteries (that is, in the simpler terms that the client used, a spasm of a heart artery). Deep vein thrombosis occurs in the limbs and does not affect coronary artery blood flow. Blood clots in a coronary artery cause acute coronary syndrome rather than Prinzmetal angina. Respiratory disease does not cause Prinzmetal angina.

A client has chronic kidney failure and develops Kussmaul respirations. The nurse suspects the client is experiencing which condition? 1 Anemia 2 Dehydration 3 Metabolic acidosis 4 Chronic hyperlipidemia

3 Pulmonary edema and metabolic acidosis can cause Kussmaul respirations. Fluid overload (not dehydration), leading to pulmonary edema, causes Kussmaul respirations. Anemia does not cause Kussmaul respirations but does result from chronic kidney failure due to reduced erythropoietin secretion. Chronic hyperlipidemia may contribute to chronic renal disease, but it does not cause Kussmaul respirations.

lient is receiving radiation and asks the nurse how radiation works. How should the nurse respond? 1 Radiation blocks the normal growth pathways in cells. 2 Radiation can cause reversible changes in normal tissues. 3 Radiation is used to kill cancer cells while minimizing damage to normal structures. 4 Radiation requires poor local delivery of oxygen to effectively kill cell

3 Radiation therapy is used to kill cancer cells while minimizing damage to normal structures. Effective cell killing using radiation also requires good local delivery of oxygen. Radiation produces slow changes in most cancers and irreversible changes in normal tissues as well.

Which intervention should the nurse implement for a client with rhabdomyolysis? 1 Restrict fluids as prescribed 2 Increase myoglobin levels as prescribed 3 Administer rapid intravenous hydration as prescribed 4 Encourage excessive exercise regimen as prescribed

3 Rapid intravenous hydration maintains adequate kidney flow. Maintaining adequate urinary flow and prevention of kidney failure are goals of treatment. Fluids would be administered, not restricted. Myoglobin levels and exercise would be decreased, not increased.

A client has rheumatoid arthritis. Which pathophysiologic process that causes joint inflammation should the nurse consider when planning care? 1 Trauma 2 Bacterial infection 3 Autoimmune injury 4 Congenital hypermobility

3 Rheumatoid arthritis is a chronic, systemic, inflammatory autoimmune disease distinguished by joint swelling and tenderness and by destruction of synovial joints, leading to disability and premature death. Rheumatoid arthritis is not caused by trauma, infection, or congenital hypermobility.

client has schistosomiasis. The nurse is caring for which client? 1 One with varices 2 One with gallstones 3 One with a parasitic infection 4 One with protein-energy malnutritio

3 Schistosomiasis is a parasitic infection, resulting in portal hypertension. Varices are distended, tortuous, collateral veins in the lower esophagus, stomach, abdominal wall, or rectum. Kwashiorkor is protein-energy malnutrition in the presence of carbohydrate intake. Gallstone formation is termed cholelithiasis. `

A client has ascites but no signs of liver disease. How should the nurse interpret this finding? 1 The tests are inaccurate. 2 The client is dehydrated. 3 The client may have low albumin. 4 The diastolic blood pressure is elevated

3 Several factors contribute to the development of ascites, including decreased synthesis of albumin by the liver and fluid retention, not dehydration. The tests are not inaccurate; ascites can develop from other reasons than just liver disease. Elevated diastolic blood pressure does not lead to ascites, but it does lead to hypertension.

Upon assessment, the nurse notices that a client has one eye that deviates from the other when looking at an object. What condition did the nurse observe? 1 Nystagmus 2 Amblyopia 3 Strabismus 4 Presbyopia

3 Strabismus is when one eye deviates from the other when looking at an object. It is due to a weak (hypotonic) muscle in that eye. Amblyopia is reduced or dimmed vision caused by cerebral blockage of the visual stimuli. Presbyopia is loss of accommodative capacity seen with advancing age. Nystagmus is involuntary unilateral or bilateral rhythmic movement of the eyes.

A client has displacement of two bones in which the articular surfaces partially lose contact with each other. Which term should the nurse use in report to describe this finding? 1 Strain 2 Sublimation 3 Subluxation 4 Subjugation

3 Subluxation is partial dislocation of a joint. Tearing or stretching of a muscle or tendon is commonly known as a strain. Sublimation and subjugation are not terms used for partial dislocation of a joint.

Early diagnosing of ankylosing spondylitis in a client is best aided by which form of testing? 1 Radiographic imaging 2 Computed tomography (CT) 3 Magnetic resonance imaging (MRI) 4 Positron emission tomography (PET

3 The best method to diagnose ankylosing spondylitis is MRI. It shows evidence 7 years earlier than radiography. Radiography is the traditional method; CT and PET scans are not usually used.

A client with a kidney obstruction developed compensatory hypertrophy and hyperfunction. What physiologic process occurred? 1 Release of somatomedin to break down renal calculi in the affected kidney 2 Sympathetic nervous system responses to compensate for fluid volume depletion 3 Increase in glomerular and tubular size to increase filtration of the unaffected kidney 4 Relief of obstruction followed by a brief period of diuresi

3 The body is able to partially counteract the negative consequences of unilateral obstruction by processes called compensatory hypertrophy and hyperfunction. These processes cause the unobstructed kidney to increase the size of individual glomeruli and tubules but not the total number of functioning nephrons. Relief of obstruction followed by a brief period of diuresis is commonly called postobstructive diuresis, not compensatory hypertrophy and hyperfunction. Sympathetic nervous system responses can compensate for fluid volume depletion by release of hormones and neurotransmitters, but such release does not cause compensatory hypertrophy and hyperfunction. Somatomedins cause the obligatory growth that occurs in compensatory hypertrophy, not the breakdown of renal calculi.

The nurse receives a report from a licensed practical nurse about care provided to clients on a neurologic unit. It is most important for the nurse to follow up on which statement first? 1 "The 84-year-old client with Alzheimer disease is depressed and anxious." 2 "The 45-year-old client with Huntington disease has writhing movements of both hands." 3 "The 72-year-old client with Parkinson disease complains of dizziness when standing up." 4 "The 15-year-old client with Tourette syndrome is repeating words over and over again."

3 The client with Parkinson disease has orthostatic hypotension (dizziness when standing) and is at risk for falls. The clinical manifestations exhibited by the other clients are expected; these clinical manifestations are not priority client safety concerns.

A client has acute tubular necrosis. A coworker says, "I've been taking care of this client for over 2 weeks. After my days off, I come back and find that the client has mild diuresis. I'm really worried by this change, but no one else is. What's going on?" How should the nurse respond? 1 It's good you noticed this change. It could be something serious that was not caught while you were away. 2 You had better measure the blood sugar. The client may have developed diabetes, and what you see may be osmotic diuresis from glucose in the urine. 3 The renal tubules are recovering, so the client is making more urine, but the kidney is not able to concentrate urine well because it is not fully recovered. 4 Given that the client was oliguric for so long, this is probably rebound polyuria. As long as the blood pressure is stable and there is no hypovolemia, you should not worry.

3 The coworker should have recognized the diuretic phase of acute tubular necrosis. The diuresis is part of acute tubular necrosis and is not a problem in itself; it is not an osmotic diuresis such as occurs in diabetes mellitus. The client does not have rebound polyuria.

Which clinical manifestations would the nurse expect to find during assessment of the client with extracellular fluid volume excess? 1 Tachycardia 2 Postural hypotension 3 Decreased hematocrit 4 Increased serum sodium concentratio

3 The hematocrit is reduced from the dilutional effect of water excess. Tachycardia, increased serum sodium concentration, and postural hypotension are signs of extracellular fluid volume deficit.

A client has rhabdomyosarcoma. Which structure in the body is affected most? 1 Skin 2 Bone 3 Muscle 4 Cartilage

3 The malignant tumor of striated muscle is called rhabdomyosarcoma. Rhabdomyosarcoma is not a tumor in the skin. Osteogenic tumor affects the bone. Chondrogenic cancer affects the cartilage and bone.

Which client is at most risk for silent myocardial ischemia? 1 A woman with emphysema 2 A man with hypertension 3 A woman with diabetes mellitus 4 A man with Alzheimer disease

3 The most common cause of autonomic dysfunction leading to silent ischemia is diabetes mellitus. Silent myocardial ischemia is more common in women. Emphysema and other chronic lung diseases can exacerbate myocardial ischemia but are not specific risk factors for silent myocardial ischemia. Although hypertension is a risk factor for atherosclerosis, it is not specifically associated with silent myocardial ischemia. Alzheimer disease does not necessarily affect perception of cardiac pain.

Which information from the client indicates teaching by the nurse was successful about glomerular disorders? 1 A type of glomerular disorder is pyelonephritis. 2 A type of glomerular disorder is interstitial cystitis. 3 A type of glomerular disorder is nephrotic syndrome. 4 A type of glomerular disorder is obstructive uropathy

3 The pathophysiology of nephrotic syndrome occurs at the glomerular membrane. Pyelonephritis generally does not affect the glomerulus. Interstitial cystitis is a bladder disorder. Obstructive uropathy affects the upper and lower urinary tract but does not directly involve the glomerulus.

Which client will the nurse monitor most closely for autonomic dysreflexia? 1 A client before spinal shock 2 A client during spinal shock 3 A client after spinal shock resolves 4 A client who did not develop spinal shock

3 The possibility of developing autonomic dysreflexia occurs after spinal shock has resolved. Autonomic dysreflexia usually does not occur before spinal shock or in someone who did not develop spinal shock. Autonomic function is not present below the level of injury during spinal shock.

A client is in the final stage of gout, characterized by crystalline deposits in cartilage, synovial membranes, and soft tissue. Which diagnosis will the nurse observe documented on the chart? 1 Gouty arthritis 2 Complicated gout 3 Tophaceous gout 4 Asymptomatic hyperuricemia

33 The final chronic stage of gout is called tophaceous gout. Complicated gout is not the final stage. Acute gouty arthritis is the second stage. Asymptomatic hyperuricemia is the first stage of gout.

A client is being transferred from another medical facility with a diagnosis of acute myocardial infarction (AMI). Which assessment findings will the nurse typically observe with the presence of an AMI? Select all that apply. 1 High fever 2 Cough 3 Nausea and vomiting 4 Crushing chest pain 5 Cool, clammy skin

345 A client may experience crushing chest pressure or pain, nausea, vomiting, and cool and clammy skin. A high fever and cough are not considered signs of an AMI but occur with infections.

Which assessment findings are common in a client with erosive reflux esophagitis? Select all that apply. 1 Ascites 2 Anorexia 3 Heartburn 4 Dysphagia 5 Chronic cough

345 Heartburn, dysphagia, and chronic dry cough are all common manifestations of erosive reflux esophagitis. Ascites and anorexia are not common manifestations of gastroesophageal reflux disease but can occur in liver disease.

A client was found at home in the fetal position complaining of a severe, sudden onset headache. Assessments in the emergency room reveal confusion, disorientation, and one dilated pupil. A computed tomography (CT) scan of the brain reveals a severe cerebral aneurysm. The client has cyclic respiratory rate and depth changes, hiccups, a full and bounding pulse, and a pulse pressure greater than 80. The primary healthcare provider talks to the family about the stage and treatment option. For follow-up teaching, the nurse should reinforce which stage and treatment option? 1 Late Stage 1 of intracranial hypertension; primary healthcare provider to insert pressure monitor and continue to monitor for increasing pressures 2 Early Stage 2 of intracranial hypertension; schedule surgery for the morning and assure family that the surgery is precautionary 3 Late Stage 3/Early Stage 4 of intracranial hypertension; beginning decompensation, immediate surgery required 4 Very Late Stage 4 of intracranial hypertension; decompensation (herniation), impending death, surgery futile

3 The signs and symptoms presented are indicative of late Stage 3 and early Stage 4 of intracranial hypertension: beginning decompensation, and immediate surgery usually is required. Late Stage 3 and Early Stage 4 indicate one dilated pupil and Cheyne-Stokes breathing. Wide pulse pressures and a full and bounding pulse also are noted, indicating severe increases in intracranial pressure and potential herniation. Stage 1 is when the client is awake and alert with equal and reactive pupils. Stage 2 has episodes of confusion, restlessness, and lethargy, and pupils remain equal and reactive with normal breathing. Late Stage 4 exhibits a deep coma, bilateral dilation and fixation of the pupils, ataxic breathing, slightly irregular pulse, and narrowing pulse pressures with impending death.

A client experiences chronic intermittent pain in the epigastric area when the stomach is empty and in the middle of the night. Which diagnosis will the nurse most likely see documented on the chart? 1 Gastric ulcer 2 Acute gastritis 3 Duodenal ulcer 4 Chronic gastritis

3 These symptoms support a diagnosis of a duodenal ulcer. Gastric ulcer symptoms include epigastric pain that may occur immediately after eating. Acute gastritis often has vague abdominal discomfort, epigastric tenderness, and bleeding. Chronic gastritis has symptoms that include anorexia, fullness, nausea, vomiting, and pain.

Which information indicates the nurse has a correct understanding of the pathophysiologic process or processes that cause tissue damage in a client with acute pancreatitis? 1 Insulin toxicity and beta cell hyperplasia 2 Autoimmune destruction of the pancreas 3 Inappropriate activation of pancreatic enzymes 4 Hydrochloric acid reflux into the pancreatic duc

3 Tissue damage in acute pancreatitis is caused by inappropriate activation of pancreatic enzymes in the pancreas and the resulting autodigestion of the gland. Insulin toxicity and beta cell hyperplasia do not cause the tissue damage in acute pancreatitis. Autoimmune destruction of the pancreas does not cause the tissue damage in acute pancreatitis. Hydrochloric acid reflux into the pancreatic duct is not associated with acute pancreatitis.

A young adult client with acute pericarditis presents with severe chest pain that worsens with respiratory movements and with lying down. Which is the most appropriate intervention for this condition? 1 Place on total bed rest 2 Give beta-blockers 3 Administer anti-inflammatory drugs 4 Prepare for percutaneous coronary intervention

3 Treatment for uncomplicated acute pericarditis consists of relieving symptoms and includes administration of anti-inflammatory agents, such as salicylates and nonsteroidal anti-inflammatory drugs. Total bed rest, beta-blocker administration, and percutaneous coronary intervention are appropriate for an acute myocardial infarction.

What alteration should the nurse expect to find in a person who has untreated Cushing syndrome? 1 Pale skin 2 Weight loss 3 Truncal obesity 4 Pulmonary edema

3 Truncal obesity due to central fat deposition is a common physical manifestation of untreated Cushing syndrome. Pale skin is not a common manifestation of Cushing syndrome. Elevated cortisol generally leads to weight gain, not weight loss. Pulmonary edema is not a common manifestation of Cushing syndrome.

Which information from the client indicates teaching by the nurse was successful about renal calculi? 1 Calcium inhibits stone formation. 2 Alkaline urine promotes the formation of uric acid stones. 3 Urinary stasis increases the likelihood of renal stone formation. 4 Stones that are 2 to 3 mm have no chance of passing through the urinary tract.

3 Urinary stasis increases the risk of stone formation by preventing prompt flushing of crystals from the urinary system. Acidic urine greatly increases the risk for uric acid stones. Calcium is the most common type of renal stone. Stones < 5 mm have a 50% chance of passing. Stones > 1 cm have little chance of passing.

A nurse is talking about the bacteria that resist flushing during normal micturition because of type 1 fimbriae. Which organisms is the nurse describing? 1 Proteus 2 Klebsiella 3 Escherichia coli 4 Staphylococcus

3 Uropathic strains of E coli have type 1 fimbriae that bind to latex catheters and receptors on uroepithelium. They resist flushing during normal micturition. Proteus, Klebsiella, and Staphylococcus can lead to urinary tract infection but do not contain the type 1 fimbriae that E coli has.

A client has esophageal varices. While planning care the nurse should focus interventions on which cause of esophageal villustarices? 1 Biliary obstruction 2 Systemic hypotension 3 Portal hypertension 4 Weakness of esophageal walls

3 Vomiting of blood (hematemesis) from bleeding esophageal varices is the most common clinical manifestation of portal hypertension. Biliary obstruction could result in jaundice. Systemic hypotension would not result in esophageal varices. Esophageal vessels, not the esophageal wall itself, are affected.

A client has an X-linked recessive immunodeficiency disease characterized by decreased immunoglobulin M (IgM) production. Which medical diagnosis will the nurse observe written in the chart? 1 DiGeorge syndrome 2 Selective IgA deficiency 3 Wiskott-Aldrich syndrome 4 Severe combined immunodeficiency disease (SCID

3 Wiskott-Aldrich syndrome is an X-linked recessive disease characterized by decreased IgM production. Individuals with this disorder are especially susceptible to encapsulated bacterial infections. DiGeorge syndrome is characterized by decreased T lymphocyte number and function, rather than decreased IgM. Selective IgA deficiency is characterized by a lack of IgA, rather than IgM. SCID is an autosomal recessive defect that causes multiple immune defects including IgM and IgA.

Which clients have increased serum calcium and therefore an increased risk of renal stone formation? Select all that apply. 1 A client with acidic urine 2 A client with hypocalcemia 3 A client with hyperparathyroidism 4 A client with prolonged immobilization 5 A client with increased intestinal absorption of calcium

345 Hyperparathyroidism and increased intestinal absorption of calcium increase serum calcium levels, and immobilization causes bone demineralization, increasing serum levels and increasing the risk of renal stone formation. Alkaline urine, not acidic, is associated with calcium stones. Hypocalcemia is decreased calcium; increased calcium is associated with increased risk of renal stones.

A client had a myocardial infarction. Which laboratory results should the nurse monitor? Select all that apply. 1 Aspartate aminotransferase (AST) 2 Alanine aminotransferase (ALT) 3 Lactate dehydrogenase (LDH) 4 Creatine kinase-myocardial bound (CK-MB) 5 Troponin

345 LDH, CK-MB, and troponin I are all indicators of myocardial damage and will rise with a myocardial infarction. Although LDH is measured in liver function, it also has a purpose for myocardial infarction. AST and ALT are liver enzymes that indicate damage to the liver cells and are not monitored for a myocardial infarction.

Which information should the nurse consider when planning care for a client with a hiatal hernia? Select all that apply. 1 Sliding hernia is triggered by standing. 2 The most common type is paraesophageal. 3 Avoiding the recumbent position after eating can help. 4 Paraesophageal hernia may lead to gastritis and ulcer formation. 5 Paraesophageal hernia is herniation of the greater curvature of the stomach.

345 Paraesophageal hiatal hernia is caused by herniation of the greater curvature of the stomach through a second opening in the diaphragm. Symptoms caused by congestion of mucosal blood flow include gastritis and ulcer formation. The individual can diminish reflux by eating small, frequent meals and avoiding the recumbent position after eating. Sliding hiatal hernia is the most common type, and it reverses with standing. It moves into the thoracic cavity with sitting, bending, tight clothing, ascites, and obesity.

A client has obstructive uropathy. Which factors should the nurse consider to help determine the severity of the obstruction? Select all that apply. 1 Smell of the urine 2 Color of the urine 3 Duration of the blockage 4 Location of the blockage 5 Degree of completeness of the blockage

345 The factors that determine severity for obstructive uropathy are the location of the blockage, the degree of completeness, the duration, the involvement of one or both upper urinary tracts, and the cause of the lesion. The smell and color of the urine do not determine the severity of the obstruction.

A nurse is performing screenings at a health fair. Which findings make individuals predisposed to heart failure? Select all that apply. 1 Anemia 2 Young age 3 Hypertension 4 Obesity 5 Diabetes 6 Ischemic heart disease

3456 Risk factors of heart failure are ischemic heart disease and hypertension, followed by other factors such as cardiomyopathies, obesity, diabetes, and valvular heart disease. Anemia and young age are not risk factors for heart failure.

Which clients are exhibiting manifestations of a pyramidal motor syndrome? Select all that apply. 1 A client with a resting tremor 2 A client with an unsteady gait 3 A client with hyperactive reflexes 4 A client with a positive Babinski sign 5 A client with clasp-knife phenomeno

345Pyramidal motor syndrome manifestations include hyperactive reflexes, spasticity in muscles (e.g., clasp-knife phenomenon), and a positive Babinski sign. Resting tremors and an unsteady gait are manifestations of extrapyramidal motor syndrome.

A client has a locally invasive malignant bladder tumor. Which treatment options should the nurse consider while planning care for this client? Select all that apply. 1 Endoscopy 2 Nephrectomy 3 Radical cystectomy 4 Adjuvant chemotherapy 5 Transurethral resection

34Radical cystectomy with urinary diversion and adjuvant chemotherapy is required for locally invasive tumors. Endoscopy is a diagnostic procedure (or a component of treatment in endoscopic surgery); it is not a treatment by itself. Nephrectomy is the removal of a kidney and is not indicated for bladder tumors. Transurethral resection or laser ablation, combined with intravesical chemotherapy or immunotherapy, is effective for superficial tumors.

n which order should the nurse list the following information to indicate a correct understanding of the sequence of bone healing? 1. Remodeling 2. Callus formation 3. Hematoma formation 4. Invasion of osteoblasts 5. Organization of hematoma into a fibrous network

35421 Bone healing occurs in the following sequence: A, Bleeding at broken ends of the bone with subsequent hematoma formation. B, Organization of hematoma into a fibrous network. C, Invasion of osteoblasts, lengthening of collagen strands, and deposition of calcium. D, Callus formation; new bone is built while osteoclasts destroy dead bone. E, Remodeling is accomplished while excess callus is reabsorbed and trabecular bone is deposited.

A nurse is preparing to teach the staff about diabetic ketoacidosis (DKA). Which information should the nurse include? Select all that apply. 1 An increase in insulin is observed. 2 Condition peaks in the older adult. 3 It is more common in type 1 diabetes. 4 Assessment shows a decrease in catecholamines. 5 Increased glyconeogenesis and ketogenesis are present

35DKA is much more common in type 1 diabetes because insulin is more deficient. Profound insulin deficiency results in accelerated gluconeogenesis and ketogenesis. DKA can develop when an increase in insulin counterregulatory hormones is present, including catecholamines, cortisol, glucagon, and growth hormone. Old age is not a risk factor.

Which information from the staff indicates teaching by the nurse was successful for saccular aneurysms? Select all that apply. 1 They grow rapidly. 2 They occur commonly in childhood. 3 They may be round, broad-based, or cylindrical. 4 Saccular aneurysm is another name for an arteriovenous malformation. 5 They can be due to a combination of congenital and degenerative changes

35Saccular (berry) aneurysms can be due to a combination of congenital and degenerative changes. Such an aneurysm may be round with a narrow stalk connecting it to the parent artery, broad-based without a stalk, or cylindrical. They grow slowly over time and are rare in childhood. An arteriovenous malformation is a tangled mass of dilated blood vessels creating abnormal channels between the arterial and venous systems; it is not a saccular aneurysm.

What is the nurse's best response to a client who is asking about autoantibodies and Graves disease? 1 That is why you take the thyroid hormone replacement—because your thyroid gland is damaged. 2 That is why your thyroid has shut down—because the autoantibodies attacked the gland. 3 The autoantibodies bind to the thyroid cells and stimulate them to secrete thyroid hormones. 4 The autoimmune process acts on the anterior pituitary, which normally secretes a hormone.

3Autoantibodies in Graves disease stimulate the release of thyroid hormones by binding to and activating thyroid-stimulating hormone (TSH) receptors, increasing the release of thyroid hormones, especially T3. People who have Graves disease do not need thyroid hormone replacement, because the thyroid is overactive; it has not shut down. The autoimmune process does not act on the anterior pituitary in Graves disease, but rather on the thyroid.

A client with type 2 diabetes complains of blurred vision. Which mechanism most likely caused this client's visual changes? 1 Exophthalmos 2 Myxedema coma 3 Sorbitol accumulation in the lens 4 Overhydration of lens and aqueous humo

3Blurring of vision is a consequence of hyperglycemia and sorbitol accumulation in the lens. Exophthalmos is a clinical manifestation of Graves disease, not type 2 diabetes. Myxedema coma can occur with hypothyroidism, not type 2 diabetes. Dehydration of the lens, aqueous humor, and vitreous humor also may reduce visual acuity.

hich client does the nurse most closely monitor for a chronic subdural hematoma? 1 An infant who fell 2 A teenager who fell 3 An elderly client who fell 4 A middle-aged client who fel

3Chronic subdural hematomas (commonly found in elderly persons and persons who abuse alcohol and have some degree of brain atrophy with a subsequent increase in extradural space) develop over weeks to months. An infant, teenager, and middle-aged adult are not as predisposed to subdural hematomas as the older adult.

A client has Guillain-Barré syndrome. Which assessment finding will the nurse typically observe? 1 Symptoms beginning in the arms and face 2 Increased intestinal motility 3 Ascending motor paralysis 4 Seizures

3Clients often experience ascending motor paralysis. It usually begins in the legs, spreading often to the arms and face. Increased intestinal motility may occur in a cholinergic crisis or intestinal infections. Seizures occur in epilepsy.

A patient who sustained a motor vehicle accident has a vertebral injury. The nurse determines that the patient may have cord swelling in the cervical region based on which assessment finding? 1 Polycythemia 2 Hypertension 3 Breathing difficulty 4 Abdominal distenti

3Cord swelling in the cervical region damages the phrenic nerves that are located at C3 to C5. This results in impaired diaphragm functioning which may result in breathing difficulty. Spinal cord edema causes disintegration of red blood cells, not increased production (polycythemia). Cord swelling is associated with inflammation and loss of vasomotor tone, leading to low and unstable blood pressure. Cord swelling in the cervical region does not cause abdominal distention.

A client had a closed-head injury and is now having polyuria with a low urine specific gravity. Which complication does the nurse suspect the client is experiencing? 1 Kidney damage 2 Diabetes mellitus 3 Diabetes insipidus 4 Psychogenic polydipsi

3Diabetes insipidus is a well-recognized complication of closed-head injury. Insufficient antidiuretic hormone (ADH) release makes the kidneys unable to concentrate urine, resulting in the large quantity of dilute urine typical of diabetes insipidus. Kidney damage would not likely result from closed-head injury, nor would it result in a large diuresis of very dilute urine. Diabetes mellitus is related to insufficient insulin, not to ADH dysfunction. Psychogenic polydipsia may be confused with diabetes insipidus. It is caused by the chronic ingestion of extremely large quantities of fluid that wash out the renal medullary concentration gradient, which results in a partial resistance to ADH.

A client has diverticulosis. Which pathophysiologic process should the nurse consider when planning care? 1 Polyps or growths of the colon wall mucosa and submucosa 2 Inflammation of small pockets in the colon wall mucosa and submucosa 3 Acquired herniations of the colon wall mucosa and submucosa 4 Congenital outpouchings of the colon wall mucosa and submucosa

3Diverticula are herniations or saclike outpouchings of the mucosa and submucosa through the muscle layers, usually in the wall of the sigmoid colon. Diverticulosis is characterized not by growths but rather by herniations or pouches; cancer usually produces polyps or growths. Diverticulitis is an inflammation of the herniations. Diverticulosis is not a congenital disorder.

A nurse is caring for a client with encephalitis. Which information should the nurse remember when planning care for this client? 1 It is an afebrile illness. 2 It is treated with chemotherapy. 3 It is usually a result of arthropod borne viruses. 4 It is caused exclusively by herpes simplex II

3Encephalitis is an acute febrile illness that is caused by a virus, often arthropod borne (tick/mosquito borne) viruses or herpes simplex I, not herpes simplex II. Some clients are treated with antiviral agents and steroids, not chemotherapy.

A client exposed to herpes simplex virus has been exhibiting symptoms of an acute febrile illness with nervous system involvement. Which term will the nurse use in report to describe this condition? 1 Bacteremia 2 Astrocytoma 3 Encephalitis 4 Brain absces

3Encephalitis is an acute febrile illness, usually of viral origin, with nervous system involvement. The most common forms are caused by arthropod-borne (mosquito-borne) viruses and herpes simplex type 1. Bacteremia is bacteria in the blood. Astrocytoma is a type of glioma or primary brain (intracerebral) tumor. Brain abscesses act like space-occupying lesions and are often localized collections of pus.

A client has a tumor that is most commonly found deep within the frontal lobe of the brain. What medical diagnosis will the nurse observe written on the chart? 1 Meningioma 2 Neurilemmoma 3 Oligodendroglioma 4 Glioblastoma multiform

3Oligodendrogliomas occur most commonly in frontal lobes deep in white matter. Meningiomas may be located in the sylvian fissure region, the superior parasagittal surface of the frontal and parietal lobes, the olfactory groove, the wing of the sphenoid bone, the superior surface of the cerebellum, the cerebellopontine angle, or the spinal cord. Glioblastoma multiforme occurs predominantly in cerebral hemispheres. Neurilemmomas are slow-growing tumors in the cranial nerves, mainly cranial nerve VIII.

A client developed a subdural hematoma after being hit in the head by a golf ball during a tournament and will be taken to surgery. Which information is best for answering the parents' questions about surgery? 1 The accumulation of blood under the dura mater is increasing the intracranial pressure. It can damage the brain unless surgery is done to relieve the pressure. 2 The bleeding deep within the brain is squeezing the brain and causing it to function poorly. It can damage the brain unless surgery is done to relieve the pressure. 3 The bleeding above the surface of the brain is squeezing the brain and causing it to function poorly. It can damage the brain unless surgery is done to remove the blood clot and relieve the pressure. 4 The accumulation of blood in the fluid that surrounds the brain is increasing the pressure and compressing the brain. It can damage the brain unless surgery is done to relieve the pressure.

3Explaining that the bleeding is increasing pressure and squeezing the brain accurately explains the necessity for surgery in terms the parents can understand. Using terms such as dura mater and intracranial pressure, although accurate, is inappropriate because they are not easily understood. The bleeding is not occurring deep within the brain, nor is it mixing with cerebrospinal fluid, as would be seen in intracerebral or subarachnoid hemorrhages, respectively.

A client presents with a wild, flinging movement of the left arm and left leg. What term would the nurse use to document this clinical manifestation? 1 Akinesia 2 Akathisia 3 Hemiballism 4 Hemiparesis

3Hemiballism is a disorder of the proximal muscle postural fixation with wild flinging movements of the limbs on one side of the body. Akinesia is a decrease in voluntary movements. Akathisia is a mild compusion to move. Hemiparesis is weakness on one side of the body.

Which pathophysiologic mechanism should the nurse consider when a laboratory result indicates a client with nephrotic syndrome has hyperlipidemia? 1 Triglycerides are decreased. 2 Lipids are not excreted in the urine. 3 Liver synthesizes lipoproteins. 4 Loss of immunoglobulins occurs

3Hyperlipidemia in nephrotic syndrome is caused by increased hepatic synthesis of lipoproteins. Triglycerides are increased, not decreased. People who have nephrotic syndrome have both hyperlipidemia and lipiduria (loss of fat in the urine). Loss of immunoglobulins may increase susceptibility to infections; it does not affect lipids.

A client has myasthenia gravis. Which pathophysiologic process should the nurse consider when planning care for this client? 1 Injury of the spinal roots caused by compression or direct trauma 2 Autoimmune inflammatory response, resulting in axonal demyelination 3 Immunoglobulin G (IgG) antibody production against acetylcholine receptors 4 Lower and upper motor neuron degeneratio

3IgG antibodies are produced against the acetylcholine receptors in clients with myasthenia gravis. Radiculopathies are injuries to spinal roots caused by compression, inflammation, or direct trauma. Amyotrophic lateral sclerosis is lower and upper motor neuron degeneration, although without inflammation. Guillain-Barré syndrome is an autoimmune inflammatory response, resulting in axonal demyelination.

A client has hypovolemic shock. Which early compensation mechanisms does the nurse expect? 1 Decreased heart rate 2 Decreased systemic vascular resistance 3 Release of epinephrine from adrenal glands 4 Release of antidiuretic hormone, causing water excretion

3In the early stage of shock, the adrenal medulla is stimulated to release increased amounts of the catecholamines epinephrine and norepinephrine, which circulate to the heart and stimulate contractility and heart rate. Heart rate initially increases. Systemic vascular resistance is not decreased in early hypovolemic shock. Antidiuretic hormone is released from the posterior pituitary gland and causes water retention.

A nurse is assessing a client with a suspected temporal extradural hematoma. Which assessment findings would be consistent with an extradural hematoma? 1 Progressive dementia with generalized rigidity 2 Slurred speech with drooping on one side of the face 3 Loss of consciousness at injury, followed by a lucid interval 4 Homonymous hemianopia followed by dysconjugate gaz

3Individuals with classic temporal extradural hematomas lose consciousness at injury; one third of those affected then become lucid for a few minutes to a few days (if a vein is bleeding). Most persons with a chronic subdural hematoma appear to have a progressive dementia with generalized rigidity (paratonia). Stroke victims may have slurred speech with drooping on one side of the face. Homonymous hemianopia (defective vision in either the right or the left field, occurring in both eyes), dysconjugate gaze, and gaze palsies may occur in an acute subdural hematoma.

A nurse is assessing a client with a suspected temporal extradural hematoma. Which assessment findings would be consistent with an extradural hematoma? 1 Progressive dementia with generalized rigidity 2 Slurred speech with drooping on one side of the face 3 Loss of consciousness at injury, followed by a lucid interval 4 Homonymous hemianopia followed by dysconjugate gaze

3Individuals with classic temporal extradural hematomas lose consciousness at injury; one third of those affected then become lucid for a few minutes to a few days (if a vein is bleeding). Most persons with a chronic subdural hematoma appear to have a progressive dementia with generalized rigidity (paratonia). Stroke victims may have slurred speech with drooping on one side of the face. Homonymous hemianopia (defective vision in either the right or the left field, occurring in both eyes), dysconjugate gaze, and gaze palsies may occur in an acute subdural hematoma.

A client has a type of diabetes that is caused by autosomal dominant mutations, affecting beta cell function or insulin action. Which diagnosis will the nurse most likely observe written in the chart? 1 Type 1 diabetes 2 Gestation diabetes 3 Maturity-onset diabetes of youth (MODY) 4 Drug-induced beta-cell dysfunctio

3MODY includes six specific autosomal dominant mutations that affect critical enzymes involved in beta-cell function or insulin action. Type 1 diabetes is beta cell destruction leading to absolute insulin deficiency. Gestation diabetes is any degree of glucose intolerance with onset of first recognition during pregnancy. Drug-induced beta-cell dysfunction commonly is associated with glucocorticoids and thiazide diuretics.

The nurse is preparing to administer medications to a client with a history of renal insufficiency. The nurse understands which category of drugs is most likely to cause a nephrotoxic acute tubular necrosis (ATN) in this client? 1 Vasodilators 2 Diuretics 3 Antibiotics 4 Narcotics

3Nephrotoxic ATN can be produced by radiocontrast media and numerous antibiotics, particularly the aminoglycosides (neomycin, gentamicin, tobramycin) because these drugs accumulate in the renal cortex. Diuretics, vasodilators, and narcotics do not cause ATN.

Which information from the client indicates teaching by the nurse was successful about the cause of Cushing syndrome? 1 Excessive production of antidiuretic hormone (ADH) 2 Excessive destruction of the adrenal cortex 3 Excessive production of cortisol 4 Excessive production of aldosterone

3One of the causes of Cushing syndrome is excessive production of cortisol. Cushing syndrome does not involve excessive production of ADH (syndrome of inappropriate ADH syndrome) or the destruction of the adrenal cortex (Addison disease). Cushing syndrome does not involve excessive production of aldosterone (hyperaldosteronism); excessive production of aldosterone is usually from an adenoma or from angiotensin II.

A nurse is teaching about a common cause of prerenal acute kidney injury. Which information should the nurse include in the teaching session? 1 One cause is prostatic hypertrophy. 2 One cause is neurogenic bladder. 3 One cause is inadequate cardiac output. 4 One cause is an allograft rejection

3Poor perfusion can result from renal vasoconstriction, hypotension, hypovolemia, hemorrhage, or inadequate cardiac output. A decreased cardiac output causes decreased renal perfusion and decreased glomerular filtration rate. An allograft rejection may cause intrarenal acute kidney injury, whereas neurogenic bladder and prostatic hypertrophy may cause postrenal acute kidney injury.

vA client has an infection of the renal pelvis, ureter, and interstitium. Which diagnosis will the nurse observe written on the chart? 1 Cystitis 2 Urethritis 3 Pyelonephritis 4 Glomerulonephritis

3Pyelonephritis is an infection of the renal pelvis, ureter, and interstitium. Glomerulonephritis includes an assortment of immune-mediated conditions that produce inflammation of glomeruli and other areas of the kidney. Urethritis is swelling and irritation (inflammation) of the urethra. The urethra is the tube that carries urine from the body. Cystitis is a term that refers to urinary bladder inflammation that can result from any of a number of distinct syndromes.

A nurse is assessing a patient with a peripheral nervous system disorder. The computed tomography scan shows injury to spinal roots caused by compression, inflammation, and direct trauma. Which disorder does the nurse expect to find in the chart? 1 Neuropathy 2 Plexus injury 3 Radiculopathy 4 Guillain-Barré syndrome

3Radiculopathy is a peripheral nervous system disorder that affects strength, tone, and bulk of the muscles innervated by the involved roots. Radiculopathy occurs due to injury to the spinal roots caused by compression, inflammation, and direct trauma. Therefore, the nurse will expect that the patient has radiculopathy. Neuropathy is a peripheral nervous system disorder caused by hereditary mechanisms, leprosy, or industrial solvents. While trauma and compression can cause plexus injuries, it involves the nerve plexus distal to spinal roots. Guillain-Barré syndrome is caused by an autoimmune inflammatory response causing axonal demyelination. Therefore, the nurse does not expect Guillain-Barré syndrome in this patient.

An annual physical assessment of a child reveals that the child has hypopituitarism affecting the growth hormone. The nurse would prepare the parents and child for which treatment option? 1 Parathyroid preparations 2 Surgical removal of the tumor 3 Replacement of target gland hormones 4 Palliative care because no treatment is know

3Replacement of target gland hormones, such as cortisol and growth hormone, is essential. Childhood growth deficiencies are not treated with parathyroid preparations, are not always related to tumors, and are not generally terminal.

A client has complications from a brain injury that occurred 3 months ago. What type of injury will the nurse observe written on the chart? 1 Primary 2 Clinical 3 Tertiary 4 Secondar

3Tertiary injury develops days or months later as a consequence of primary and secondary injury and can result from systemic complications, such as pneumonia, fever, infections, and immobility, that contribute to further brain injury. Primary injury is caused by impact, and secondary injury is an indirect consequence of the primary injury that involves a cascade of molecular and cellular events. Clinical is not a type of injury.

A client has hypoparathyroidism. The nurse is preparing to check the client for a Trousseau sign. Which piece of equipment should the nurse obtain? 1 Thermometer 2 Ophthalmoscope 3 Sphygmomanometer 4 None; all that is needed is the nurse's finger

3The Trousseau sign is elicited by sustained inflation of a sphygmomanometer placed on the upper arm to a level above the systolic blood pressure with resultant painful carpal spasm. A thermometer and ophthalmoscope would be ineffective. The Chvostek sign is elicited by tapping the cheek, resulting in twitching of the upper lip.

Which assessment is the priority if a client experiences a myasthenic crisis? 1 Cardiac 2 Vascular 3 Pulmonary 4 Neurologic

3The individual in myasthenic crisis is in danger of respiratory arrest. Although cardiac, vascular, and neurologic systems are necessary to assess, they are not the priority.

Which assessment finding in a client with a spinal cord injury will most alert the nurse that autonomic hyperreflexia may occur? 1 Dehydration 2 Stress and anxiety 3 Distended bladder 4 Bowel movemen

3The most common cause of autonomic hyperreflexia is a distended bladder or rectum. Dehydration usually does not cause autonomic dysreflexia. Stress and anxiety usually do not cause autonomic dysreflexia. Elimination of bowel and bladder contents often can relieve autonomic dysreflexia.

lient has hypoparathyroidism. Which treatment should the nurse implement? 1 Administration of potassium 2 Restriction of calcium 3 Administration of calcium 4 Restriction of potassium

3Treatment of hypoparathyroidism is directed toward alleviation of the hypocalcemia. Maintenance of the serum calcium level is achieved with pharmacologic doses of an active form of vitamin D and oral calcium, not potassium.

A client is in end-stage chronic renal disease. Which life-threatening assessment finding will cause the nurse to notify the primary healthcare provider immediately? 1 Azotemia 2 Hypertension 3 Hyperkalemia 4 Increased creatinin

3With the onset of oliguria, total body potassium can increase to life-threatening levels and must be controlled by dialysis. Although azotemia, hypertension, and increased creatinine are consequences of end-stage chronic renal disease, they usually are not acutely life-threatening like potassium.

A client has Guillain-Barré syndrome. When the nurse takes the history, which finding will be typical? 1 Had a tumor 2 Had spinal trauma 3 Had multiple blood transfusions 4 Had a viral respiratory infection

4

A client has deep vein thrombosis. Which action will the nurse take? 1 Prepare for sclerotherapy 2 Administer oral contraceptives 3 Prepare for vein stripping 4 Administer hepari

4

A client has a bone fracture that broke the bone into multiple fragments. How should the nurse report this type of fracture to the oncoming shift? 1 Open fracture 2 Stress fracture 3 Greenstick fracture 4 Comminuted fracture

4 A fracture in which the bone breaks into multiple fragments is called a comminuted fracture. An open fracture does not necessarily involve two or more bone fragments; it occurs when the bone has punctured the skin. A greenstick fracture does not involve two or more bone fragments; it is a break in one cortex of bone with splintering of the inner bone surface. A stress fracture occurs after repeated strain and is classified as an incomplete fracture.

Which client is demonstrating normal cerebral hemodynamics? 1 A client with an intracranial pressure of 20 mm Hg 2 A client with an intracranial pressure of 25 mm Hg 3 A client with a cerebral perfusion pressure of 50 mm Hg 4 A client with a cerebral perfusion pressure of 75 mm Hg

4 A normal range for cerebral perfusion pressure is 70 to 90 mm Hg. A normal range for intracranial pressure is 1 to 15 mm Hg.

A client presents with several hours of anuria with flank pain, followed by polyuria. Which diagnosis is the nurse most likely to find documented in the chart? 1 Ultrarenal acute kidney injury 2 Prerenal acute kidney injury 3 Intrarenal acute kidney injury 4 Postrenal acute kidney injury

4 A pattern of several hours of anuria with flank pain followed by polyuria is a characteristic finding of postrenal acute kidney injury and usually occurs with urinary tract obstruction. Prerenal has a high urine osmolality and a decreased urine sodium concentration with a high blood urea nitrogen (BUN)/creatinine ratio. Intrarenal has a low urine osmolality, and high urine sodium concentration with a low BUN/creatinine ratio. There is no classification for ultrarenal.

A client has acute glomerulonephritis. When the nurse reviews the laboratory report, which organism will the nurse most likely see? 1 Escherichia coli 2 Klebsiella 3 Staphylococcus 4 Streptococcu

4 Acute glomerulonephritis commonly results from inflammatory damage to the glomerulus as a consequence of immune reactions after a streptococcal infection. Although other bacteria have been implicated, streptococci are usually associated with this disorder.

A client has acute rheumatic fever. While taking the history, which prior infection is typical of this disease? 1 Fungal lung infection 2 Staphylococcus skin infection 3 Intestinal enterococci infection 4 Pharyngeal group A beta-streptococcus infection

4 Acute rheumatic fever can develop as a sequel to pharyngeal infection by group A beta-hemolytic streptococci. Bacteria are the most common cause of infective endocarditis, especially streptococci, staphylococci, and enterococci. A fungus can lead to infective endocarditis, but not to acute rheumatic fever.

hich finding will indicate to the nurse that a client is in the clonic phase of a seizure? 1 Loss of consciousness 2 Muscle contraction alternating with placidity 3 Muscle contraction with increased muscle tone 4 Alternating contraction and relaxation of muscle

4 Alternating muscle contraction and relaxation represents the clonic phase. The phase of muscle contraction with increased muscle tone is the tonic phase, which is associated with loss of consciousness. There is no phase that is characterized by muscle contraction alternating with placidity.

A nurse is caring for a client who has acute cholecystitis. Which information in the current medical record does the nurse recognize as most likely indicating an additional problem and should be brought to the attention of a primary healthcare provider? 1 Temperature: 99.4 degrees F 2 Epigastric pain, 7 on a 10-point scale 3 Serum bilirubin elevated 4 Serum amylase and lipase elevated

4 Amylase and lipase are pancreatic enzymes; elevation of these enzymes in the blood occurs with acute pancreatitis and the primary healthcare provider must be notified of this complication. Cholecystitis can lead to obstruction, which may also lead to reflux of bile into the pancreatic duct, causing acute pancreatitis. Fever may accompany cholecystitis. Both cholecystitis and acute pancreatitis can cause epigastric pain. Elevated serum bilirubin occurs with cholecystitis.

Which disorder occurs due to amyloidosis? 1 Hypopituitarism 2 Psychogenic polydipsia 3 Primary hyperthyroidism 4 Nephrogenic diabetes insipidus

4 Amyloidosis results in impaired renal function and nephrogenic diabetes insipidus. Hypopituitarism is caused by removal or destruction of the pituitary gland, head trauma, infections, or autoimmune hypophysitis. Amyloidosis does not lead to psychogenic polydipsia. Diseases like Graves' disease and solitary toxic adenoma cause primary hyperthyroidism.

A client has an intestinal obstruction. The nurse closely monitors the client for a perforated bowel. What is the rationale for the nurse's actions? 1 Osmotic forces from high particle content of static chyme draw extracellular fluid into the bowel wall. 2 Atelectasis causes severe inflammation, release of cytokines, and loss of bowel wall integrity. 3 Stasis of chyme allows bacterial overgrowth, causing inflammation and loss of bowel wall integrity. 4 Increased intraluminal pressure causes tissue hypoxia, ischemia, and loss of bowel wall integrity

4 An intestinal obstruction leads to perforation of the bowel when increased intraluminal pressure causes occlusion of the arterial circulation, leading to ischemia, necrosis, and perforation. Osmosis does not cause perforation of the bowel. Atelectasis can lead to pneumonia, not bowel perforation. Peritonitis or sepsis, not bowel perforation, can be directly caused by bacteria.

A client has a chronic inflammatory joint disease characterized by stiffening and fusion of the spine and sacroiliac joints. The nurse is caring for which client? 1 One with gout 2 One with osteoarthritis 3 One with rheumatoid arthritis 4 One with ankylosing spondylitis

4 Ankylosing spondylitis is a chronic, inflammatory joint disease characterized by stiffening and fusion of the spine and sacroiliac joints. Gout is a syndrome caused by incomplete purine metabolism, resulting in excess serum uric acid levels and inflammation and painful joints, but it does not cause fusion. Osteoarthritis is a common age-related disorder of synovial joints. Rheumatoid arthritis is a systemic inflammatory autoimmune disease associated with swelling and pain in multiple joints.

A client has upper motor neuron paralysis. What term will the nurse use to document this condition? 1 Diplegia 2 Hemiplegia 3 Quadriparesis 4 Spastic paralysis

4 Another term for upper motor neuron paralysis is spastic paralysis. Diplegia is paralysis of corresponding parts of both sides of the body. Hemiplegia is paralysis of the upper and lower extremities on one side. Quadriparesis is weakness of all four extremeties.

A client has high-output failure. Which action by the nurse is most appropriate? 1 Manage endocardial vegetations 2 Administer sodium 3 Manage hypothyroidism 4 Administer thiamine

4 Beriberi (thiamine deficiency) causes high-output failure. Thiamine must be replaced. Endocardial vegetations occur with infective endocarditis, not high-output failure. Sodium is not the treatment for high-output failure; in fact, sodium usually makes heart problems worse. Hyperthyroidism causes high-output failure, not hypothyroidism.

client has chorea and progressive loss of memory. Which medical diagnosis will the nurse observe written in the chart? 1 Epilepsy 2 Bulbar palsy 3 Alzheimer disease 4 Huntington disease

4 Clinical manifestations of Huntington disease include chorea and progressive loss of memory. Clients with a diagnosis of epilepsy (seizures) and bulbar palsy (cranial nerve involvement) do not exhibit signs and symptoms of chorea or memory loss. Clients with Alzheimer disease have progressive loss of memory but not chorea.

Which condition could indicate to the nurse that the client is experiencing one sign of systemic inflammatory response syndrome (SIRS)? 1 Temperature 37o C 2 Respiratory rate 16 3 Heart rate less than 60 beats/min 4 White blood cell count greater than 12,000 cells/mm

4 Criteria for SIRS include: white blood cell count >12,000 cells/mm3, <4000 cells/mm3, or containing <10% immature forms (bands), temperature greater than 38° C (not 37), respiratory rate > 20 breaths/min (not 16), heart rate > 90 beats/min (not less than 60), or arterial blood carbon dioxide level < 32 mm Hg.

nurse is teaching a health class about the most common site for cystitis. Which site should the nurse include in the teaching session? 1 Kidneys 2 Urethra 3 Ureters 4 Bladder

4 Cystitis (a urinary tract infection affecting the bladder) is the most common type of urinary tract infection. Infections of the kidneys, urethra, and ureters are not the most common.

Which client is at highest risk for developing dementia and mental status changes? 1 A 42-year-old client who has a history of spinal shock 2 A 58-year-old client who has upper motor neuron syndrome 3 A 65-year-old client who has a history of Guillain-Barré syndrome 4 A 74-year-old client who takes several medications to treat Parkinson disease

4 Dementia in clients with Parkinson disease is more common in clients older than 70 years; mental status changes are also more common in clients who take medications. Spinal shock, upper motor neuron syndrome, and Guillain-Barré syndrome do not affect mental status.

nurse is caring for clients with various types of dementia. Which assessment finding is consistent for all clients with dementias? 1 Seizures 2 Abrupt onset 3 Receptive dysphasia 4 Progressive deterioratio

4 Dementia is a progressive failure of many cerebral functions. Delirium, not dementia, includes an abrupt onset and possible seizures. Receptive (Wernicke) dysphasia is a disruption of verbal comprehension and repetition but not of verbal expression.

Which laboratory result should the nurse check to help determine the cause and severity of septic shock in a client? 1 Troponin 2 Calcium 3 Vasopressors 4 C-reactive protein

4 Determining the cause and severity of septic shock can be aided by measurement of levels of serum lactate, C-reactive protein, and procalcitonin. Troponin is used for myocardial infarctions, not for septic shock. Procalcitonin levels provide more insight for this purpose than do calcium levels. Vasopressors are a treatment for septic shock. It decreases when septic shock. is anticoagulant and anti inflammatory

A motor vehicle accident has caused a fracture in a client. The articular surfaces of the tibial and femoral condyles are no longer in contact with one another. What condition will the nurse be caring for in this client? 1 Malunion 2 Nonunion 3 Subluxation 4 Dislocation

4 Dislocation is the displacement of bone from its normal position to the extent that articulating surfaces entirely lose contact. Malunion results when healing leads to improper alignment of fracture fragments. Nonunion is failure of the bone ends to grow together. A subluxation is the displacement of bone from its normal joint position to the extent that articulating surfaces partially lose contact.

Which complication in a client with a hip fracture from osteoporosis would be most concerning to the nurse? 1 Hip pain 2 Kyphosis 3 Muscle spasms 4 Pulmonary embolu

4 Fatal complications of fractures include fat or pulmonary embolism, pneumonia, hemorrhage, and shock. Hip pain and muscle spasms are normal assessment findings with a fracture. Kyphosis (hunchback) is common in osteoporosis.

client has diabetes, smokes a pack of cigarettes daily, refuses to drink milk, and does not eat green leafy vegetables. A broken toe took twice as long as expected to heal. What term should nurses use when discussing the healing of this client's broken toe? 1 Malunion 2 Nonunion 3 Slow union 4 Delayed union

4 Fracture healing that does occur but takes longer than expected is called delayed healing or delayed union. Malunion involves fracture healing in an incorrect position. Nonunion occurs when the fracture does not heal. In this case, the fracture did heal, but the healing took longer than normal. Slow union is semantically accurate but is not the established term for delayed fracture healing.

client with cancer has granulocytopenia from chemotherapy. Which drug will the nurse prepare to administer? 1 Platelets 2 Erythropoietin 3 Packed red blood cells 4 Granulocyte colony-stimulating facto

4 Granulocytopenia and hence the risk of serious infection can be lessened by treatment with recombinant human granulocyte colony-stimulating factor (rhG-CSF, filgrastim). Platelets would be administered for thrombocytopenia. Erythropoietin and packed red blood cells would be administered for anemia.

A nurse is checking a client's laboratory results. Which values for total cholesterol level, high-density lipoprotein (HDL) level, and low-density lipoprotein (LDL) level would indicate to the nurse that the levels are elevated? 1 Total cholesterol level of 40 mg/dl, HDL of 200 mg/dl, and LDL of 160 mg/dl 2 Total cholesterol level of 90 mg/dl, HDL of 160 mg/dl, and LDL of 200 mg/dl 3 Total cholesterol level of 200 mg/dl, HDL of 160 mg/dl, and LDL of 40 mg/dl 4 Total cholesterol level of 250 mg/dl, HDL of 65 mg/dl, and LDL of 165 mg/dl

4 High values for cholesterol levels are as follows: total cholesterol level over 240 mg/dl, HDL level over 60 mg/dl, and LDL level over 160 mg/dl. The other values are either too high or too low. In addition, the options in which the sum of the HDL and LDL values far exceeds the total cholesterol value are not realistic, because the total includes both HDL and LDL (as well as VLDL).

Which client will most likely need emergency surgery? 1 A client who has diverticulosis 2 A client who has diabetic gastroparesis 3 A client who has irritable bowel syndrome 4 A client who has a strangulated hernia

4 Immediate surgical intervention is required for complete obstruction, strangulation, or perforation. A strangulated hernia needs emergency surgery to restore the blood supply or it will develop gangrene. A client with diverticulosis does not need surgical repair. A client with diabetic gastroparesis does not need surgery but requires dietary management, prokinetic drugs, and, in some cases, gastric electrical stimulation. Irritable bowel syndrome is not treated with surgery; care is individualized, because there is no cure.

A client is having detrusor hyperreflexia with vesicosphincter dyssynergia. What physiologic process is the client experiencing? 1 Kidney stones cause the bladder to fail. 2 Bladder fullness is sensed, but the detrusor fails to contract. 3 The bladder protrudes into the vagina and blocks the bladder outlet. 4 The bladder and the external sphincter contract simultaneously

4 In detrusor hyperreflexia with vesicosphincter dyssynergia, there is loss of pontine coordination of detrusor muscle contraction and external sphincter relaxation, so both the bladder and the sphincter are contracting at the same time, causing a functional obstruction of the bladder outlet. Kidney stones do not cause detrusor hyperreflexia with vesicosphincter dyssynergia. Both the detrusor muscle and the bladder contract; it is not a matter of the detrusor failing to contract. Pelvic organ prolapse, not detrusor hyperreflexia with vesicosphincter dyssynergia, causes the bladder to protrude into the vagina (cystocele).

The nurse teaches a client about mitral valve prolapse syndrome. Which information from the client indicates teaching was effective? 1 The mitral valve is absent from birth. 2 The mitral valve is simultaneously hard and contracted. 3 The mitral valve closes prematurely, reducing blood flow. 4 The mitral valve balloons back into the left atrium during contraction.

4 In mitral valve prolapse syndrome, one or both of the cusps of the mitral valve billow upward (prolapse) into the left atrium during systole (contraction). Mitral valve prolapse is not congenital absence of the valve or a simultaneously stenosed (hard and contracted) valve. Mitral valve prolapse does not involve premature valve closure.

A client with shock has an increased heart rate, vasoconstriction (increased systemic vascular resistance), and movement of interstitial fluid into the vascular compartment. Which type of shock will the nurse observe documented on the chart? 1 Septic 2 Neurogenic 3 Anaphylactic 4 Hypovolemic

4 In response to hypovolemia, sympathetic nervous system activation causes tachycardia and vasoconstriction, while fluid shifts from the interstitial compartment into the blood to preserve blood volume and pressure. Septic shock, neurogenic shock, and anaphylactic shock all manifest with vasodilation.

A nurse is caring for a client with reflux esophagitis. Which pathophysiologic process should the nurse consider when planning care for this client? 1 Congenital anomaly of the esophagus 2 Dysplasia of the epithelial lining of the esophagus 3 Autoimmune destruction of the esophageal mucosa 4 Inflammatory response to gastric contents in the esophagus

4 In some individuals, a combination of factors causes an inflammatory response to reflux called reflux esophagitis. Reflux esophagitis is not a congenital anomaly. It is not an autoimmune process or dysplasia of the epithelial lining of the esophagus.

A client has colorectal cancer of the descending colon. Which assessment findings will the nurse typically observe? 1 Reduced red blood cells 2 Loss of appetite for meat 3 Dark red blood mixed with stools 4 Narrow and pencil-shaped stoo

4 In tumors of the left colon, including the descending colon, obstruction is common but occurs slowly, and stools become narrow and pencil-shaped. Colorectal cancer of the right colon has anemia (reduced red blood cells) and dark red or mahogany-colored blood mixed with stools. A loss of appetite for meat occurs in gastric cancer.

nurse monitors a client with end-stage chronic renal disease for anemia. What is the best rationale for the nurse's action? 1 Chronic loss of blood in the urine 2 Increased secretion of aldosterone 3 Poor appetite with lack of iron intake 4 Decreased secretion of erythropoieti

4 Inadequate production of erythropoietin decreases red blood cell production, and uremia decreases red blood cell life span. As a result, anemia develops. Hematuria is not a characteristic of end-stage chronic renal disease. Aldosterone levels do not contribute to anemia. Iron deficiency does not cause the anemia in end-stage chronic renal disease. Poor appetite and lack of iron intake may occur in this client, but they are not the best reason for the nurse's action.

A client has cholecystitis. Which physiologic process should the nurse consider as the cause of the client's condition? 1 A fatty diet 2 Chronic gastritis 3 Spicy food intake 4 Inflammatory reactions

4 Inflammation of the gallbladder or cystic duct is known as cholecystitis. The continued presence of gallstones within the gallbladder ultimately promotes inflammatory changes in the gallbladder wall, with fibrosis and thickening. A fatty diet does not cause cholecystitis, but it can aggravate the condition. Chronic gastritis is not associated with cholecystitis. Spicy food may not be tolerated well, but that is not the cause of cholecystitis.

Which adult client should the nurse monitor for the development of valvular stenosis or valvular regurgitation of the mitral or aortic valves? 1 A client with heart failure 2 A client with syphilis infection 3 A client with myocardial infarction 4 A client with rheumatic heart disease

4 Inflammatory damage from rheumatic heart disease can cause either valvular stenosis, through scarring, or valvular regurgitation, through leaflet stretching. Syphilis infection is associated primarily with aortic regurgitation. Myocardial infarction does not lead to valvular stenosis or regurgitation. Heart failure is a consequence, not a cause, of valvular disease.

A client has osteoarthritis. Which pathophysiologic process should the nurse remember when planning care? 1 Widening of the joint space is attributable to cartilage loss and bone spurs. 2 Decalcification of articular cartilage is a complex interaction of enzymes and matrix molecules. 3 Proteoglycan content is increased in cartilage, affecting the strength of the cartilage. 4 Interleukin-1 and tumor necrosis factor play a major role in cartilage degradation.

4 Interleukin-1 and tumor necrosis factor play a major role in cartilage degradation. Advancing disease shows narrowing of the joint space attributable to cartilage loss and bone spurs. Calcification of articular cartilage occurs in osteoarthritis. Proteoglycan content is decreased in cartilage and is a hallmark of the osteoarthritic process.

The nurse is reviewing laboratory results for a 25-year-old woman experiencing urinary frequency, urgency, and dysuria. The nurse is concerned the client has interstitial cystitis based on which laboratory result? 1 Urine culture demonstrating Escherichia coli 2 Urinalysis demonstrating an acidic pH 3 Urinalysis demonstrating cloudy urine 4 Urine culture demonstrating no bacteria

4 Interstitial cystitis occurs most commonly in women aged 20 to 30 years who have symptoms of cystitis, such as frequency, urgency, dysuria, and nocturia, but with negative urine cultures and no other known cause. Urine pH is supposed to be acidic, and cloudy urine is associated with a urinary tract infection. Interstitial cystitis would have a negative urine culture, not a culture with E coli, which would indicate a urinary tract infection.

What is the initial treatment for a client with malignant hyperthermia? 1 Succinylcholine 2 Bisphosphonates 3 Volatile anesthetics 4 Dantrolene sodium (Dantrium

4 Malignant hyperthermia and myoglobinuria can be treated by infusing dantrolene sodium. Succinylcholine and volatile anesthetics are contributing factors to malignant hyperthermia; if given, these would cause the condition to get worse. Bisphosphonates are first-line medications for treating osteoporosis, not malignant hyperthermia.

young client has nephrotic syndrome. "Tell me what causes the urine to be so full of protein," says the parent. Which pathophysiologic mechanism should be the basis for the nurse's response? 1 The renal tubules are full of cellular debris. 2 The liver is extremely active in synthesizing protein. 3 Many glomeruli have been destroyed by autoantibodies from the immune system. 4 Glomerular basement membrane disturbances have increased permeability to protein.

4 Massive proteinuria occurs in nephrotic syndrome because the glomerular basement membrane disturbances have led to increased permeability to protein. The liver is active in synthesizing proteins, but this does not describe why protein is present in the urine. The tubules are not damaged in nephrotic syndrome and thus should not contain cellular debris. Glomerulonephritis, not nephrotic syndrome, is from an immune response.

Which piece of equipment should the nurse obtain to best monitor a client with complicated hypertension for early signs of impending renal dysfunction? 1 Pulse oximeter 2 Blood pressure cuff 3 Stethoscope 4 Urine test kit

4 Microalbuminuria (small amounts of protein in the urine) is now recognized as an early sign of impending renal dysfunction. A urine test kit can detect it. A blood pressure cuff and stethoscope are good for measuring blood pressure, but they do not indicate the damage to the kidneys. A pulse oximeter will determine oxygen saturation but not the damage to the kidneys.

A client is diagnosed with obstructive sleep apnea syndrome (OSAS). Which clinical manifestations should the nurse expect? 1 Somnambulism, night terrors, and insomnia 2 Hallucinations, sleep paralysis, and cataplexy 3 Hypotension, prickling sensations, and depression 4 Apneic episodes, polycythemia, and hypersomnia

4 OSAS is associated with apneic episodes, polycythemia, and hypersomnia (excessive daytime sleepiness). OSAS and insomnia are classified as dyssomnias; somnambulism and night terrors are classified as parasomnias. Hallucinations, sleep paralysis, and cataplexy are associated with narcolepsy. Hypertension is associated with OSAS, not hypotension.

"My mother didn't get enough vitamin D for the last few years, and now she has a bone condition with a strange name," says a client. "Please remind me what it is called." How should the nurse respond? 1 This is called rickets. 2 This is called subluxation. 3 This is called osteoporosis. 4 This is called osteomalacia

4 Osteomalacia is caused by vitamin D deficiency in adults. Rickets is caused by vitamin D deficiency in children. Subluxation is partial dislocation. Osteoporosis is loss of bone density.

A client has osteomalacia. Which information indicates the nurse has a correct understanding of the pathophysiology of osteomalacia? 1 Increased osteoclast activity 2 Crowding of cells in the osteoid 3 Collagen breakdown in the bone matrix 4 Inadequate mineralization in the osteoid

4 Osteomalacia is characterized by inadequate or delayed mineralization in the osteoid in mature compact and spongy bone. Osteomalacia does not involve increased osteoclast activity, collagen breakdown in the bone matrix, or crowding of cells in the osteoid.

A nurse is asked what the difference is between osteoporosis and osteopenia. How should the nurse respond? 1 Women get osteoporosis and men get osteopenia. 2 These two words mean the same thing: severely infected bone mass. 3 Osteoporosis is a moderately reversible inflammatory condition, but osteopenia is a chronic irreversible inflammatory condition. 4 Osteoporosis is severely reduced bone mass, but osteopenia is more moderately reduced bone mass.

4 Osteoporosis is severely reduced bone mass, but osteopenia is more moderately reduced bone mass. Gender does not differentiate osteoporosis and osteopenia. Osteoporosis and osteopenia are not the same condition, although both relate to bone mass. Neither relates to infection. Both osteoporosis and osteopenia are reversible, and both can be treated with medication. Neither is directly related to inflammation.

A client has a disorder that is characterized by softening and enlargement of bones. The nurse is caring for a client with which condition? 1 Rickets 2 Osteoporosis 3 Osteomyelitis 4 Paget disease

4 Paget disease is characterized by excessive bone resorption and formation, causing fractures and deformities. Osteoporosis is a loss of bone density. Osteomyelitis is a bone infection with inflammation. Rickets does have softening but involves no enlargement of bones.

A nurse is asked what causes pain in fibromyalgia. How should the nurse respond? 1 The pain is from nerve inflammation. 2 The pain is from muscle inflammation. 3 The pain is from autoimmune destruction of muscle tissue. 4 The pain is from changes in pain transmission in the spinal cord

4 Pain in fibromyalgia involves changes in pain transmission in the spinal cord that are called central sensitization. Pain in fibromyalgia is not caused by nerve or muscle inflammation. Autoimmune destruction of muscle tissue does not cause the pain in fibromyalgia.

A client is admitted to the emergency room with pain in the costovertebral angle. What does the nurse suspect as the most likely cause, based on the location of the pain? 1 Peptic ulcer 2 Herniated disc 3 Arthritis of the spine 4 Urinary tract infection

4 Pain located at the costovertebral angle of the back may signal the presence of a urinary tract infection, specifically acute pyelonephritis. Peptic ulcer pain is not associated with the costovertebral angle of the back. Herniated disc pain is not generally associated with the costovertebral angle of the back. Arthritis of the spine is not usually associated with pain in the costovertebral angle of the back.

A client has ascites from cirrhosis of the liver. Which factor contributes to ascites in this client? 1 Increased capillary oncotic pressure 2 Decreased hydrostatic pressure 3 Decreased capillary permeability 4 Increased arterial vasodilatio

4 Peripheral vasodilation, associated with increased nitric oxide produced by the diseased liver, decreases effective circulating blood volume, activating aldosterone and antidiuretic hormone, which promote renal sodium and water retention, thereby accelerating portal hypertension and ascites formation. Decreased, not increased, capillary oncotic pressure leads to ascites. Increased, not decreased, capillary permeability leads to ascites. Increased, not decreased, hydrostatic pressure leads to ascites.

A client presents with chest pain that occurs at rest, and abnormal vasospasm of the coronary arteries is detected. The nurse will most likely observe which diagnosis on the chart? 1 Stable angina 2 Silent ischemia 3 Angina pectoris 4 Prinzmetal angina

4 Prinzmetal angina is an abnormal vasospasm of the coronary vessels. It typically produces pain at rest. Stable angina occurs with activity and is often limited and resolves with rest. Silent ischemia is occlusion of the coronary arteries without pain. Angina pectoris is chest pain, usually substernal chest discomfort, and includes stable and unstable angina variants.

Which assessment question demonstrates an understanding of a possible cause of achalasia? 1 "Are you chronically constipated?" 2 "Do you become 'gassy' after meals?" 3 "What types of high-fiber foods do you eat?" 4 "Have you been under a lot of stress lately?

4 Psychosocial achalasia has been documented and may be the result of life stressors. Constipation does not occur with achalasia; achalasia leads to difficulty swallowing. Becoming "gassy" after meals is not associated with achalasia. An intake of high-fiber foods would help prevent constipation but does not relate to achalasia. A rare disorder making it difficult for food and liquid to pass into the stomach.

A client is admitted for suspected rheumatoid arthritis (RA). Which diagnostic test is recommended for diagnosing RA? 1 Complete blood count 2 Dual x-ray absorptiometry 3 Serum alkaline phosphatase 4 Anti-citrullinated protein antibody (ACPA

4 Research has found that the autoantibodies rheumatoid factor and ACPA can be present for years to decades before synovial or radiographic involvement becomes apparent. ACPA is a much more specific serum marker for rheumatoid arthritis than rheumatoid factor. Serum alkaline phosphatase levels are elevated in bone lytic tumors and significantly elevated in osteosarcoma. Dual x-ray absorptiometry is the current gold standard for detecting and monitoring osteoporosis. Complete blood count does not help diagnose rheumatoid arthritis.

n older adult diagnosed with osteoporosis falls, sustains blunt trauma to the hip, and is not found for nearly two days. What is likely to occur because of this delay in treatment? 1 Legg-Calve'-Perthes (LCP) disease of the hip 2 Hematogenous osteomyelitis where blood flow is sluggish 3 Bundles of myofibrils rearranged to parallel along the longitudinal axis 4 Increased levels of serum creatine kinase (CK) and myoglobinuri

4 Rhabdomyolysis, which causes increased levels of serum CK and myoglobinuria, is a result of blunt trauma or crush injury. LCP disease is a common osteochondrosis usually occurring in children (3--10 years of age) involving areas of significant tensile or compressive stress. Hematogenous osteomyelitis in the metaphysis where blood flow is sluggish is most common in children. The excessive compression from immobility results in the breakdown of muscle and releases myoglobin into the blood stream, which is filtered by the kidneys; it does not cause bundles of myofibrils to rearrange..

Which type of glomerular lesion will the nurse expect in the patient whose diagnostic report shows scarring between the glomerulus and tubules? 1 Mesangial 2 Crescentic 3 Membranous 4 Interstitial fibrosi

4 Scarring between the glomerulus and tubules is the characteristic sign of interstitial fibrosis. Therefore, the nurse will expect that the patient has interstitial fibrosis. If the diagnostic report shows deposits of immunoglobulins in mesangial matrix, the nurse will expect a mesangial type of glomerular lesion. If the diagnostic report shows an accumulation of proliferating cells within the Bowman space making a crescent appearance, the nurse will expect a crescentic type of glomerular lesion. If the diagnostic report shows thickening of the glomerular capillary wall with immune deposits, the nurse will expect a membranous type of glomerular lesion.

Which type of glomerular lesion will the nurse expect in the patient whose diagnostic report shows scarring between the glomerulus and tubules? 1 Mesangial 2 Crescentic 3 Membranous 4 Interstitial fibrosis

4 Scarring between the glomerulus and tubules is the characteristic sign of interstitial fibrosis. Therefore, the nurse will expect that the patient has interstitial fibrosis. If the diagnostic report shows deposits of immunoglobulins in mesangial matrix, the nurse will expect a mesangial type of glomerular lesion. If the diagnostic report shows an accumulation of proliferating cells within the Bowman space making a crescent appearance, the nurse will expect a crescentic type of glomerular lesion. If the diagnostic report shows thickening of the glomerular capillary wall with immune deposits, the nurse will expect a membranous type of glomerular lesion.

Which assessment finding should indicate to the nurse that the client is experiencing severe glomerulonephritis? 1 No protein in the urine 2 Clear urine 3 Nocturia 4 Oliguria

4 Severe or progressive glomerular disease causes oliguria (urine output of 30 ml/hr or less), hypertension, and renal failure. Two major symptoms distinctive of more severe glomerulonephritis are (1) hematuria with red blood cell casts and (2) proteinuria exceeding 3 to 5 g/d with albumin (macroalbuminuria) as the major protein. No protein in the urine and clear urine are normal findings. Nocturia is voiding at night and is not indicative of severe glomerulonephritis.

A nurse is asked to define spinal shock. How should the nurse respond? 1 Slow destruction of efferent pathways 2 Presence of hyperactive deep tendon reflexes 4 Increased spinal reflexes resulting in a hypertensive crisis 4 Complete cessation of spinal cord function below the lesio

4 Spinal shock is the complete cessation of spinal cord function below the lesion. Other characteristics of spinal shock include the absence of reflexes (not hyperactive or increased) and sudden destruction of efferent pathways (not slow).

A client with dermatomyositis has Gottron lesions. Which area should the nurse check for these lesions? 1 Eyelids 2 Nose 3 Thigh 4 Knee

4 The two most classic signs of skin involvement for dermatomyositis are rashes: a typical heliotrope (reddish purple) rash that generally covers the eyelids and periorbital tissue; and erythematous, scaly lesions that cover joints such as the knees and elbows, known as Gottron lesions. Gottron lesions cover joints, not the nose, eyelids, or thigh.

Which information should the nurse consider to help distinguish between stress incontinence and urge incontinence in clients? 1 Stress incontinence occurs with dementia, whereas urge incontinence occurs with immobility. 2 Stress incontinence occurs during sleep, whereas urge incontinence occurs during the waking hours. 3 Stress incontinence is caused by a detrusor muscle problem, whereas urge incontinence is caused by neurologic lesions below sacral nerve 1 (S1). 4 Stress incontinence is caused by increased abdominal pressure, whereas urge incontinence is caused by a problem with the detrusor muscle

4 Stress incontinence is caused by increased abdominal pressure, whereas urge incontinence is caused by a problem with the detrusor muscle. Functional incontinence occurs with involuntary loss of urine attributable to dementia or immobility. Definitions of stress and urge incontinence do not include the time of day that they occur. Overflow incontinence is associated with neurologic lesions below S1.

What term should the nurse use to describe the presence of a higher concentration of a salt within a fluid than the volume is able to dissolve to maintain equilibrium? 1 Inhibition 2 Precipitation 3 Crystallization 4 Supersaturation

4 Supersaturation is the presence of a higher concentration of a salt within a fluid (in this case, the urine) than the volume is able to dissolve to maintain equilibrium. Crystallization is the process by which crystals grow from a small nidus or nucleus to larger stones in the presence of supersaturated urine. Precipitation is the formation of salts from a liquid to a solid state. Inhibition stops the process of precipitation, preventing subsequent stone formation.

A client has fibromyalgia. Which assessment finding is a unique characteristic feature of fibromyalgia? 1 Headaches 2 Contractures 3 Muscle atrophy 4 Tender point pain

4 Tender point pain is a unique characteristic feature of fibromyalgia. Headaches, contractures, and muscle atrophy are not specific to fibromyalgia. Widespread muscle pain and tenderness

Which clinical manifestation will the nurse expect to find in a client with prolonged upper urinary tract obstruction? 1 Elevated serum sodium level 2 Metabolic alkalosis 3 Fluid overload 4 Elevated serum potassium level

4 The affected kidney is unable to conserve sodium, bicarbonate, and water or to excrete hydrogen or potassium, leading to metabolic acidosis (not metabolic alkalosis) and dehydration (not fluid overload). The loss of bicarbonate and the excess of hydrogen and potassium leads to metabolic acidosis and elevated serum potassium levels. Because the kidney cannot conserve sodium, bicarbonate, or water, these are lost in the urine. Hydrogen and potassium cannot be excreted, so they build up in the blood. The loss of water and sodium leads to dehydration and low levels of sodium.

Which information indicates the nurse has a correct understanding of the results of shock in a client? 1 Shock causes a decrease in catecholamine release. 2 Shock enables unlimited use of lactic acid for cardiac muscles. 3 Shock leads to metabolic alkalosis from anaerobic metabolism. 4 Shock of any type causes impairment of cellular metabolism

4 The final common pathway in shock of any type is impairment of cellular metabolism. Shock causes an increase in catecholamine release. A compensatory mechanism enables cardiac and skeletal muscles to use lactic acid as a fuel source, but only for a limited time. In addition to decreasing adenosine triphosphate stores, anaerobic metabolism affects the pH of the cell, and metabolic acidosis develops.

Which information indicates the nurse has an accurate understanding of the function of viral protease in an HIV virus? 1 It makes a DNA copy of viral RNA. 2 It assists budding of new HIV virions. 3 It incorporates viral DNA into host cell DNA. 4 It processes newly synthesized viral proteins

4 The function of viral protease is to process newly synthesized viral proteins. Integrase incorporates viral DNA into host cell DNA. Reverse transcriptase makes a DNA copy of viral RNA. Integrase does not assist budding of new HIV virions or separate newly synthesized viral proteins.

client has acute post-streptococcal glomerulonephritis and asks the nurse why the urine is pink. How should the nurse respond? 1 When parts of your kidneys stopped working, your blood kept flowing and broke some of your little blood vessels, so red blood cells are flowing into your urine and making it pink. 2 Normally, red blood cells that enter the urine are taken back into the blood, but in glomerulonephritis, the kidney disease you have, they stay in the urine and make it pink. 3 The bacteria that caused your disease have traveled to your kidneys and are causing minor damage there that allows some red blood cells to leak into your urine and make it pink. 4 Your immune system was activated and has caused some damage in the urinary tract that allows red blood cells to leak into the fluid that becomes urine, making it pink

4 The immune system damages glomeruli in post-streptococcal glomerulonephritis. Bleeding from sites lower in the urinary tract produces pink or red urine. Broken blood vessels are not part of the pathophysiology of post-streptococcal glomerulonephritis. Blood cells do not normally enter urine. The immune response, not the bacteria, causes this damage.

A client has portal hypertension. Which is the most common clinical manifestation the nurse will find upon assessment? 1 Ascites 2 Altered mental status 3 Abdominal pain from splenomegaly 4 Vomiting blood from esophageal varice

4 The most common clinical manifestation of portal hypertension is vomiting of blood from bleeding esophageal varices. Other problems related to portal hypertension include splenomegaly, ascites, and hepatic encephalopathy (producing altered mental status), but these are not more common than vomiting blood.

client has portal hypertension. Which is the most common clinical manifestation the nurse will find upon assessment? 1 Ascites 2 Altered mental status 3 Abdominal pain from splenomegaly 4 Vomiting blood from esophageal varice

4 The most common clinical manifestation of portal hypertension is vomiting of blood from bleeding esophageal varices. Other problems related to portal hypertension include splenomegaly, ascites, and hepatic encephalopathy (producing altered mental status), but these are not more common than vomiting blood.

A nurse is caring for a client with amyotrophic lateral sclerosis (ALS). Which pathophysiologic process should the nurse consider when planning care for this client? 1 Degeneration of skeletal muscle tissue 2 Degeneration of myelin in the central nervous system 3 Degeneration of the neuromuscular junction 4 Degeneration of upper and lower motor neurons

4 The principal pathologic feature of ALS is lower and upper motor neuron degeneration, although without inflammation. ALS is not caused by degeneration of skeletal muscle or degeneration of myelin, which are characteristic of muscular dystrophy and multiple sclerosis, respectively. The neuromuscular junction is not degenerated in ALS, but blocking of the receptors at the neuromuscular junction is characteristic of myasthenia gravis.

A 6-year-old client presents with polyarthritis, and the parents report no medical history except for a sore throat 4 weeks ago. On physical examination, a murmur is noted. Which diagnosis is supported by the assessment data? 1 Lyme disease 2 Reiter syndrome 3 Rheumatoid arthritis 4 Acute rheumatic feve

4 This child most likely has acute rheumatic fever. It most commonly affects those 5 to 15 years old. It is an inflammatory disease that follows group A β-hemolytic streptococcal pharyngeal infection. The incubation period is 1 to 5 weeks. Rheumatoid arthritis commonly affects women, with a peak incidence in the fourth to sixth decade of life. Arthritis with Lyme disease from a tick bite can occur, but generally it affects the knee and a large effusion is present. Reiter syndrome is generally found after an infection of the gastrointestinal tract.

A nurse is checking laboratory results to help determine whether a client has primary or secondary hypothyroidism. Which laboratory result is best for the nurse to check? 1 T3 2 T4 3 Thyroxine 4 Thyroid-stimulating hormone

4 Thyroid-stimulating hormone (TSH) levels are helpful in differentiating between primary (high TSH) and secondary (low TSH) causes of hypothyroidism. Serum T3 or T4 levels do not help differentiate between primary and secondary hypothyroidism. Thyroxine is also known as T4.

Which information from a coworker would indicate teaching by the nurse was successful for identifying a major risk factor for the development of acute pyelonephritis? 1 Glomerulonephritis 2 Respiratory disease 3 Nephrotic syndrome 4 Vesicoureteral reflux

4 Urinary obstruction and reflux of urine from the bladder (vesicoureteral reflux) are the most common underlying risk factors for acute pyelonephritis. Glomerulonephritis, respiratory disease, and nephrotic syndrome are not risk factors for pyelonephritis.

A client is diagnosed with vasogenic cerebral edema after a traumatic brain injury. What information should the nurse include when teaching the client about the pathophysiology of vasogenic cerebral edema? 1 Bleeding of cranial vessels results in a severe decrease in intracranial pressure. 2 Brain swelling causes cerebrospinal fluid to flow more readily through the ventricles. 3 Toxins that are released cause brain cells to transport potassium in exchange for sodium. 4 When the blood-brain barrier is disrupted, plasma proteins leak into extracellular spaces

4 Vasogenic cerebral edema is caused by increased capillary permeability when the vessels in the brain are injured; this causes a disruption of the blood-brain barrier, and plasma proteins leak into the extracellular spaces. Cranial bleeding results in a severe increase (not decrease) in intracranial pressure. Interstital edema (not vasogenic cerebral edema) is caused by movement of cerebrospinal fluid from the ventricles into the extracellular spaces of the brain tissue. Cytotoxic edema (not vasogenic cerebral edema) results from toxins that are released and cause brain cells to lose intracellular potassium and gain larger amounts of sodium.

ient has jaundice due to increased unconjugated bilirubin. The nurse is caring for which client? 1 A client with gallstones 2 A client with hypertension 3 A client with blocked bile duct 4 A client with hemolytic anemi

4 With hemolytic anemia, unconjugated bilirubin accumulates faster than the liver is able to conjugate it. Gallstones do not cause jaundice unless the stones cause a blockage leading to jaundice from conjugated bilirubin. Hypertension is not associated with jaundice. With bile duct obstruction, the liver conjugates bilirubin but is unable to excrete it in the backed-up bile. The conjugated bilirubin accumulates, causing jaundice.

Which assessment finding should the nurse expect to observe in a woman diagnosed with prolactinoma? 1 Exophthalmos 2 Enlarged tongue 3 Heat intolerance 4 Galactorrhea

4 Women with hyperprolactinemia from a prolactinoma generally present with galactorrhea (nonpuerperal milk production) and menstrual disturbances including amenorrhea. Heat intolerance occurs with thyrotoxicosis, not prolactinoma. Exophthalmos, large and protruding eyeballs, occurs in hyperthyroidism. Acromegaly is associated with enlarged tongue.

A radiograph shows a fracture where one cortex is perforated and the spongy bone is splintered. Which type of fracture will the nurse be caring for in this client? 1 Stress 2 Impacted 3 Comminuted 4 Greenstick

4 vGreenstick fractures cause an incomplete break in the bone where one cortex is perforated and the spongy bone is splintered. They usually occur in children. Comminuted fractures present with multiple bone fragments. Impacted fractures have one fragment driven into another. A stress fracture is a failure of one cortical surface of the bone, often caused by repetitive activity such as running.

Which macrovascular complications of diabetes mellitus should the nurse monitor for in a client with diabetes? 1 Retinopathy 2 Nephropathy 3 Autonomic neuropathy 4 Cerebrovascular accident

4 Macrovascular complications of diabetes mellitus are defined as damage to large and medium-sized arteries providing circulation to the brain, heart, and extremities, which leads to coronary artery disease, cerebrovascular accident (stroke), and peripheral vascular disease. Retinopathy, nephropathy, and autonomic neuropathy are related to microvascular complications.

A nurse is assessing a client with acute kidney injury. Which assessment findings are characteristic of this condition? Select all that apply. 1 Paresthesia 2 Crystals in the urine 3 Increased serum calcium 4 Decreased glomerular filtration 5 Accumulation of nitrogenous waste products in the bloo

45 Acute kidney injury is a sudden decline in kidney function, with a decrease in glomerular filtration and accumulation of nitrogenous waste products in the blood, as demonstrated by an elevation in plasma creatinine and blood urea nitrogen (BUN) levels. Increased serum calcium is not characteristic of acute kidney injury; increased creatinine and BUN are. Crystals in the urine are not characteristic of acute kidney injury. Paresthesia is not characteristic of renal failure; increased creatinine and BUN are.

client has a central nervous system injury that is causing vomiting. Which information should the nurse remember when planning care? Select all that apply. 1 Vomiting can result from decompression of the brain stem. 2 Vomiting can result from a decrease in intracranial pressure. 3 Vomiting can result from impingement on the third ventricle. 4 Vomiting can result from involvement of the medulla oblongata. 5 Vomiting can result from impingement directly on the floor of the fourth ventricle

45Vomiting that is associated with central nervous system injuries involves the vestibular nuclei or their immediate projections or the medulla oblongata. Vomiting can be caused by an impingement on the fourth ventricle, not on the third ventricle. It can also be the result of brain stem compression from an increase in intracranial pressure.

The nurse is caring for a patient who has moderate blunt trauma due to diffuse axonal injury. Which complication does the nurse anticipate in the patient? 1 Blindness 2 Paraplegia 3 Prolonged coma 4 Retrograde amnesi

4A patient with moderate blunt trauma caused by a diffuse axonal injury may have a classic cerebral concussion, which is characterized by retrograde and anterograde amnesia. A moderate focal injury associated with occipital damage results in blindness. Paraplegia is a focal injury associated with moderate blunt trauma, not a diffuse axonal injury. Severe diffuse axonal injury results in a prolonged coma.

client presents with several hours of anuria with flank pain, followed by polyuria. Which diagnosis is the nurse most likely to find documented in the chart? 1 Ultrarenal acute kidney injury 2 Prerenal acute kidney injury 3 Intrarenal acute kidney injury 4 Postrenal acute kidney injury

4A pattern of several hours of anuria with flank pain followed by polyuria is a characteristic finding of postrenal acute kidney injury and usually occurs with urinary tract obstruction. Prerenal has a high urine osmolality and a decreased urine sodium concentration with a high blood urea nitrogen (BUN)/creatinine ratio. Intrarenal has a low urine osmolality, and high urine sodium concentration with a low BUN/creatinine ratio. There is no classification for ultrarenal.

A nurse is caring for a client with meningitis. Which information indicates a nurse has an accurate understanding about meningitis? 1 Fungal meningitis is very common. 2 Aseptic meningitis is most commonly caused by a fungus. 3 Bacterial meningitis is a primary infection of the gray matter. 4 Viral meningitis generally has milder symptoms than bacterial meningitis

4Aseptic (viral or nonpurulent meningitis) has similar symptoms to bacterial meningitis, but milder. Fungal meningitis is uncommon. Aseptic meningitis is most commonly caused by a virus. Bacterial meningitis is caused by primary infection of the pia mater and arachnoid.

Which client is most predisposed to postrenal acute kidney injury? 1 A client with glomerulonephritis 2 A client with severe hypotension 3 A client with acute tubular necrosis 4 A client with bilateral kidney stones

4Bilateral kidney stones can cause postrenal acute kidney injury. Glomerulonephritis does not cause postrenal acute kidney injury, because the glomeruli are located within the kidneys; it is classified as intrarenal injury. Severe hypotension can cause prerenal acute kidney injury but does not cause postrenal acute kidney injury. Acute tubular necrosis does not cause postrenal acute kidney injury, because the renal tubules are located within the kidneys; it is classified as intrarenal injury.

A client was injured 6 weeks ago in a diving accident and has been diagnosed with a spinal cord injury at the level of the fourth cervical vertebra. The client starts to have a pounding headache, bradycardia, and sweating above the level of the lesion. Which complication does the nurse suspect? 1 Spinal shock 2 Focal brain injury 3 Horner syndrome 4 Autonomic hyperreflexi

4Characteristics of autonomic hyperreflexia include paroxysmal hypertension (up to 300 mm Hg, systolic), a pounding headache, blurred vision, and sweating above the level of the lesion with flushing of the skin. In spinal shock, reflex function is completely lost in all segments below the lesion. Focal brain injury occurs when a specific area of the brain is injured. Horner syndrome causes ptosis of the affected eyeball and lack of perspiration on the ipsilateral side of the face.

A client has basal ganglion damage. Which muscle tone alteration will the nurse observe upon assessment? 1 Alpha 2 Dystonia 3 Flaccidity 4 Cogwhee

4Cogwheel is a type of hypertonia caused by basal ganglion damage. Alpha is caused by a loss of cerebellum input to the lateral vestibular nuclei; dystonia is produced by slow muscle contractions or lack of reciprocal inhibition of muscle. Flaccidity occurs when nerve impulses for muscle tone are lost.

After assessing a patient with brain injury, the nurse finds that the patient has contrecoup injury. What possible cause of injury does the nurse suspect? 1 The injury was caused by a missile. 2 The injury was caused by a sharp projectile. 3 The injury was caused by falls due to chronic alcohol abuse. 4 The injury was caused by a forceful impact on the opposite site

4Contrecoup injury is a focal brain injury that occurs on a particular spot in the brain. The nurse suspects that the patient's injury is on the side opposite the area of the brain that was impacted. If the patient has penetrating trauma, then the nurse suspects that the patient was injured by a missile. If the patient has penetrating trauma, then the nurse suspects that the cause of injury may have been a sharp projectile. If the patient has subdural hematoma, then the nurse suspects the injury may have been caused by falls due to chronic alcohol abuse.

Which pathologic change is associated with stage III diabetic retinopathy? 1 Formation of microaneurysm 2 Progression of retinal ischemia 3 Increased retinal capillary permeability 4 Fibrous tissue formation within the retin

4During stage III of diabetic retinopathy, fibrous tissue is formed within the retina or optic disc. Microaneurysm formation is associated with stage I of diabetic retinopathy. The progression of retinal ischemia is associated with stage II of diabetic retinopathy. An increase in retinal capillary permeability is associated with stage I of diabetic retinopathy.

A nurse has been caring for a client with a brain abscess for 11 days. The brain abscess has a decreased necrotic center with a mature collagen capsule. Which stage of brain abscess formation is the client experiencing? 1 Late cerebritis 2 Early cerebritis 3 Late capsule formation 4 Early capsule formatio

4Early capsule formation occurs on days 10 to 13. During this stage, the necrotic center decreases in size and mature collagen evolves, forming a capsule. Late cerebritis occurs on days 4 to 9, and there is a necrotic center surrounded by inflammatory infiltrate of macrophages and fibroblasts with rapid new blood vessel formation. Early cerebritis occurs on days 1 to 3 and is a localized inflammatory process with marked cerebral edema and a central core of necrosis. Late capsule formation occurs on day 14 and later with a well-formed necrotic center surrounded by a dense collagenous capsule.

The diagnoses on a client's chart include gastroparesis in addition to type 1 diabetes. Which information from the client indicates that the gastroparesis is symptomatic today? 1 I am weak, and this wound will not heal. 2 I have diarrhea today; please give me the bedpan. 3 I have that burning pain in my legs and feet again. 4 I can't eat this bedtime snack; I'm still full from dinner

4Gastroparesis is delayed emptying of the stomach. Delayed wound healing is not a symptom of gastroparesis. Constipation, not diarrhea, would occur with delayed emptying of the stomach. Burning pain in the legs and feet is common with peripheral neuropathy, not with gastroparesis. Gastroparesis results from autonomic neuropathy.

When a nurse is planning care for clients with grade III and grade IV astrocytomas, which information should the nurse consider? 1 Grade III and IV astrocytomas are slow-growing tumors that form cavities. 2 Grade III and IV astrocytomas are generally seen more in women than in men. 3 Grade III and IV astrocytomas are usually necrotic, owing to the absence of vascularity. 4 Grade III and IV astrocytomas are commonly found in the frontal lobe and cerebral hemisphe

4Grades III and IV astrocytomas are found predominantly in the frontal lobes and cerebral hemispheres and occur twice as often in men as in women. Grades I and II (not III and IV) are slow-growing tumors that may form cavities. Astrocytomas are highly vascular and extensively infiltrative.

A nurse is teaching the staff about the difference between diabetic ketoacidosis (DKA) and hyperosmolar hyperglycemic nonketotic syndrome (HHNKS). Which information should the nurse include in the teaching session? 1 Glucose levels are considerably higher in DKA than in HHNKS because of volume excess. 2 Glucose levels are considerably higher in DKA than in HHNKS because of volume depletion. 3 The degree of insulin deficiency is more profound in HHNKS, and the degree of fluid deficiency is more marked in DKA. 4 The degree of insulin deficiency is more profound in DKA, and the degree of fluid deficiency is more marked in HHNKS.

4HHNKS differs from DKA in the degree of insulin deficiency (which is more profound in DKA) and the degree of fluid deficiency (which is more marked in HHNKS). Glucose levels are considerably higher in HHNKS than in DKA because of volume depletion.

A client with acquired immunodeficiency syndrome (AIDS) has chronic neuroinflammation and neurodegeneration. Which condition related to the human immunodeficiency virus (HIV) will the nurse find documented on the chart? 1 HIV myelopathy 2 HIV demyelinating disorder 3 HIV peripheral neuropathy 4 HIV-associated dementia

4In HIV-associated dementia, HIV-infected macrophages and monocytes from the blood accumulate in the brain by up-regulation of proinflammatory mediators that enable activated macrophages and monocytes to penetrate the blood-brain barrier, promoting chronic neuroinflammation and neurodegeneration. Papovavirus may produce a demyelinating disorder. Myelopathy involving diffuse degeneration of the spinal cord may occur in persons with AIDS (HIV myelopathy). HIV has been isolated from peripheral nerves; consequently, the virus may directly infect nerves and cause neuropathy (HIV peripheral neuropathy).

A client is having detrusor hyperreflexia with vesicosphincter dyssynergia. What physiologic process is the client experiencing? 1 Kidney stones cause the bladder to fail. 2 Bladder fullness is sensed, but the detrusor fails to contract. 3 The bladder protrudes into the vagina and blocks the bladder outlet. 4 The bladder and the external sphincter contract simultaneously

4In detrusor hyperreflexia with vesicosphincter dyssynergia, there is loss of pontine coordination of detrusor muscle contraction and external sphincter relaxation, so both the bladder and the sphincter are contracting at the same time, causing a functional obstruction of the bladder outlet. Kidney stones do not cause detrusor hyperreflexia with vesicosphincter dyssynergia. Both the detrusor muscle and the bladder contract; it is not a matter of the detrusor failing to contract. Pelvic organ prolapse, not detrusor hyperreflexia with vesicosphincter dyssynergia, causes the bladder to protrude into the vagina (cystocele).

Which sign or symptom should the nurse assess as the most critical clinical index of a client's nervous system function? 1 Pupillary changes 2 Oculomotor responses 3 Patterns of breathing 4 Level of consciousness

4Level of consciousness is the most critical clinical index of nervous system function, with changes indicating either improvement or deterioration of the individual's condition. Oculomotor responses (resting, spontaneous, and reflexive eye movements) change at various levels of brain dysfunction in comatose individuals. Patterns of breathing help evaluate the level of brain dysfunction and coma. Pupillary changes indicate the presence and level of brain stem dysfunction, because brain stem areas that control arousal are adjacent to areas that control the pupils.

A client has incontinence manifested by an involuntary loss of urine with overdistention of the bladder. Which type of incontinence will the nurse observe written on the chart? 1 Urge incontinence 2 Mixed incontinence 3 Stress incontinence 4 Overflow incontinenc

4Overflow incontinence results from urinary retention and overdistended bladder secondary to obstruction or polyneuropathies. Urge incontinence involves the involuntary leakage of urine along with or immediately after the sensation of a need to urinate (urgency). Stress incontinence is precipitated by effort or exertion, such as by lifting heavy objects, or by coughing or sneezing. Mixed incontinence is a combination of both stress and urge incontinence.

While assessing a patient, the nurse finds that the patient has paresis of the legs, paralysis of the eye muscles, and respiratory insufficiency. The laboratory reports indicate that the patient has respiratory viral infection. What condition should the nurse anticipate to find in the patient? 1 Neuropathies 2 Plexus injuries 3 Radiculopathies 4 Guillain-Barré syndrome

4Paresis of the legs, paralysis of the eye muscles, and respiratory insufficiency indicate that the patient more than likely has Guillain-Barré syndrome. Guillain-Barré syndrome is usually caused by autoimmune inflammatory response or a viral respiratory infection. Neuropathies are characterized by paralysis of the foot muscles, legs, and arms, but not the eyes. Plexus injuries are associated with motor weakness, muscle atrophy, and sensory loss. Radiculopathies are associated with the absence of deep tendon reflexes from injury to spinal roots as they exit or enter the vertebral canal.

Which condition does the nurse suspect in the patient who reports a pricking and burning sensation in the extremities as well as cramps, fasciculations, and atrophy, which the nurse finds on assessment? 1 Astrocytoma 2 Neurofibroma 3 Oligodendroglioma 4 Radicular syndrome

4Radicular syndrome (irritative syndrome) combines the symptoms of cord compression with radicular pain. The patient with radicular syndrome has impaired pain and touch perceptions, and motor disturbances including cramps, atrophy, and fasciculations. Therefore, the nurse suspects radicular syndrome. Astrocytoma is the most common type of glioma; the onset of a focal seizure disorder suggests astrocytoma. Therefore, the nurse does not expect astrocytoma in this patient. Neurofibroma is suspected if the patient has hearing loss or deafness. Oligodendroglioma is suspected if the patient has a focal or generalized seizure.

hich condition does the nurse suspect in the patient who reports a pricking and burning sensation in the extremities as well as cramps, fasciculations, and atrophy, which the nurse finds on assessment? 1 Astrocytoma 2 Neurofibroma 3 Oligodendroglioma 4 Radicular syndrom

4Radicular syndrome (irritative syndrome) combines the symptoms of cord compression with radicular pain. The patient with radicular syndrome has impaired pain and touch perceptions, and motor disturbances including cramps, atrophy, and fasciculations. Therefore, the nurse suspects radicular syndrome. Astrocytoma is the most common type of glioma; the onset of a focal seizure disorder suggests astrocytoma. Therefore, the nurse does not expect astrocytoma in this patient. Neurofibroma is suspected if the patient has hearing loss or deafness. Oligodendroglioma is suspected if the patient has a focal or generalized seizure.

Which finding in a client with subarachnoid hemorrhage will cause the nurse to notify the primary healthcare provider immediately? 1 Mild headache 2 Low back pain 3 Negative Brudzinski sign 4 Sudden increase in blood pressur

4Rebleeding is manifested by a sudden increase in blood pressure and intracranial pressure, along with a deteriorating neurologic status. Rebleeding is a significant risk with a high mortality (up to 70%). A mild headache and low back pain are not indications to call the primary healthcare provider immediately, because these can be expected. A negative Brudzinski sign would indicate the hemorrhage has improved; a positive result indicates nuchal rigidity and meningeal irritation.

Which finding in a client with subarachnoid hemorrhage will cause the nurse to notify the primary healthcare provider immediately? 1 Mild headache 2 Low back pain 3 Negative Brudzinski sign 4 Sudden increase in blood pressure

4Rebleeding is manifested by a sudden increase in blood pressure and intracranial pressure, along with a deteriorating neurologic status. Rebleeding is a significant risk with a high mortality (up to 70%). A mild headache and low back pain are not indications to call the primary healthcare provider immediately, because these can be expected. A negative Brudzinski sign would indicate the hemorrhage has improved; a positive result indicates nuchal rigidity and meningeal irritation.

Which findings in the patient's diagnostic report are consistent with spondylolisthesis? 1 Bulging disk and facet hypertrophy 2 Thick ossified posterior longitudinal ligament 3 Defect of the lamina or neural arch of the vertebra 4 Vertebra sliding forward and onto the vertebra below

4Spondylolisthesis occurs when a vertebra slides forward and onto the vertebra below it; therefore, the nurse expects that the patient has spondylolisthesis. If the findings showed a bulging disk and facet hypertrophy, the nurse would identify the defect as spinal stenosis. If the findings showed a thick ossified posterior longitudinal ligament, the nurse would identify the defect as spinal stenosis. If the report showed a developmental defect of the spine involving the lamina or neural arch of the vertebra, the nurse would identify the defect as spondylolysis.

A nurse is describing spasticity, gegenhalten, dystonia, and rigidity. What is the nurse discussing? 1 Paratonia 2 Hypokinesia 3 Paralysis 4 Hypertonia

4The four types of hypertonia are spasticity, gegenhalten (paratonia), dystonia, and rigidity. Paratonia is resistance to passive movement, which varies in direct proportion to the force applied. Types of hypokinesia include akinesia, bradykinesia, and loss of associated movement. Paralysis is loss of motor function so that a muscle group is unable to overcome gravity.

A client has been admitted with a diagnosis of bacterial meningitis with a severely depressed level of consciousness. The spouse wants to know why this illness makes the client so sleepy and difficult to awaken. How should the nurse respond? 1 The brain tissue cells have become infected and swollen, pushing out too much fluid and causing an imbalance. 2 The infection causes the passages by which this fluid is removed to dilate, thereby causing brain cells to shrivel and die. 3 The infection causes an overproduction of this fluid, so that more fluid is produced than can be removed. This process puts pressure on brain cells, causing them to undergo metaplasia. 4 The infection blocks the passages by which this fluid is removed, thereby causing it to accumulate. This accumulation of fluid puts pressure on brain cells and prevents them from functioning normally.

4The inflammatory exudate thickens the cerebrospinal fluid (CSF) and interferes with normal CSF flow around the brain and spinal cord, possibly obstructing arachnoid villi and producing hydrocephalus. Meningeal cells become edematous, and the combined exudate and edematous cells increase intracranial pressure, which decreases the level of consciousness. Bacterial meningitis does not decrease the level of consciousness by pushing fluid out of brain cells nor by dilation of arachnoid villi, metaplasia, or increased production of CSF.

A nurse teaches a client with multiple sclerosis about prevention of short-lived attacks of neurologic deficits. Which information from the client indicates teaching was successful? 1 These are completely irreversible. 2 Sunbathing in hot weather is helpful. 3 Calcium intake should be increased with meals. 4 Emotional and physical stress should be avoided

4Triggering events of short-lived attacks of neurologic deficits include hypercalcemia and physical and emotional stress, and they should be avoided. Short-lived attacks are reversible. An increase in temperature will cause an attack; thus sunbathing in hot weather is discouraged. Calcium increases will cause an attack.

Which symptoms support the nurse's suspicion that a patient is suffering from migraine? Select all that apply. A Leg weakness B Phonophobia C Throbbing pain D Unilateral head pain E Generalized convulsions

BCDMigraine is a neurologic disorder characterized by repeated, episodic headache. One symptom of migraine is phonophobia, which is fear of sounds. The other symptoms include throbbing and unilateral pain. Leg weakness is a clinical manifestation seen in patients with human immunodeficiency virus (HIV) myelopathy. Generalized convulsions are a clinical manifestation of meningoencephalitis, which caused by West Nile virus.

posturing

Decerebrate posturing is demonstrated by abnormal extension. The arms are extended, with hyperpronation of the forearms and plantar extension of the feet. Coma is not characterized by physical characteristics but rather by deviations in consciousness. Decorticate posturing is characterized by abnormal flexion of the arms, wrists, and fingers with adduction in upper extremities. The lower extremities experience extension, internal rotation, and plantar flexion.

Which information indicates the nurse has a correct understanding of the cause for acromegaly in adult clients and giantism in children? 1 Hyperparathyroidism 2 Asymptomatic hypercalcemia 3 Adenoma in the pituitary gland 4 Pituitary response to excessive calcium intak

Growth hormone excess is nearly always caused by a growth-hormone-secreting pituitary adenoma. Growth hormone has significant effects on glucose, lipid, and protein metabolism, not calcium. Hyperparathyroidism causes hypercalcemia and hypophosphatemia. Hypercalcemia does not affect growth hormone, nor does excessive calcium intake.

Upon assessment, a nurse found that a client with osteoarthritis has Heberden nodes. Which picture best illustrates what the nurse observed?

Heberden nodes are enlargement and bulging of the joint contour on the distal joints (end joints) of the fingers in osteoarthritis. Bouchard nodes can also occur with osteoarthritis, but these appear on the proximal joints (middle joints) of the fingers. Nodes located below the finger joints occur with rheumatoid arthritis and are called subcutaneous nodes. When the hand shifts toward the ulnar bone, the deformation is called ulnar drift, which also occurs with rheumatoid arthritis.

What are the signs and symptoms of syndrome of inappropriate antidiuretic hormone (SIADH)? Select all that apply. A Polyuria b Dyspnea c Anorexia D Coarse skin e Dulled sensorium

bceA patient with SIADH has dyspnea, anorexia, dulled sensorium, impaired taste, and fatigue. Polyuria is a symptom of diabetes insipidus. Coarse skin is a symptom of acromegaly.

Which diseases cause primary hyperthyroidism? Select all that apply. A Meningitis B Cystic fibrosis C Graves' disease D Solitary toxic adenoma E Toxic multinodular goite

cde Graves' disease, solitary toxic adenoma, and toxic multinodular goiter are the common diseases that impair the function of the thyroid gland. Meningitis and cystic fibrosis cause syndrome of inappropriate antidiuretic hormone (SIADH).


Related study sets

Strategic Management: Chapter 1 T/F

View Set

Module 4 Chemical Reactions and Calculations

View Set

Strategic Management Chapter 5 and 6

View Set

Sr Med Surg Prep U Ch 23: Mgmnt of Pts w/Chest & Lower Respiratory Tract Disorders

View Set

Introduction to the Internet of Things

View Set

Networking Essentials Practice Final

View Set

OB Unit 1 Test (Chapter 1, 3, 4)

View Set

crónica de una muerte anunciada

View Set